SlideShare a Scribd company logo
1 of 1085
Download to read offline
A 53 year old man undergoes a reversal of a loop colostomy. He recovers well and is
discharged home. He is readmitted 10 days later with symptoms of vomiting and
colicky abdominal pain. On examination he has a swelling of the loop colostomy site
and it is tender. What is the most likely underlying diagnosis?
A. Haematoma
B. Intra abdominal adhesions
C. Anastomotic leak
D. Anastomotic stricture
E. Obstructed incisional hernia
Theme from September 2011 Exam
In this scenario the most likely diagnosis would be obstructed incisional hernia. The
tender swelling coupled with symptoms of obstruction point to this diagnosis. Prompt
surgical exploration is warranted. Loop colostomy reversals are at high risk of this
complication as the operative site is at increased risk of the development of post
operative wound infections.
Acute incisional hernia
 Any surgical procedure involving entry into a cavity containing viscera may
be complicated by post operative hernia
 The abdomen is the commonest site
 The deep layer of the wound has usually broken down, allowing internal
viscera to protrude through
 Management is dictated by the patients clinical status and the timing of the
hernia in relation to recent surgery
 Bowel obstruction or tenderness at the hernia site both mandate early surgical
intervention to reduce the risk of bowel necrosis
 Mature incisional hernias with a wide neck and no symptoms may be either
left or listed for elective repair
 Risk factors for the development of post operative incisional hernias include
post operative wound infections, long term steroid use, obesity and chronic
cough
Theme: Abdominal stomas
A. End ileostomy
B. End colostomy
C. Loop ileostomy
D. Loop colostomy
E. End jejunostomy
F. Loop jejunostomy
G. Caecostomy
For each of the following scenarios, please select the most appropriate type of stoma
to be constructed. Each option may be selected once, more than once or not at all.
2. A 56 year old man is undergoing a low anterior resection for carcinoma of the
rectum. It is planned to restore intestinal continuity.
You answered End colostomy
The correct answer is Loop ileostomy
Colonic resections with an anastomosis below the peritoneal reflection may
have an anastomotic leak rate (both clinical and radiological) of up to 15%.
Therefore most surgeons will defunction such an anastomosis to reduce the
clinical severity of an anastomotic leak. A loop ileostomy will achieve this end
point and is relatively easy to reverse.
3. A 23 year old man with uncontrolled ulcerative colitis is undergoing an
emergency sub total colectomy.
You answered Loop ileostomy
The correct answer is End ileostomy
Following a sub total colectomy the immediate surgical options include an end
ileostomy or ileorectal anastomosis. In the emergency setting an ileorectal
anastomosis would be unsafe.
4. A 63 year old women presents with large bowel obstruction. On examination
she has a carcinoma 10cm from the anal verge.
You answered End colostomy
The correct answer is Loop colostomy
Large bowel obstruction resulting from carcinoma should be resected, stented
or defunctioned. The first two options typically apply to tumours above the
peritoneal reflection. Lower tumours should be defunctioned with a loop
colostomy and then formal staging undertaken prior to definitive surgery. An
emergency attempted rectal resection carries a high risk of involvement of the
circumferential resection margin and is not recommended.
Abdominal stomas
Stomas may be sited during a range of abdominal procedures and involve bringing the
lumen or visceral contents onto the skin. In most cases this applies to the bowel.
However, other organs or their contents may be diverted in case of need.
With bowel stomas the type method of construction and to a lesser extent the site will
be determined by the contents of the bowel. In practice, small bowel stomas should be
spouted so that their irritant contents are not in contact with the skin. Colonic stomas
do not need to be spouted as their contents are less irritant.
In the ideal situation the site of the stoma should be marked with the patient prior to
surgery. Stoma siting is important as it will ultimately influence the ability of the
patient to manage their stoma and also reduce the risk of leakage. Leakage of stoma
contents and subsequent maceration of the surrounding skin can rapidly progress into
a spiraling loss of control of stoma contents.
Types of stomas
Name of stoma Use Common sites
Gastrostomy  Gastric decompression or fixation
 Feeding
Epigastrium
Loop
jejunostomy
 Seldom used as very high output
 May be used following
emergency laparotomy with
planned early closure
Any location according to
need
Percutaneous
jejunostomy
 Usually performed for feeding
purposes and site in the proximal
bowel
Usually left upper
quadrant
Loop ileostomy  Defunctioning of colon e.g.
following rectal cancer surgery
 Does not decompress colon (if
ileocaecal valve competent)
Usually right iliac fossa
End ilestomy  Usually following complete
excision of colon or where ileo-
colic anastomosis is not planned
 May be used to defunction colon,
but reversal is more difficult
Usually right iliac fossa
End colostomy Where a colon is diverted or resected and
anastomosis is not primarily achievable
or desirable
Either left or right iliac
fossa
Loop
colostomy
 To defunction a distal segment of
colon
 Since both lumens are present the
distal lumen acts as a vent
May be located in any
region of the abdomen,
depending upon colonic
segment used
Caecostomy Stoma of last resort where loop
colostomy is not possible
Right iliac fossa
Mucous fistula  To decompress a distal segment
of bowel following colonic
division or resection
 Where closure of a distal
resection margin is not safe or
achievable
May be located in any
region of the abdomen
according to clinical need
Theme: Acute abdominal pain
A. Ruptured abdominal aortic aneurysm
B. Perforated peptic ulcer
C. Perforated appendicitis
D. Mesenteric infarction
E. Small bowel obstruction
F. Large bowel obstruction
G. Pelvic inflammatory disease
H. Mesenteric adenitis
I. Pancreatitis
J. None of the above
Please select the most likely cause of abdominal pain for the scenario given. Each
option may be used once, more than once or not at all.
5. A 75 year old man is admitted with sudden onset severe generalised abdominal
pain, vomiting and a single episode of bloody diarrhoea. On examination he
looks unwell and is in uncontrolled atrial fibrillation. Although diffusely tender
his abdomen is soft.
Mesenteric infarction
In mesenteric infarction there is sudden onset of pain together with vomiting
and occasionally passage of bloody diarrhoea. The pain present is usually out of
proportion to the physical signs.
6. A 19 year old lady is admitted with lower abdominal pain. On examination she
is diffusely tender. A laparoscopy is performed and at operation multiple fine
adhesions are noted between the liver and abdominal wall. Her appendix is
normal.
You answered Small bowel obstruction
The correct answer is Pelvic inflammatory disease
This is Fitz Hugh Curtis syndrome in which pelvic inflammatory disease
(usually Chlamydia) causes the formation of fine peri hepatic adhesions.
7. A 78 year old man is walking to the bus stop when he suddenly develops severe
back pain and collapses. On examination he has a blood pressure of 90/40 and
pulse rate of 110. His abdomen is distended and he is obese. Though tender his
abdomen itself is soft.
Ruptured abdominal aortic aneurysm
This will be a retroperitoneal rupture (anterior ones generally don't survive to
hospital). The debate regarding CT varies, it is the authors opinion that a
systolic BP of <100mmHg at presentation mandates immediate laparotomy.
Acute mesenteric ischaemia- Pain out of
proportion to the physical signs.
Atrial fibrillation is often present.
Fitz Hugh Curtis = Fine Hepatic
Connections
Acute abdominal pain-diagnoses
Conditions presenting with acute abdominal pain
Condition Features Investigations Management
Appendicitis History of
migratory pain.
Fever.
Anorexia.
Evidence of right
iliac fossa
tenderness.
Mild pyrexia.
Differential white cell
count
Pregnancy test
C-Reactive protein
Amylase
Urine dipstick testing
Appendicectomy
Mesenteric
adenitis
Usually recent
upper respiratory
tract infection.
High fever.
Generalised
Full blood count- may
show slightly raised
white cell count
Urine dipstick often
normal
Conservative
management-
appendicectomy if
diagnostic doubt
abdominal
discomfort- true
localised pain and
signs are rare.
Abdominal ultrasound
scan - usually no free
fluid
Mittelschmerz Only seen in
females.
Mid cycle pain.
Usually occurs two
weeks after last
menstrual period.
Pain is usually has
a supra-pubic
location.
Usually subsides
over a 24-48 hour
period.
Full blood count-
normal
Urine dipstick- normal
Abdominal and pelvic
ultrasound- may show a
trace of pelvic free fluid
Manage conservatively
if doubt or symptoms
fail to settle then
laparoscopy
Fitz-Hugh
Curtis
syndrome
Disseminated
infection with
Chlamydia.
Usually seen in
females.
Consists of
evidence of pelvic
inflammatory
disease together
with peri-hepatic
inflammation and
subsequent
adhesion formation.
Abdominal ultrasound
scan- may show free
fluid
High vaginal swabs -
may show evidence of
sexually transmitted
infections
Usually medically
managed- doxycycline
or azithromycin
Abdominal
aortic
aneurysm
(ruptured)
Sudden onset of
abdominal pain
radiating to the
back in older adults
(look for risk
factors).
Collapse.
May be moribund
on arrival in
casualty, more
stable if contained
haematoma.
Careful clinical
assessment may
reveal pulsatile
mass.
Patients who are
haemodynamically
stable should have a CT
scan
Unstable patients
should undergo
immediate surgery
(unless it is not in their
best interests).
Those with evidence of
contained leak on CT
should undergo
immediate surgery
Increasing aneurysmal
size is an indication for
urgent surgical
intervention (that can
wait until the next
working day)
Perforated
peptic ulcer
Sudden onset of
pain (usually
epigastric).
Often preceding
Erect CXR may show
free air. A CT scan may
be indicated where there
is diagnostic doubt
Laparotomy
(laparoscopic surgery
for perforated peptic
ulcers is both safe and
history of upper
abdominal pain.
Soon develop
generalised
abdominal pain.
On examination
may have clinical
evidence of
peritonitis.
feasible in experienced
hands)
Intestinal
obstruction
Colicky abdominal
pain and vomiting
(the nature of
which depends on
the level of the
obstruction).
Abdominal
distension and
constipation (again
depending upon
site of obstruction).
Features of
peritonism may
occur where local
necrosis of bowel
loops is occurring.
A plain abdominal film
may help with making
the diagnosis. A CT
scan may be useful
where diagnostic
uncertainty exists
In those with a virgin
abdomen and lower and
earlier threshold for
laparotomy should exist
than in those who may
have adhesional
obstruction
Mesenteric
infarction
Embolic events
present with sudden
pain and forceful
evacuation.
Acute on chronic
events usually have
a longer history and
previous weight
loss.
On examination the
pain is typically
greater than the
physical signs
would suggest.
Arterial pH and lactate
Arterial phase CT
scanning is the most
sensitive test
Immediate laparotomy
and resection of
affected segments, in
acute embolic events
SMA embolectomy
may be needed.
Theme: Gastrointestinal bleeding
A. Haemorroids
B. Meckels diverticulum
C. Angiodysplasia
D. Colonic cancer
E. Diverticular bleed
F. Ulcerative colitis
G. Ischaemic colitis
Please select the most likely cause of colonic bleeding for the scenario given. Each
option may be used once, more than once or not at all
8. A 73 year old lady is admitted with a brisk rectal bleed. She is otherwise well
and the bleed settles. On examination her abdomen is soft and non tender.
Elective colonoscopy shows a small erythematous lesion in the right colon,
but no other abnormality.
Angiodysplasia
Angiodysplasia can be difficult to identify and treat. The colonoscopic
stigmata are easily missed by poor bowel preparation.
9. A 23 year old man complains of passing bright red blood rectally. It has been
occurring over the past week and tends to occur post defecation. He also
suffers from pruritus ani.
Haemorroids
Classical haemorroidal symptoms include bright red rectal bleeding, it
typically occurs post defecation and is noticed on the toilet paper and in the
toilet pan. It is usually painless, however, thrombosed external haemorroids
may be very painful.
10. A 63 year old man presents with episodic rectal bleeding the blood tends to be
dark in colour and may be mixed with stool. His bowel habit has been erratic
since an abdominal aortic aneurysm repair 6 weeks previously.
Ischaemic colitis
The inferior mesenteric artery may have been ligated and being an arteriopath
collateral flow through the marginal may be imperfect.
Gastrointestinal bleeding
Colonic bleeding
This typically presents as bright red or dark red blood per rectum. Colonic bleeding
rarely presents as malaena type stool, this is because blood in the colon has a powerful
laxative effect and is rarely retained long enough for transformation to occur and
because the digestive enzymes present in the small bowel are not present in the colon.
Up to 15% of patients presenting with haemochezia will have an upper
gastrointestinal source of haemorrhage.
As a general rule right sided bleeds tend to present with darker coloured blood than
left sided bleeds. Haemorrhoidal bleeding typically presents as bright red rectal
bleeding that occurs post defecation either onto toilet paper or into the toilet pan. It is
very unusual for haemorrhoids alone to cause any degree of haemodynamic
compromise.
Causes
Cause Presenting features
Colitis Bleeding may be brisk in advanced cases, diarrhoea is commonly
present. Abdominal x-ray may show featureless colon.
Diverticular
disease
Acute diverticulitis often is not complicated by major bleeding and
diverticular bleeds often occur sporadically. 75% all will cease
spontaneously within 24-48 hours. Bleeding is often dark and of
large volume.
Cancer Colonic cancers often bleed and for many patients this may be the
first sign of the disease. Major bleeding from early lesions is
uncommon
Haemorrhoidal
bleeding
Typically bright red bleeding occurring post defecation. Although
patients may give graphic descriptions bleeding of sufficient
volume to cause haemodynamic compromise is rare.
Angiodysplasia Apart from bleeding, which may be massive, these arteriovenous
lesions cause little in the way of symptoms. The right side of the
colon is more commonly affected.
Management
 Prompt correction of any haemodynamic compromise is required. Unlike
upper gastrointestinal bleeding the first line management is usually supportive.
This is because in the acute setting endoscopy is rarely helpful.
 When haemorrhoidal bleeding is suspected a proctosigmoidoscopy is
reasonable as attempts at full colonoscopy are usually time consuming and
often futile.
 In the unstable patient the usual procedure would be an angiogram (either CT
or percutaneous), when these are performed during a period of haemodynamic
instability they may show a bleeding point and may be the only way of
identifying a patch of angiodysplasia.
 In others who are more stable the standard procedure would be a colonoscopy
in the elective setting. In patients undergoing angiography attempts can be
made to address the lesion in question such as coiling. Otherwise surgery will
be necessary.
 In patients with ulcerative colitis who have significant haemorrhage the
standard approach would be a sub total colectomy, particularly if medical
management has already been tried and is not effective.
Indications for surgery
Patients > 60 years
Continued bleeding despite endoscopic intervention
Recurrent bleeding
Known cardiovascular disease with poor response to hypotension
Surgery
Selective mesenteric embolisation if life threatening bleeding. This is most helpful if
conducted during a period of relative haemodynamic instability. If all haemodynamic
parameters are normal then the bleeding is most likely to have stopped and any
angiography normal in appearance. In many units a CT angiogram will replace
selective angiography but the same caveats will apply.
If source of colonic bleeding unclear perform a laparotomy, on table colonic lavage
and following this attempt a resection. A blind sub total colectomy is most unwise, for
example bleeding from an small bowel arterio-venous malformation will not be
treated by this manoeuvre.
Summary of Acute Lower GI bleeding recommendations
Consider admission if:
* Over 60 years
* Haemodynamically unstable/profuse PR bleeding
* On aspirin or NSAID
* Significant co morbidity
Management
 All patients should have a history and examination, PR and proctoscopy
 Colonoscopic haemostasis aimed for in post polypectomy or diverticular
bleeding
References
http://www.sign.ac.uk/guidelines/fulltext/105/index.html
heme: Surgical signs
A. Rovsing's sign
B. Boas' sign
C. Psoas stretch sign
D. Cullen's sign
E. Grey-Turner's sign
F. Murphy's sign
G. None of the above
Please select the most appropriate eponymous abdominal sign for the scenario given.
Each option may be used once, more than once or not at all.
11. Severe acute peri-umbilical bruising in the setting of acute pancreatitis.
Cullen's sign
Cullens sign occurs when there has been intraabdominal haemorrage. It is seen
in cases of severe haemorrhagic pancreatitis and is associated with a poor
prognosis. It is also seen in other cases of intraabdominal haemorrhage (such
as ruptured ectopic pregnancy).
12. In acute cholecystitis there is hyperaesthesia beneath the right scapula.
Boas' sign
Boas sign refers to this hyperaesthesia. It occurs because the abdominal wall
innervation of this region is from the spinal roots that lie at this level.
13. In appendicitis palpation of the left iliac fossa causes pain in the right iliac
fossa.
Rovsing's sign
Rovsings sign elicits tenderness because the deep palpation induces shift of
the appendix (which is inflamed) against the peritoneal surface. This has
somatic innervation and will therefore localise the pain. It is less reliable in
pelvic appendicitis and when the appendix is truly retrocaecal
Abdominal signs
A number of eponymous abdominal signs are noted. These include:
 Rovsings sign- appendicitis
 Boas sign -cholecystitis
 Murphys sign- cholecystitis
 Cullens sign- pancreatitis (other intraabdominal haemorrhage)
 Grey-Turners sign- pancreatitis (or other retroperitoneal haemorrhage)
In clinical practice haemorrhagic pancreatitis is thankfully rare. The signs are
important and thus shown below:
Cullen's sign
Image sourced from Wikipedia
Grey Turner's sign
Image sourced from Wikipedia
Theme: Surgical access
A. Gridiron
B. Lanz
C. McEvedy
D. Midline abdominal
E. Rutherford Morrison
F. Battle (abdominal)
G. Lower midline
Please select the most appropriate incision for the procedure required. Each option
may be used once, more than once or not at all.
14. A 78 year old lady is admitted with a tender lump in her right groin. It is
within the femoral triangle and there is concern that there may be small bowel
obstruction developing.
McEvedy
This is one approach to an obstructed femoral hernia. It is possible to
undertake a small bowel resection through this approach. Although recourse to
laparotomy may be needed if access is difficult.
15. A 45 year old woman with end stage renal failure is due to undergo a
cadaveric renal transplant. This will be her first transplant.
Rutherford Morrison
This is the incision of choice for the extraperitoneal approach to the iliac
vessels which will be required for a renal transplant.
16. A slim 20 year old lady is suffering from appendicitis and requires an
appendicectomy.
Lanz
Either a Lanz or Gridiron incision will give access for appendicectomy.
However, in the case described a Lanz incision will give better cosmesis and
can be extended should pelvic surgery be required eg for gynaecological
disease.
Abdominal incisions
Theme in January 2012 exam
Midline incision  Commonest approach to the abdomen
 Structures divided: linea alba, transversalis fascia,
extraperitoneal fat, peritoneum (avoid falciform ligament
above the umbilicus)
 Bladder can be accessed via an extraperitoneal approach
through the space of Retzius
Paramedian
incision
 Parallel to the midline (about 3-4cm)
 Structures divided/retracted: anterior rectus sheath, rectus
(retracted), posterior rectus sheath, transversalis fascia,
extraperitoneal fat, peritoneum
 Incision is closed in layers
Battle  Similar location to paramedian but rectus displaced medially
(and thus denervated)
 Now seldom used
Kocher's Incision under right subcostal margin e.g. Cholecystectomy (open)
Lanz Incision in right iliac fossa e.g. Appendicectomy
Gridiron Oblique incision centered over McBurneys point- usually
appendicectomy (less cosmetically acceptable than Lanz
Gable Rooftop incision
Pfannenstiel's Transverse supra pubic, primarily used to access pelvic organs
McEvedy's Groin incision e.g. Emergency repair strangulated femoral hernia
Rutherford
Morrison
Extraperitoneal approach to left or right lower quadrants. Gives
excellent access to iliac vessels and is the approach of choice for
first time renal transplantation.
Image sourced from Wikipedia
Theme: Hernias
A. Littres hernia
B. Richters hernia
C. Bochdalek hernia
D. Morgagni hernia
E. Spigelian hernia
F. Lumbar hernia
G. Obturator hernia
Please select the type of hernia that most closely matches the description given. Each
option may be used once, more than once or not at all.
17. A 73 year old lady presents with peritonitis and tenderness of the left groin. At
operation she has a left femoral hernia with perforation of the anti mesenteric
border of ileum associated with the hernia.
Richters hernia
When part of the bowel wall is trapped in a hernia such as this it is termed a
Richters hernia and may complicate any hernia although femoral and obturator
hernias are most typically implicated.
18. A 22 year old man is operated on for a left inguinal hernia, at operation the sac
is opened to reveal a large Meckels diverticulum.
Littres hernia
Hernia containing Meckels diverticulum is termed a Littres hernia.
19. A 45 year old man has recurrent colicky abdominal pain. As part of a series of
investigations he undergoes a CT scan and this demonstrates a hernia lateral to
the rectus muscle at the level of the arcuate line.
Spigelian hernia
This is the site for a spigelian hernia.
Theme: Groin masses
A. Femoral hernia
B. Lymphadenitis
C. Inguinal hernia
D. Psoas abscess
E. Saphenous varix
F. Femoral artery aneurysm
G. Metastatic lymphadenopathy
H. Lymphangitis
I. False femoral artery aneurysm
What is the likely diagnosis for groin mass described? Each option may be used once,
more than once, or not at all.
20. A 52 year old obese lady reports a painless grape sized mass in her groin area.
She has no medical conditions apart from some varicose veins. There is a
cough impulse and the mass disappears on lying down.
Saphenous varix
The history of varicose veins should indicate a more likely diagnosis of a
varix. The varix can enlarge during coughing/sneezing. A blue discolouration
may be noted.
21. A 32 year old male is noted to have a tender mass in the right groin area.
There are also red streaks on the thigh, extending from a small abrasion.
You answered Lymphangitis
The correct answer is Lymphadenitis
The red streaks are along the line of the lymphatics, indicating infection of the
lymphatic vessels. Lymphadenitis is infection of the local lymph nodes.
22. A 23 year old male suffering from hepatitis C presents with right groin pain
and swelling. On examination there is a large abscess in the groin. Adjacent to
this is an expansile swelling. There is no cough impulse.
False femoral artery aneurysm
False aneurysms may occur following arterial trauma in IVDU. They may
have associated blood borne virus infections and should undergo duplex
scanning prior to surgery. False aneurysms do not contain all layers of the
arterial wall.
Groin masses clinical
Groin masses are common and include:
 Herniae
 Lipomas
 Lymph nodes
 Undescended testis
 Femoral aneurysm
 Saphena varix (more a swelling than a mass!)
In the history features relating to systemic illness and tempo of onset will often give a
clue as to the most likely underlying diagnosis.
Groin lumps- some key questions
 Is there a cough impulse
 Is it pulsatile AND is it expansile (to distinguish between false and true
aneurysm)
 Are both testes intra scrotal
 Any lesions in the legs such as malignancy or infections (?lymph nodes)
 Examine the ano rectum as anal cancer may metastasise to the groin
 Is the lump soft, small and very superficial (?lipoma)
Scrotal lumps - some key questions
 Is the lump entirely intra scrotal
 Does it transilluminate (?hydrocele)
 Is there a cough impulse (?hernia)
In most cases a diagnosis can be made clinically. Where it is not clear an ultrasound
scan is often the most convenient next investigation.
heme: Right iliac fossa pain
A. Urinary tract infection
B. Appendicitis
C. Mittelschmerz
D. Mesenteric adenitis
E. Crohns disease
F. Ulcerative colitis
G. Meckels diverticulum
Please select the most likely cause for right iliac fossa pain for the scenario given.
Each option may be used once, more than once or not at all.
23. A 17 year old male is admitted with lower abdominal discomfort. He has been
suffering from intermittent right iliac fossa pain for the past few months. His
past medical history includes a negative colonoscopy and gastroscopy for iron
deficiency anaemia. The pain is worse after meals. Inflammatory markers are
normal.
Meckels diverticulum
This scenario should raise suspicion for Meckels as these may contain ectopic
gastric mucosa which may secrete acid with subsequent bleeding and
ulceration.
24. A 14 year old female is admitted with sudden onset right iliac fossa pain. She
is otherwise well and on examination has some right iliac fossa tenderness but
no guarding. She is afebrile. Urinary dipstick is normal. Her previous
menstrual period two weeks ago was normal and pregnancy test is negative.
Mittelschmerz
Typical story and timing for mid cycle pain. Mid cycle pain typically occurs
because a small amount of fluid is released at the time of ovulation. It will
usually resolve over 24-48 hours.
25. A 21 year old male is admitted with a 3 month history of intermittent right
iliac fossa pain. He suffers from episodic diarrhoea and has lost 2 kilos in
weight. On examination he has some right iliac fossa tenderness and is febrile.
Crohns disease
Weight loss and chronic symptoms coupled with change in bowel habit should
raise suspicion for Crohns. The presence of intermittent right iliac fossa pain
is far more typical of terminal ileal Crohns disease. Both UC and Crohns may
be associated with a low grade pyrexia. The main concern here would be
locally perforated Crohns disease with a small associated abscess.
Right iliac fossa pain
Differential diagnosis
Appendicitis  Pain radiating to right iliac fossa
 Anorexia
 Typically short history
 Diarrhoea and profuse vomiting rare
Crohn's disease  Often long history
 Signs of malnutrition
 Change in bowel habit, especially diarrhoea
Mesenteric adenitis  Mainly affects children
 Causes include Adenoviruses, Epstein Barr Virus,
beta-haemolytic Streptococcus, Staphylococcus
spp., Escherichia coli, Streptococcus viridans and
Yersinia spp.
 Patients have a higher temperature than those with
appendicitis
 If laparotomy is performed, enlarged mesenteric
lymph nodes will be present
Diverticulitis  Both left and right sided disease may present with
right iliac fossa pain
 Clinical history may be similar, although some
change in bowel habit is usual
 When suspected a CT scan may help in refining the
diagnosis
Meckel's diverticulitis  A Meckel's diverticulum is a congenital abnormality
that is present in about 2% of the population
 Typically 2 feet proximal to the ileocaecal valve
 May be lined by ectopic gastric mucosal tissue and
produce bleeding
Perforated peptic ulcer  This usually produces upper quadrant pain but pain
may be lower
 Perforations typically have a sharp sudden onset of
pain in the history
Incarcerated right
inguinal or femoral
hernia
 Usually only right iliac fossa pain if right sided or
bowel obstruction.
Bowel perforation
secondary to caecal or
colon carcinoma
 Seldom localised to right iliac fossa, although
complete large bowel obstruction with caecal
distension may cause pain prior to perforation.
Gynaecological causes  Pelvic inflammatory disease/salpingitis/pelvic
abscess/Ectopic pregnancy/Ovarian
torsion/Threatened or complete
abortion/Mittelschmerz
Urological causes  Ureteric colic/UTI/Testicular torsion
Other causes  TB/Typhoid/Herpes Zoster/AAA/Situs inversus
A 78 year old lady presents with colicky abdominal pain and a tender mass in her
groin. On examination there is a small firm mass below and lateral to the pubic
tubercle. Which of the following is the most likely underlying diagnosis?
A. Incarcerated inguinal hernia
B. Thrombophlebitis of a saphena varix
C. Incarcerated femoral hernia
D. Incarcerated obturator hernia
E. Deep vein thrombosis
Femoral hernia = High risk of
strangulation (repair urgently)
Femoral herniae account for <10% of all groin hernias. In the scenario the
combination of symptoms of intestinal compromise with a mass in the region of the
femoral canal points to femoral hernia as the most likely cause.
Femoral canal
The femoral canal lies at the medial aspect of the femoral sheath. The femoral sheath
is a fascial tunnel containing both the femoral artery laterally and femoral vein
medially. The canal lies medial to the vein.
Borders of the femoral canal
Laterally Femoral vein
Medially Lacunar ligament
Anteriorly Inguinal ligament
Posteriorly Pectineal ligament
Image showing dissection of femoral canal
Image sourced from Wikipedia
Contents
 Lymphatic vessels
 Cloquet's lymph node
Physiological significance
Allows the femoral vein to expand to allow for increased venous return to the lower
limbs.
Pathological significance
As a potential space, it is the site of femoral hernias. The relatively tight neck places
these at high risk of strangulation.
hich of the following is not a typical feature of acute appendicitis?
A. Neutrophilia
B. Profuse vomiting
C. Anorexia
D. Low grade pyrexia
E. Small amounts of protein on urine analysis
Profuse vomiting and diarrhoea are
rare in early appendicitis
Whilst patients may vomit once or twice, profuse vomiting is unusual, and would fit
more with gastroenteritis or an ileus. A trace of protein is not an uncommon
occurrence in acute appendicitis. A free lying pelvic appendix may result in localised
bladder irritation, with inflammation occurring as a secondary phenomena. This latter
feature may result in patients being incorrectly diagnosed as having a urinary tract
infection. A urine dipstick test is useful in differentiating between the two conditions.
Appendicitis
History
 Peri umbilical abdominal pain (visceral stretching of appendix lumen and
appendix is mid gut structure) radiating to the right iliac fossa due to localised
parietal peritoneal inflammation.
 Vomit once or twice but marked and persistent vomiting is unusual.
 Diarrhoea is rare. However, pelvic appendicitis may cause localised rectal
irritation of some loose stools. A pelvic abscess may also cause diarrhoea.
 Mild pyrexia is common - temperature is usually 37.5 -38o
C. Higher
temperatures are more typical of conditions like mesenteric adenitis.
 Anorexia is very common. It is very unusual for patients with appendicitis to
be hungry.
Examination
 Generalised peritonitis if perforation has occurred or localised peritonism.
 Retrocaecal appendicitis may have relatively few signs.
 Digital rectal examination may reveal boggy sensation if pelvic abscess is
present, or even tenderness with a pelvic appendix.
Diagnosis
 Typically raised inflammatory markers coupled with compatible history and
examination findings should be enough to justify appendicectomy.
 Urine analysis may show mild leucocytosis but no nitrites.
 Ultrasound is useful if females where pelvic organ pathology is suspected.
Although it is not always possible to visualise the appendix on ultrasound the
presence of free fluid (always pathological in males) should raise suspicion.
Ultrasound examination may show evidence of lumenal obstruction and thickening of
the appendiceal wall as shown below
Image sourced from Wikipedia
Treatment
 Appendicectomy which can be performed via either an open or laparoscopic
approach.
 Administration of metronidazole reduces wound infection rates.
 Patients with perforated appendicitis require copious abdominal lavage.
 Patients without peritonitis who have an appendix mass should receive broad
spectrum antibiotics and consideration given to performing an interval
appendicectomy.
 Be wary in the older patients who may have either an underlying caecal
malignancy or perforated sigmoid diverticular disease.
Laparoscopic appendicectomy is becoming increasing popular as demonstrated
below
Image sourced from Wikipedia
An 28 year old man presents with a direct inguinal hernia. A decision is made to
perform an open inguinal hernia repair. Which of the following is the best option for
abdominal wall reconstruction in this case?
A. Suture plication of the transversalis fascia using PDS
only
B. Suture plication of the hernial defect with nylon and
placement of prolene mesh anterior to external
oblique
C. Suture plication of the hernia defect using nylon and
re-enforcing with a sutured repair of the abdominal
wall
D. Sutured repair of the hernial defect with prolene and
placement of prolene mesh over the cord structures
in the inguinal canal
E. Sutured repair of the hernial defect using nylon and
placement of a prolene mesh posterior to the cord
structures
Laparoscopic repair- bilateral
and recurrent cases
During an inguinal hernia repair in males the cord structures will always lie anterior to
the mesh. In the conventional open repairs the cord structures are mobilised and the
mesh placed behind them, with a slit made to allow passage of the cord structures
through the deep inguinal ring. Placement of the mesh over the cord structures results
in chronic pain and usually a higher risk of recurrence.
Laparoscopic inguinal hernia repair is the procedure of choice for bilateral inguinal
hernias.
Types of surgery include:
 Onlay mesh repair (Litchenstein style)
 Inguinal herniorrhaphy
 Shouldice repair
 Darn repair
 Laparoscopic mesh repair
Open mesh repair and laparoscopic repair are the two main procedures in mainstream
use. The Shouldice repair is a useful procedure in cases where a mesh repair would be
associated with increased risk of infection, e.g. repair of case with strangulated bowel,
as it avoids the use of mesh. It is, however, far more technically challenging to
perform.
Inguinal hernia surgery
Inguinal hernias occur when the abdominal viscera protrude through the anterior
abdominal wall into the inguinal canal. They may be classified as being either direct
or indirect. The distinction between these two rests on their relation to Hesselbach's
triangle.
Boundaries of Hesselbach's Triangle
 Medial: Rectus abdominis
 Lateral: Inferior epigastric vessels
 Inferior: Inguinal ligament
Image sourced from Wikipedia
Hernias occurring within the triangle tend to be direct and those outside - indirect.
Diagnosis
Most cases are diagnosed clinically, a reducible swelling may be located at the level
of the inguinal canal. Large hernia may extend down into the male scrotum, these will
not trans-illuminate and it is not possible to "get above" the swelling.
Cases that are unclear on examination, but suspected from the history, may be further
investigated using ultrasound or by performing a herniogram.
Treatment
Hernias associated with few symptoms may be managed conservatively. Symptomatic
hernias or those which are at risk of developing complications are usually treated
surgically.
First time hernias may be treated by performing an open inguinal hernia repair; the
inguinal canal is opened, the hernia reduced and the defect repaired. A prosthetic
mesh may be placed posterior to the cord structures to re-inforce the repair and reduce
the risk of recurrence.
Recurrent hernias and those which are bilateral are generally managed with a
laparoscopic approach. This may be via an intra or extra peritoneal route. As in open
surgery a mesh is deployed. However, it will typically lie posterior to the deep ring.
Inguinal hernia in children
Inguinal hernias in children are almost always of an indirect type and therefore are
usually dealt with by herniotomy, rather than herniorraphy. Neonatal hernias
especially in those children born prematurely are at highest risk of strangulation and
should be repaired urgently. Other hernias may be repaired on an elective basis.
References
The UK Based National Institute of Clinical Excellence has published guidelines
relating to the choice between open and laparoscopic inguinal hernia repair. Which
users may find interesting:
Theme: Abdominal closure methods
A. Looped 1/0 PDS (polydiaxone)
B. Looped 1/0 silk
C. 1/0 Vicryl (polyglactin)
D. 1/0 Vicryl rapide
E. 2/0 Prolene (Polypropylene)
F. Re-inforced 1/0 Nylon
G. Re-inforced 1/0 Silk
H. Application of VAC system without separation film
I. Application of VAC System with separation film
J. Application of a 'Bogota Bag'
Please select the most appropriate wound closure method (for the deep layer) for the
abdominal surgery described.
29. A 59 year old man with morbid obesity undergoes a laparotomy and Hartmans
procedure for perforated sigmoid diverticular disease. At the conclusion of the
procedure the abdomen cannot be primarily closed. The Vac system is not
available for use.
Application of a 'Bogota Bag'
Application of a Bogota bag is safest as attempted closure will almost
certainly fail. Repeat look at 48 hours to determine the best definitive option is
needed.
30. A 73 year old lady undergoes a low anterior resection for carcinoma of the
rectum.
Looped 1/0 PDS (polydiaxone)
Mass closure obeying Jenkins rule is required and this states that the suture
must be 4 times the length of the wound with tissue bites 1cm deep and 1 cm
apart.
31. A 67 year old is returned to theatre after developing a burst abdomen on the
ward. She has originally undergone a right hemicolectomy and the SHO who
closed the wound had failed to tie the midline suture correctly. The wound
edges appear healthy.
You answered Re-inforced 1/0 Silk
The correct answer is Re-inforced 1/0 Nylon
Attempt at re-closing the wound is reasonable in which case 1/0 nylon
(reinforced with drainage tubing) is often used.
Abdominal wound dehiscence
 This is a significant problem facing all surgeons who undertake abdominal
surgery on a regular basis. Traditionally it is said to occur when all layers of
an abdominal mass closure fail and the viscera protrude externally (associated
with 30% mortality).
 It can be subdivided into superficial, in which the skin wound alone fails and
complete, implying failure of all layers.
Factors which increase the risk are:
* Malnutrition
* Vitamin deficiencies
* Jaundice
* Steroid use
* Major wound contamination (e.g. faecal peritonitis)
* Poor surgical technique (Mass closure technique is the preferred method-Jenkins
Rule)
When sudden full dehiscence occurs the management is as follows:
* Analgesia
* Intravenous fluids
* Intravenous broad spectrum antibiotics
* Coverage of the wound with saline impregnated gauze (on the ward)
* Arrangements made for a return to theatre
Surgical strategy
 Correct the underlying cause (eg TPN or NG feed if malnourished)
 Determine the most appropriate strategy for managing the wound
Options
Resuturing of
the wound
This may be an option if the wound edges are healthy and there is
enough tissue for sufficient coverage. Deep tension sutures are
traditionally used for this purpose.
Application of a
wound manager
This is a clear dressing with removable front. Particularly suitable
when some granulation tissue is present over the viscera or where
there is a high output bowel fistula present in the dehisced wound.
Application of a
'Bogota bag'
This is a clear plastic bag that is cut and sutured to the wound edges
and is only a temporary measure to be adopted when the wound
cannot be closed and will necessitate a return to theatre for
definitive management.
Application of a
VAC dressing
system
These can be safely used BUT ONLY if the correct layer is
interposed between the suction device and the bowel. Failure to
adhere to this absolute rule will almost invariably result in the
development of multiple bowel fistulae and create an extremely
difficult management problem.
Theme: Hernias
A. Umbilical hernia
B. Para umbilical hernia
C. Morgagni hernia
D. Littres hernia
E. Bochdalek hernia
F. Richters hernia
G. Obturator hernia
Please select the hernia that most closely matches the description given. Each option
may be used once, more than once or not at all.
32. A 1 day old infant is born with severe respiratory compromise. On
examination he has a scaphoid abdomen and an absent apex beat.
Bochdalek hernia
Theme from 2011 exam
The large hernia may displace the heart although true dextrocardia is not
present. The associated pulmonary hypoplasia will compromise lung
development.
33. A 2 month old infant is troubled by recurrent colicky abdominal pain and
intermittent intestinal obstruction. On imaging the transverse colon is
herniated into the thoracic cavity, through a mid line defect.
Morgagni hernia
Morgagni hernia may contain the transverse colon. Unless there is substantial
herniation pulmonary hypoplasia is uncommon.
34. A 78 year old lady is admitted with small bowel obstruction, on examination
she has a distended abdomen and the leg is held semi flexed. She has some
groin pain radiating to the ipsilateral knee.
Obturator hernia
The groin swelling in obturator hernia is subtle and hard to elicit clinically.
Hernia
Hernias occur when a viscus or part of it protrudes from within its normal anatomical
cavity. Specific hernias are covered under their designated titles the remainder are
addressed here.
Spigelian hernia
 Interparietal hernia occurring at the level of the arcuate line.
 Rare.
 May lie beneath internal oblique muscle. Usually between internal and
external oblique.
 Equal sex distribution.
 Position is lateral to rectus abdominis.
 Both open and laparoscopic repair are possible, the former in cases of
strangulation.
Lumbar hernia
 The lumbar triangle (through which these may occur) is bounded by:
Crest of ilium (inferiorly)
External oblique (laterally)
Latissimus dorsi (medially)
 Primary lumbar herniae are rare and most are incisional hernias following
renal surgery.
 Direct anatomical repair with or without mesh re-enforcement is the procedure
of choice.
Obturator hernia
 Herniation through the obturator canal.
 Commoner in females.
 Usually lies behind pectineus muscle.
 Elective diagnosis is unusual most will present acutely with obstruction.
 When presenting acutely most cases with require laparotomy or laparoscopy
(and small bowel resection if indicated).
Richters hernia
 Condition in which part of the wall of the small bowel (usually the anti
mesenteric border) is strangulated within a hernia (of any type).
 They do not present with typical features of intestinal obstruction as luminal
patency is preserved.
 Where vomiting is prominent it usually occurs as a result of paralytic ileus
from peritonitis (as these hernias may perforate).
Incisional hernia
 Occur through sites of surgical access into the abdominal cavity.
 Most common following surgical wound infection.
 To minimise following midline laparotomy Jenkins Rule should be followed
and this necessitates a suture length 4x length of incision with bites taken at
1cm intervals, 1 cm from the wound edge.
 Repair may be performed either at open surgery or laparoscopically and a
wide variety of techniques are described.
Bochdalek hernia
 Typically congenital diaphragmatic hernia.
 85% cases are located in the left hemi diaphragm.
 Associated with lung hypoplasia on the affected side.
 More common in males.
 Associated with other birth defects.
 May contain stomach.
 May be treated by direct anatomical apposition or placement of mesh. In
infants that have severe respiratory compromise mechanical ventilation may
be needed and mortality rate is high.
Morgagni Hernia
 Rare type of diaphragmatic hernia (approx 2% cases).
 Herniation through foramen of Morgagni.
 Usually located on the right and tend to be less symptomatic.
 More advanced cases may contain transverse colon.
 As defects are small pulmonary hypoplasia is less common.
 Direct anatomical repair is performed.
Umbilical hernia
 Hernia through weak umbilicus.
 Usually presents in childhood.
 Often symptomatic.
 Equal sex incidence.
 95% will resolve by the age of 2 years. Thereafter surgical repair is warranted.
Paraumbilical hernia
 Usually a condition of adulthood.
 Defect is in the linea alba.
 More common in females.
 Multiparity and obesity are risk factors.
 Traditionally repaired using Mayos technique - overlapping repair, mesh may
be used though not if small bowel resection is required owing to acute
strangulation.
Littres hernia
 Hernia containing Meckels diverticulum.
 Resection of the diverticulum is usually required and this will preclude a mesh
repair.
Theme: Right iliac fossa pain
A. Open Appendicectomy
B. Laparoscopic appendicectomy
C. Laparotomy
D. CT Scan
E. Colonoscopy
F. Ultrasound scan abdomen/pelvis
G. Active observation
For each scenario please select the most appropriate management option from the list.
Each option may be used once, more than once or not at all.
35. A 21 year old women is admitted with a 48 hour history of worsening right
iliac fossa pain. She has been nauseated and vomited twice. On examination
she is markedly tender in the right iliac fossa with localised guarding. Vaginal
examination is unremarkable. Urine dipstick is negative. Blood tests show a
WCC of 13.5 and CRP 70.
You answered Open Appendicectomy
The correct answer is Laparoscopic appendicectomy
She is likely to have appendicitis. In women of this age there is always
diagnostic uncertainty. With a normal vaginal exam laparoscopy would be
preferred over USS.
36. An 8 year old boy presents with a 4 hour history of right iliac fossa pain with
nausea and vomiting. He has been back at school for two days after being kept
home with a flu like illness. On examination he is tender in the right iliac
fossa, although his abdomen is soft. Temperature is 38.3o
c. Blood tests show a
CRP of 40 and a WCC of 8.1.
You answered CT Scan
The correct answer is Active observation
This is mesenteric adenitis. Note history of flu like illness and temp > 38o
c.
The decision as to how to manage this situation is based on the abdominal
findings. Patients with localising signs such as guarding or peritonism should
undergo surgery.
37. A 21 year old women presents with right iliac fossa pain. She reports some
bloodstained vaginal discharge. She has a HR of 65 bpm.
Ultrasound scan abdomen/pelvis
This patient is suspected of having an ectopic pregnancy. She needs an urgent
β HCG and USS of the pelvis. If she were haemodynamically unstable then
laparotomy would be indicated.
Right iliac fossa pain
Differential diagnosis
Appendicitis  Pain radiating to right iliac fossa
 Anorexia
 Typically short history
 Diarrhoea and profuse vomiting rare
Crohn's disease  Often long history
 Signs of malnutrition
 Change in bowel habit, especially diarrhoea
Mesenteric adenitis  Mainly affects children
 Causes include Adenoviruses, Epstein Barr Virus,
beta-haemolytic Streptococcus, Staphylococcus
spp., Escherichia coli, Streptococcus viridans and
Yersinia spp.
 Patients have a higher temperature than those with
appendicitis
 If laparotomy is performed, enlarged mesenteric
lymph nodes will be present
Diverticulitis  Both left and right sided disease may present with
right iliac fossa pain
 Clinical history may be similar, although some
change in bowel habit is usual
 When suspected a CT scan may help in refining the
diagnosis
Meckel's diverticulitis  A Meckel's diverticulum is a congenital abnormality
that is present in about 2% of the population
 Typically 2 feet proximal to the ileocaecal valve
 May be lined by ectopic gastric mucosal tissue and
produce bleeding
Perforated peptic ulcer  This usually produces upper quadrant pain but pain
may be lower
 Perforations typically have a sharp sudden onset of
pain in the history
Incarcerated right
inguinal or femoral
hernia
 Usually only right iliac fossa pain if right sided or
bowel obstruction.
Bowel perforation
secondary to caecal or
colon carcinoma
 Seldom localised to right iliac fossa, although
complete large bowel obstruction with caecal
distension may cause pain prior to perforation.
Gynaecological causes  Pelvic inflammatory disease/salpingitis/pelvic
abscess/Ectopic pregnancy/Ovarian
torsion/Threatened or complete
abortion/Mittelschmerz
Urological causes  Ureteric colic/UTI/Testicular torsion
Other causes  TB/Typhoid/Herpes Zoster/AAA/Situs inversus
Which of the following is not a typical feature of irritable bowel syndrome?
A. A change in the consistency of stools
B. Abdominal pain relieved with defecation
C. A change in frequency of defecation
D. Abdominal bloating
E. Pain at a single fixed site
The pain or discomfort of IBS is typically migratory and variable in intensity. Pain at
a fixed site is suggestive of malignancy.
Abdominal bloating is an extremely common feature.
Irritable bowel syndrome
The diagnosis of irritable bowel syndrome is made according to the ROME III
diagnostic criteria which state:
Recurrent abdominal pain or discomfort at 3 days per month for the past 3
months associated with two or more of the following:
 Improvement with defecation.
 Onset associated with a change in the frequency of stool.
 Onset associated with a change in the form of the stool.
Features such as lethargy, nausea, backache and bladder symptoms may also support
the diagnosis
Red flag features should be inquired about:
 Rectal bleeding
 Unexplained/unintentional weight loss
 Family history of bowel or ovarian cancer
 Onset after 60 years of age
Suggested investigations are:
 Full blood count
 ESR/CRP
 Coeliac disease screen (tissue transglutaminase antibodies)
 Colonoscopy (if worrying symptoms, positive family history)
 Thyroid function tests
 Glucose (ensure not diabetic)
The NICE criteria state that blood tests alone will suffice in people fulfilling the
diagnostic criteria. We would point out that luminal colonic studies should be
considered early in patients with altered bowel habit referred to hospital and a
diagnosis of IBS should still be largely one of exclusion.
Treatment
 Usually reduce fibre intake.
 Tailored prescriptions of laxatives or loperamide according to clinical picture.
 Dietary modification (caffeine avoidance, less carbonated drinks).
 Consider low dose tricyclic antidepressants if pain is a dominant symptom.
 Biofeedback may help.
Theme: Causes of abdominal pain
A. Acute on chronic mesenteric ischaemia
B. Ruptured aortic aneurysm
C. Acute Pancreatitis
D. Acute mesenteric embolus
E. Acute appendicitis
F. Chronic pancreatitis
G. Mesenteric vein thrombosis
Please select the most likely underlying diagnosis from the list above. Each option
may be used once, more than once or not at all.
39. A 41 year old man is admitted with peritonitis secondary to a perforated
appendix. He is treated with a laparoscopic appendicectomy but has a stormy
post operative course. He in now developing increasing abdominal pain and
has been vomiting. A laparotomy is performed and at operation a large
amount of small bowel shows evidence of patchy areas of infarction.
Mesenteric vein thrombosis
Mesenteric vein thrombosis may complicate severe intra abdominal sepsis and
when it progresses may impair bowel perfusion. The serosa is quite resistant
to ischaemia so in this case the appearances are usually patchy.
40. A 68 year old man is admitted with abdominal pain and vomiting of 48 hours
duration, the pain radiates to his back and he has required a considerable
amount of volume replacement. Amylase is 741.
Acute Pancreatitis
Although back pain and abdominal pain coupled with haemodynamic
compromise may suggest ruptured AAA the 48 hour history and amylase >3
times normal go against this diagnosis.
41. A 79 year old lady develops sudden onset of abdominal pain and collapses,
she has passed a large amount of diarrhoea. In casualty her pH is 7.35 and
WCC is 18.
You answered Acute on chronic mesenteric ischaemia
The correct answer is Acute mesenteric embolus
Although mesenteric infarct may raise the lactate the pH may be raised often
secondary to vomiting.
Mesenteric vessel disease
Mesenteric ischaemia accounts for 1 in 1000 acute surgical admissions. It is primarily
caused by arterial embolism resulting in infarction of the colon. It is more likely to
occur in areas such as the splenic flexure that are located at the borders of the territory
supplied by the superior and inferior mesenteric arteries.
Types
Acute mesenteric
embolus
(commonest 50%)
 Sudden onset abdominal pain followed by profuse
diarrhoea.
 May be associated with vomiting.
 Rapid clinical deterioration.
 Serological tests: WCC, lactate, amylase may all be
abnormal particularly in established disease. These can be
normal in the early phases.
Acute on chronic
mesenteric
ischaemia
 Usually longer prodromal history.
 Post prandial abdominal discomfort and weight loss are
dominant features. Patients will usually present with an
acute on chronic event, but otherwise will tend not to
present until mesenteric flow is reduced by greater than
80%.
 When acute thrombosis occurs presentation may be as
above. In the chronic setting the symptoms will often be
those of ischaemic colitis (mucosa is the most sensitive
area to this insult).
Mesenteric vein
thrombosis
 Usually a history over weeks.
 Overt abdominal signs and symptoms will not occur until
venous thrombosis has reached a stage to compromise
arterial inflow.
 Thrombophilia accounts for 60% of cases.
Low flow
mesenteric
infarction
 This occurs in patients with multiple co morbidities in
whom mesenteric perfusion is significantly compromised
by overuse of inotropes or background cardiovascular
compromise.
 The end result is that the bowel is not adequately perfused
and infarcts occur from the mucosa outwards.
Diagnosis
 Serological tests: WCC, lactate, CRP, amylase (can be normal in early
disease).
 Cornerstone for diagnosis of arterial AND venous mesenteric disease is CT
angiography scanning in the arterial phase with thin slices (<5mm). Venous
phase contrast is not helpful.
 SMA duplex USS is useful in the evaluation of proximal SMA disease in
patients with chronic mesenteric ischaemia.
 MRI is of limited use due to gut peristalsis and movement artefact.
Management
 Overt signs of peritonism: Laparotomy
 Mesenteric vein thrombosis: If no peritonism: Medical management with IV
heparin
 At operation limited resection of frankly necrotic bowel with view to relook
laparotomy at 24-48h. In the interim urgent bowel revascularisation via
endovascular (preferred) or surgery.
Prognosis
Overall poor. Best outlook is from an acute ischaemia from an embolic event where
surgery occurs within 12h. Survival may be 50%. This falls to 30% with treatment
delay. The other conditions carry worse survival figures.
Theme: Groin masses
A. Femoral aneurysm
B. Lymphadenitis
C. Saphena varix
D. Femoral hernia
E. Indirect inguinal hernia
F. Direct inguinal hernia
G. Psoas abscess
What is the likely diagnosis for the groin mass described? Each option may be used
once, more than once or not at all.
42. A 3 year old boy is referred to the clinic with a scrotal swelling. On
examination the mass does not transilluminate and it is impossible to palpate
normal cord above it.
Indirect inguinal hernia
This is likely to be an indirect hernia. In children these arise from persistent
processus vaginalis and require herniotomy.
43. A 52 year old obese lady reports a painless mass in the groin area. A mass is
noted on coughing. It is below and lateral to the pubic tubercle.
Femoral hernia
A mass below and lateral to the pubic tubercle is indicative of a femoral
hernia.
44. A 21 year old man is admitted with a tender mass in the right groin, fevers and
sweats. He is on multiple medical therapy for HIV infection. On examination
he has a swelling in his right groin, hip extension exacerbates the pain.
Psoas abscess
Psoas abscesses may be either primary or secondary. Primary cases often
occur in the immunosuppressed and may occur as a result of haematogenous
spread. Secondary cases may complicated intra abdominal diseases such as
Crohns. Patients usually present with low back pain and if the abscess is
extensive a mass that may be localised to the inguinal region or femoral
triangle . Smaller collections may be percutaneously drained. If the collection
is larger, or the percutaneous route fails, then surgery (via a retroperitoneal
approach) should be performed.
Groin masses clinical
Groin masses are common and include:
 Herniae
 Lipomas
 Lymph nodes
 Undescended testis
 Femoral aneurysm
 Saphena varix (more a swelling than a mass!)
In the history features relating to systemic illness and tempo of onset will often give a
clue as to the most likely underlying diagnosis.
Groin lumps- some key questions
 Is there a cough impulse
 Is it pulsatile AND is it expansile (to distinguish between false and true
aneurysm)
 Are both testes intra scrotal
 Any lesions in the legs such as malignancy or infections (?lymph nodes)
 Examine the ano rectum as anal cancer may metastasise to the groin
 Is the lump soft, small and very superficial (?lipoma)
Scrotal lumps - some key questions
 Is the lump entirely intra scrotal
 Does it transilluminate (?hydrocele)
 Is there a cough impulse (?hernia)
In most cases a diagnosis can be made clinically. Where it is not clear an ultrasound
scan is often the most convenient next investigation.
A 56 year old lady is admitted with colicky abdominal pain. A plain x-ray is
performed. Which of the following should not show fluid levels on a plain abdominal
film?
A. Stomach
B. Jejunum
C. Ileum
D. Caecum
E. Descending colon
Fluid levels in the distal colon are nearly always pathological. In general contents of
the left colon transit quickly and are seldom held in situ for long periods, the content
is also more solid.
Abdominal radiology
Plain abdominal x-rays are often used as a first line investigation in patients with
acute abdominal pain. A plain abdominal film may demonstrate free air, evidence of
bowel obstruction and possibly other causes of pain (e.g. renal or gallbladder stones).
Investigation of potential visceral perforation is usually best performed by obtaining
an erect chest x-ray, as this is a more sensitive investigation for suspected visceral
perforation.
Features which are usually abnormal
 Large amounts of free air (colonic perforation), smaller volumes seen with
more proximal perforations.
 A positive Riglers sign (gas on both sides of the bowel wall).
 Caecal diameter of >8cm
 Fluid levels in the colon
 Ground glass appearance to film (usually due to large amounts of free fluid).
 Sentinel loop in patients with inflammation of other organs (e.g. pancreatitis).
Features which should be expected/ or occur without pathology
 In Chialditis syndrome, a loop of bowel may be interposed between the liver
and diaphragm, giving the mistaken impression that free air is present.
 Following ERCP (and sphincterotomy) air may be identified in the biliary tree.
 Free intra abdominal air following laparoscopy / laparotomy, although usually
dissipates after 48-72 hours.
A 56 year old lady presents with a large bowel obstruction and abdominal distension.
Which of the following confirmatory tests should be performed prior to surgery
A. Abdominal ultrasound scan
B. Barium enema
C. Rectal MRI Scan
D. Endoanal ultrasound scan
E. Gastrograffin enema
Patients with clinical evidence of large bowel obstruction, should have the presence or
absence of an obstructing lesion confirmed prior to surgery. This is because colonic
pseudo-obstruction may produce a similar radiological picture. A gastrograffin enema
is the traditional test, as barium is too toxic if it spills into the abdominal cavity. An
MRI scan will not provide the relevant information, unless the lesion is rectal and
below the peritoneal reflection.
Abdominal radiology
Plain abdominal x-rays are often used as a first line investigation in patients with
acute abdominal pain. A plain abdominal film may demonstrate free air, evidence of
bowel obstruction and possibly other causes of pain (e.g. renal or gallbladder stones).
Investigation of potential visceral perforation is usually best performed by obtaining
an erect chest x-ray, as this is a more sensitive investigation for suspected visceral
perforation.
Features which are usually abnormal
 Large amounts of free air (colonic perforation), smaller volumes seen with
more proximal perforations.
 A positive Riglers sign (gas on both sides of the bowel wall).
 Caecal diameter of >8cm
 Fluid levels in the colon
 Ground glass appearance to film (usually due to large amounts of free fluid).
 Sentinel loop in patients with inflammation of other organs (e.g. pancreatitis).
Features which should be expected/ or occur without pathology
 In Chialditis syndrome, a loop of bowel may be interposed between the liver
and diaphragm, giving the mistaken impression that free air is present.
 Following ERCP (and sphincterotomy) air may be identified in the biliary tree.
 Free intra abdominal air following laparoscopy / laparotomy, although usually
dissipates after 48-72 hours.
Theme: Management of splenic trauma
A. Splenectomy
B. Angiography
C. CT Scan
D. Admit for bed rest and observation
E. Ultrasound scan
F. Splenic conservation
G. MRI of the abdomen
Please select the most appropriate intervention for the scenario given. Each option
may be used once, more than once or not at all.
1. A 7 year old boy falls off a wall the distance is 7 feet. He lands on his left side
and there is left flank bruising. There is no haematuria. He is otherwise stable
and haemoglobin is within normal limits.
Ultrasound scan
This will demonstrate any overt splenic injury. A CT scan carries a significant
dose of radiation. In the absence of haemodynamic instability or other major
associated injuries the use of USS to exclude intraabdominal free fluid (blood)
would seem safe when coupled with active observation. An USS will also show
splenic haematomas.
2. A 42 year old motorcyclist is involved in a road traffic accident. A FAST scan
in the emergency department shows free intrabdominal fluid and a laparotomy
is performed. At operation there is evidence of small liver laceration that has
stopped bleeding and a tear to the inferior pole of the spleen.
Splenic conservation
As minimum damage, attempt conservation.
3. An 18 year old man is involved in a road traffic accident. A CT scan shows
disruption of the splenic hilum and a moderate sized perisplenic haematoma.
Splenectomy
Hilar injuries usually mandate splenectomy. The main risk with conservative
management here is that he will rebleed and with hilar injuries this can be
dramatic.
Splenic trauma
 The spleen is one of the more commonly injured intra abdominal organs
 In most cases the spleen can be conserved. The management is dictated by the
associated injuries, haemodynamic status and extent of direct splenic injury.
Management of splenic trauma
Conservative Small subcapsular haematoma
Minimal intra abdominal blood
No hilar disruption
Laparotomy with conservation Increased amounts of intraabdominal blood
Moderate haemodynamic compromise
Tears or lacerations affecting <50%
Resection Hilar injuries
Major haemorrhage
Major associated injuries
Splenectomy
Technique
Trauma
 GA
 Long midline incision
 If time permits insert a self retaining retractor (e.g. Balfour/ omnitract)
 Large amount of free blood is usually present. Pack all 4 quadrants of the
abdomen. Allow the anaesthetist to 'catch up'
 Remove the packs and assess the viability of the spleen. Hilar injuries and
extensive parenchymal lacerations will usually require splenectomy.
 Divide the short gastric vessels and ligate them.
 Clamp the splenic artery and vein. Two clamps on the patient side are better
and allow for double ligation and serve as a safety net if your assistant does
not release the clamp smoothly.
 Be careful not to damage the tail of the pancreas, if you do then this will need
to be formally removed and the pancreatic duct closed.
 Wash out the abdomen and place a tube drain to the splenic bed.
 Some surgeons implant a portion of spleen into the omentum, whether you
decide to do this is a matter of personal choice.
 Post operatively the patient will require prophylactic penicillin V and
pneumococcal vaccine.
Elective
Elective splenectomy is a very different operation from that performed in the
emergency setting. The spleen is often large (sometimes massive). Most cases can be
performed laparoscopically. The spleen will often be macerated inside a specimen bag
to facilitate extraction.
Complications
 Haemorrhage (may be early and either from short gastrics or splenic hilar
vessels
 Pancreatic fistula (from iatrogenic damage to pancreatic tail)
 Thrombocytosis: prophylactic aspirin
 Encapsulated bacteria infection e.g. Strep. pneumoniae, Haemophilus
influenzae and Neisseria meningitidis
Which of the following does not increase the risk of abdominal wound dehiscence
following laparotomy?
A. Jaundice
B. Abdominal compartment syndrome
C. Poorly controlled diabetes mellitus
D. Administration of intravenous steroids
E. Use of Ketamine as an anaesthetic agent
Ketamine does not affect healing. All the other situations in the list carry a strong
association with poor healing and risk of dehisence.
Abdominal wound dehiscence
 This is a significant problem facing all surgeons who undertake abdominal
surgery on a regular basis. Traditionally it is said to occur when all layers of
an abdominal mass closure fail and the viscera protrude externally (associated
with 30% mortality).
 It can be subdivided into superficial, in which the skin wound alone fails and
complete, implying failure of all layers.
Factors which increase the risk are:
* Malnutrition
* Vitamin deficiencies
* Jaundice
* Steroid use
* Major wound contamination (e.g. faecal peritonitis)
* Poor surgical technique (Mass closure technique is the preferred method-Jenkins
Rule)
When sudden full dehiscence occurs the management is as follows:
* Analgesia
* Intravenous fluids
* Intravenous broad spectrum antibiotics
* Coverage of the wound with saline impregnated gauze (on the ward)
* Arrangements made for a return to theatre
Surgical strategy
 Correct the underlying cause (eg TPN or NG feed if malnourished)
 Determine the most appropriate strategy for managing the wound
Options
Resuturing of
the wound
This may be an option if the wound edges are healthy and there is
enough tissue for sufficient coverage. Deep tension sutures are
traditionally used for this purpose.
Application of a
wound manager
This is a clear dressing with removable front. Particularly suitable
when some granulation tissue is present over the viscera or where
there is a high output bowel fistula present in the dehisced wound.
Application of a This is a clear plastic bag that is cut and sutured to the wound edges
'Bogota bag' and is only a temporary measure to be adopted when the wound
cannot be closed and will necessitate a return to theatre for
definitive management.
Application of a
VAC dressing
system
These can be safely used BUT ONLY if the correct layer is
interposed between the suction device and the bowel. Failure to
adhere to this absolute rule will almost invariably result in the
development of multiple bowel fistulae and create an extremely
difficult management problem.
Theme: Causes of diarrhoea
A. Campylobacter jejuni infection
B. Salmonella gastroenteritis infection
C. Crohns disease
D. Ulcerative colitis
E. Irritable bowel syndrome
F. Ischaemic colitis
G. Laxative abuse
H. Clostridium difficile infection
Please select the most likely cause of diarrhoea for each scenario given. Each option
may be used once, more than once or not at all.
5. A 23 year old lady has suffered from diarrhoea for 8 months, she has also lost 2
Kg in weight. At colonoscopy appearances of melanosis coli are identified and
confirmed on biopsy
You answered Ulcerative colitis
The correct answer is Laxative abuse
This may occur as a result of laxative abuse and consists of lipofuschin laden
marcophages that appear brown.
6. A 68 year old lady has recently undergone an abdominal aortic aneurysm
repair. The operation was performed electively and was uncomplicated. Since
surgery she has had repeated episodes of diarrhoea.
Ischaemic colitis
The IMA is commonly ligated during and AAA repair and this may then render
the left colon relatively ischaemic, thereby causing mesenteric colitis.
Treatment is supportive and most cases will settle with conservative
management.
7. A 23 year old man is admitted to hospital with diarrhoea and severe abdominal
pain. He was previously well and his illness has lasted 18 hours.
You answered Irritable bowel syndrome
The correct answer is Campylobacter jejuni infection
Severe abdominal pain tends to favour Campylobacter infection.
Diarrhoea
World Health Organisation definitions
Diarrhoea: > 3 loose or watery stool per day
Acute diarrhoea < 14 days
Chronic diarrhoea > 14 days
Acute Diarrhoea
Gastroenteritis May be accompanied by abdominal pain or
nausea/vomiting
Diverticulitis Classically causes left lower quadrant pain, diarrhoea
and fever
Antibiotic therapy More common with broad spectrum antibiotics
Clostridium difficile is also seen with antibiotic use
Constipation causing
overflow
A history of alternating diarrhoea and constipation may
be given
May lead to faecal incontinence in the elderly
Chronic Diarrhoea
Irritable
bowel
syndrome
Extremely common. The most consistent features are abdominal pain,
bloating and change in bowel habit. Patients may be divided into
those with diarrhoea predominant IBS and those with constipation
predominant IBS.
Features such as lethargy, nausea, backache and bladder symptoms
may also be present
Ulcerative
colitis
Bloody diarrhoea may be seen. Crampy abdominal pain and weight
loss are also common. Faecal urgency and tenesmus may occur
Crohn's
disease
Crampy abdominal pains and diarrhoea. Bloody diarrhoea less
common than in ulcerative colitis. Other features include
malabsorption, mouth ulcers perianal disease and intestinal
obstruction
Colorectal
cancer
Symptoms depend on the site of the lesion but include diarrhoea,
rectal bleeding, anaemia and constitutional symptoms e.g. Weight loss
and anorexia
Coeliac
disease
 In children may present with failure to thrive, diarrhoea and
abdominal distension
 In adults lethargy, anaemia, diarrhoea and weight loss are
seen. Other autoimmune conditions may coexist
Other conditions associated with diarrhoea include:
 Thyrotoxicosis
 Laxative abuse
 Appendicitis with pelvic abscess or pelvic appendix
 Radiation enteritis
Diagnosis
Stool culture
Abdominal and digital rectal examination
Consider colonoscopy (radiological studies unhelpful)
Thyroid function tests, serum calcium, anti endomysial antibodies, glucose
A 6 year old child presents with colicky abdominal pain, vomiting and the passage of
red current jelly stool per rectum. On examination the child has a tender abdomen and
a palpable mass in the right upper quadrant. Imaging shows an intussusception. Which
of the conditions below is least recognised as a precipitant
A. Inflammation of Payers patches
B. Cystic fibrosis
C. Meckels diverticulum
D. Mesenteric cyst
E. Mucosal polyps
Mesenteric cysts may be associated with intra abdominal catastrophes where these
occur they are typically either intestinal volvulus or intestinal infarction. They seldom
cause intussusception. Cystic fibrosis may lead to the formation of meconium ileus
equivalent and plugs may occasionally serve as the lead points for an intussusception.
Intussusception- Paediatric
Intussusception typcially presents with colicky abdominal pain and vomiting. The
telescoping of the bowel produces mucosal ischaemia and bleeding may occur
resulting in the passage of "red current jelly" stools. Recognised causes include
lumenal pathologies such as polyps, lymphadenopathy and diseases such as cystic
fibrosis. Idiopathic intussceception of the ileocaecal valve and terminal ileum is the
most common variant and typically affects young children and toddlers.
The diagnosis is usually made by abdominal ultrasound investigation. The decision as
to the optimal treatment is dictated by the patients physiological status and abdominal
signs. In general children who are unstable with localising peritoneal signs should
undergo laparotomy as should those in whom attempted radiological reduction has
failed.
In relatively well children without localising signs attempted hydrostatic reduction
under fluroscopic guidance is the usual treatment.
Which one of the following is least likely to cause malabsorption?
A. Primary biliary cirrhosis
B. Ileo-colic bypass
C. Chronic pancreatitis
D. Whipples disease
E. Hartmans procedure
In a Hartmans procedure the sigmoid colon is removed and an end colostomy is
fashioned. The bowel remains in continuity and no absorptive ability is lost.
An ileo-colic bypass leaves a redundant loop of small bowel in continuity, where the
contents will stagnate and bacterial overgrowth will occur. Therefore this is
recognised cause of malabsorption.
Malabsorption
Malabsorption is characterised by diarrhoea, steatorrhoea and weight loss. Causes
may be broadly divided into intestinal (e.g. villous atrophy), pancreatic (deficiency of
pancreatic enzyme production or secretion) and biliary (deficiency of bile-salts
needed for emulsification of fats)
Intestinal causes of malabsorption
 coeliac disease
 Crohn's disease
 tropical sprue
 Whipple's disease
 Giardiasis
 brush border enzyme deficiencies (e.g. lactase insufficiency)
Pancreatic causes of malabsorption
 chronic pancreatitis
 cystic fibrosis
 pancreatic cancer
Biliary causes of malabsorption
 biliary obstruction
 primary biliary cirrhosis
Other causes
 bacterial overgrowth (e.g. systemic sclerosis, diverticulae, blind loop)
 short bowel syndrome
 lymphoma
Theme: Intra abdominal malignancies
A. Metastatic adenocarcinoma of the pancreas
B. Metastatic appendiceal carcinoid
C. Metastatic colonic cancer
D. Pseudomyxoma peritonei
E. MALT lymphoma
F. Retroperitoneal liposarcoma
G. Retroperitoneal fibrosis
For the disease given please give the most likely primary disease process. Each option
may be used once, more than once or not at all.
10. A 32 year old man is admitted with a distended tense abdomen. He previously
underwent a difficult appendicectomy 1 year previously and was discharged.
At laparotomy the abdomen is filled with a gelatinous substance.
You answered Metastatic appendiceal carcinoid
The correct answer is Pseudomyxoma peritonei
Pseudomyxoma is classically associated with mucin production and the
appendix is the commonest source.
11. A 62 year old man is admitted with dull lower back pain and abdominal
discomfort. On examination he is hypertensive and a lower abdominal fullness
is elicited on examination. An abdominal ultrasound demonstrates
hydronephrosis and intravenous urography demonstrated medially displaced
ureters. A CT scan shows a periaortic mass.
You answered Metastatic colonic cancer
The correct answer is Retroperitoneal fibrosis
Retroperitoneal fibrosis is an uncommon condition and its aetiology is poorly
understood. In a significant proportion the ureters are displaced medially. In
most retroperitoneal malignancies they are displaced laterally. Hypertension is
another common finding. A CT scan will often show a para-aortic mass
12. A 48 year old lady is admitted with abdominal distension. On examination she
is cachectic and has ascites. Her CA19-9 returns highly elevated.
Metastatic adenocarcinoma of the pancreas
Although not specific CA 19-9 in the context of this history is highly
suggestive of pancreatic cancer over the other scenarios.
Pseudomyxoma peritoneii- Curative
treatment is peritonectomy (Sugarbaker
procedure) and heated intra peritoneal
chemotherapy.
Pseudomyxoma Peritonei
 Rare mucinous tumour
 Most commonly arising from the appendix (other abdominal viscera are also
recognised as primary sites)
 Incidence of 1-2/1,000,000 per year
 The disease is characterised by the accumulation of large amounts of
mucinous material in the abdominal cavity
Treatment
Is usually surgical and consists of cytoreductive surgery (and often peritonectomy c.f
Sugarbaker procedure) combined with intra peritoneal chemotherapy with mitomycin
C.
Survival is related to the quality of primary treatment and in Sugarbakers own centre
5 year survival rates of 75% have been quoted. Patients with disseminated
intraperitoneal malignancy from another source fare far worse.
In selected patients a second look laparotomy is advocated and some practice this
routinely.
Theme: Abdominal pain
A. Acute mesenteric embolus
B. Acute on chronic mesenteric ischaemia
C. Mesenteric vein thrombosis
D. Ruptured abdominal aortic aneurysm
E. Pancreatitis
F. Appendicitis
G. Acute cholecystitis
Please select the most likely underlying diagnosis from the list above. Each option
may be used once, more than once or not at all.
13. A 72 year old man collapses with sudden onset abdominal pain. He has been
suffering from back pain recently and has been taking ibuprofen.
Ruptured abdominal aortic aneurysm
Back pain is a common feature with expanding aneurysms and may be miss
classified as being of musculoskeletal origin.
14. A 73 year old women collapses with sudden onset of abdominal pain and the
passes a large amount of diarrhoea. On admission she is vomiting repeatedly.
She has recently been discharged from hospital following a myocardial infarct
but recovered well.
You answered Mesenteric vein thrombosis
The correct answer is Acute mesenteric embolus
Sudden onset of abdominal pain and forceful bowel evacuation are features of
acute mesenteric infarct.
15. A 66 year old man has been suffering from weight loss and develops severe
abdominal pain. He is admitted to hospital and undergoes a laparotomy. At
operation the entire small bowel is infarcted and only the left colon is viable.
You answered Mesenteric vein thrombosis
The correct answer is Acute on chronic mesenteric ischaemia
This man is likely to have underlying chronic mesenteric vascular disease.
Only 15% of emboli will occlude SMA orifice leading to entire small bowel
infarct. The background history of weight loss also favours an acute on
chronic event.
Mesenteric vessel disease
Mesenteric ischaemia accounts for 1 in 1000 acute surgical admissions. It is primarily
caused by arterial embolism resulting in infarction of the colon. It is more likely to
occur in areas such as the splenic flexure that are located at the borders of the territory
supplied by the superior and inferior mesenteric arteries.
Types
Acute mesenteric
embolus
(commonest 50%)
 Sudden onset abdominal pain followed by profuse
diarrhoea.
 May be associated with vomiting.
 Rapid clinical deterioration.
 Serological tests: WCC, lactate, amylase may all be
abnormal particularly in established disease. These can be
normal in the early phases.
Acute on chronic
mesenteric
ischaemia
 Usually longer prodromal history.
 Post prandial abdominal discomfort and weight loss are
dominant features. Patients will usually present with an
acute on chronic event, but otherwise will tend not to
present until mesenteric flow is reduced by greater than
80%.
 When acute thrombosis occurs presentation may be as
above. In the chronic setting the symptoms will often be
those of ischaemic colitis (mucosa is the most sensitive
area to this insult).
Mesenteric vein
thrombosis
 Usually a history over weeks.
 Overt abdominal signs and symptoms will not occur until
venous thrombosis has reached a stage to compromise
arterial inflow.
 Thrombophilia accounts for 60% of cases.
Low flow
mesenteric
infarction
 This occurs in patients with multiple co morbidities in
whom mesenteric perfusion is significantly compromised
by overuse of inotropes or background cardiovascular
compromise.
 The end result is that the bowel is not adequately perfused
and infarcts occur from the mucosa outwards.
Diagnosis
 Serological tests: WCC, lactate, CRP, amylase (can be normal in early
disease).
 Cornerstone for diagnosis of arterial AND venous mesenteric disease is CT
angiography scanning in the arterial phase with thin slices (<5mm). Venous
phase contrast is not helpful.
 SMA duplex USS is useful in the evaluation of proximal SMA disease in
patients with chronic mesenteric ischaemia.
 MRI is of limited use due to gut peristalsis and movement artefact.
Management
 Overt signs of peritonism: Laparotomy
 Mesenteric vein thrombosis: If no peritonism: Medical management with IV
heparin
 At operation limited resection of frankly necrotic bowel with view to relook
laparotomy at 24-48h. In the interim urgent bowel revascularisation via
endovascular (preferred) or surgery.
Prognosis
Overall poor. Best outlook is from an acute ischaemia from an embolic event where
surgery occurs within 12h. Survival may be 50%. This falls to 30% with treatment
delay. The other conditions carry worse survival figures.
Theme: Surgical incisions
A. Lanz incision
B. Gridiron incision
C. Kochers incision
D. Rutherford Morrison
E. Rooftop incision
F. McEvedy Incision
G. Lothissen Incision
Please select the most appropriate incision for the procedure described. Each option may be
used once, more than once or not at all.
16. A 78 year old lady is admitted with an incarcerated femoral hernia. Abdominal signs
are absent and there are no symptoms of obstruction. AXR is normal.
McEvedy Incision
From the list the McEvedy approach is the most appropriate. The Lothissen incision
may compromise the posterior wall of the inguinal canal and is best avoided. The
author prefers a limited pfannenstial type incision for this procedure, as it gives
better control of the hernia, but this is not on the list.
17. A 15 year old girl presents with right iliac fossa pain and guarding, pregnancy test is
negative and WCC is 16.
Lanz incision
She requires an appendicectomy although there is an increasing vogue for
performing this procedure laparoscopically an open procedure is entirely suitable.
However, although both a Gridiron and Lanz incision are suitable for
appendicectomy a Lanz will give a superior cosmetic result and would be the
preferred option for most young females.
18. A 45 year old man is due to undergo a live donor renal transplant. This will be his
first procedure.
Rutherford Morrison
The Rutherford Morrison incision will typically give access to the iliac vessels and
bladder for the procedure
Abdominal incisions
Theme in January 2012 exam
Midline incision  Commonest approach to the abdomen
 Structures divided: linea alba, transversalis fascia, extraperitoneal
fat, peritoneum (avoid falciform ligament above the umbilicus)
 Bladder can be accessed via an extraperitoneal approach through
the space of Retzius
Paramedian
incision
 Parallel to the midline (about 3-4cm)
 Structures divided/retracted: anterior rectus sheath, rectus
(retracted), posterior rectus sheath, transversalis fascia,
extraperitoneal fat, peritoneum
 Incision is closed in layers
Battle  Similar location to paramedian but rectus displaced medially (and
thus denervated)
 Now seldom used
Kocher's Incision under right subcostal margin e.g. Cholecystectomy (open)
Lanz Incision in right iliac fossa e.g. Appendicectomy
Gridiron Oblique incision centered over McBurneys point- usually appendicectomy
(less cosmetically acceptable than Lanz
Gable Rooftop incision
Pfannenstiel's Transverse supra pubic, primarily used to access pelvic organs
McEvedy's Groin incision e.g. Emergency repair strangulated femoral hernia
Rutherford
Morrison
Extraperitoneal approach to left or right lower quadrants. Gives excellent
access to iliac vessels and is the approach of choice for first time renal
transplantation.
Image sourced from Wikipedia
Theme: Acute abdominal pain
A. Appendicitis
B. Henoch Schonlein purpura
C. Diabetes mellitus
D. Intussusception
E. Mittelschmerz
F. Pneumonia
G. Sickle cell crisis
H. Spontaneous bacterial peritonitis
I. Rupure of follicular cyst
Please select the most likely cause of abdominal pain for the scenario given. Each
option may be used once, more than once or not at all.
19. An 11 month-old girl develops sudden onset abdominal pain. She has a high
pitched scream and draws up her legs. Her BP is 90/40 mm/Hg, her pulse
118/min and abdominal examination is normal.
Intussusception
Intussusception should be considered in toddlers and infants presenting with
screaming attacks. The child often has a history of being unwell for one to
three days prior to presentation. The child may pass bloody mucus stool,
which is a late sign. Examination of the abdomen is often normal as the
sausage mass in the right upper quadrant is difficult to feel.
20. An 8 year-old West Indian boy presents with periumbilical abdominal pain.
He has vomited twice and is refusing fluids. His temperature is 38.1o
C and
blood tests are as follows: Haemoglobin 8 g/dl, WCC 13 x 109
/l, with a
neutrophilia.
Sickle cell crisis
Sickle cell anaemia is characterised by severe chronic haemolytic anaemia
resulting from poorly formed erythrocytes. Painful crises result from vaso-
occlusive episodes, which may occur spontaneously or may be precipitated by
infection. Consider this diagnosis in all children of appropriate ethnic
background.
21. A 15-month-old girl presents with a three day history of periorbital oedema.
She is brought to hospital. On examination she has facial oedema and a tender
distended abdomen. Her temperature is 39o
C and her blood pressure is 90/45
mmHg. There is clinical evidence of poor peripheral perfusion.
Spontaneous bacterial peritonitis
The 15-month-old girl is a patient with nephrotic syndrome. Patients with this
condition are at risk of septicaemia and peritonitis from Streptococcus
pneumoniae, due to the loss of immunoglobulins and opsonins in the urine.
Acute abdominal pain-diagnoses
Conditions presenting with acute abdominal pain
Condition Features Investigations Management
Appendicitis History of
migratory pain.
Fever.
Anorexia.
Evidence of right
iliac fossa
tenderness.
Mild pyrexia.
Differential white cell
count
Pregnancy test
C-Reactive protein
Amylase
Urine dipstick testing
Appendicectomy
Mesenteric
adenitis
Usually recent
upper respiratory
tract infection.
High fever.
Generalised
abdominal
discomfort- true
localised pain and
signs are rare.
Full blood count- may
show slightly raised
white cell count
Urine dipstick often
normal
Abdominal ultrasound
scan - usually no free
fluid
Conservative
management-
appendicectomy if
diagnostic doubt
Mittelschmerz Only seen in
females.
Mid cycle pain.
Usually occurs two
weeks after last
menstrual period.
Pain is usually has
a supra-pubic
location.
Usually subsides
over a 24-48 hour
period.
Full blood count-
normal
Urine dipstick- normal
Abdominal and pelvic
ultrasound- may show a
trace of pelvic free fluid
Manage conservatively
if doubt or symptoms
fail to settle then
laparoscopy
Fitz-Hugh
Curtis
syndrome
Disseminated
infection with
Chlamydia.
Usually seen in
females.
Consists of
evidence of pelvic
inflammatory
disease together
with peri-hepatic
inflammation and
subsequent
adhesion formation.
Abdominal ultrasound
scan- may show free
fluid
High vaginal swabs -
may show evidence of
sexually transmitted
infections
Usually medically
managed- doxycycline
or azithromycin
Abdominal
aortic
Sudden onset of
abdominal pain
Patients who are
haemodynamically
Unstable patients
should undergo
aneurysm
(ruptured)
radiating to the
back in older adults
(look for risk
factors).
Collapse.
May be moribund
on arrival in
casualty, more
stable if contained
haematoma.
Careful clinical
assessment may
reveal pulsatile
mass.
stable should have a CT
scan
immediate surgery
(unless it is not in their
best interests).
Those with evidence of
contained leak on CT
should undergo
immediate surgery
Increasing aneurysmal
size is an indication for
urgent surgical
intervention (that can
wait until the next
working day)
Perforated
peptic ulcer
Sudden onset of
pain (usually
epigastric).
Often preceding
history of upper
abdominal pain.
Soon develop
generalised
abdominal pain.
On examination
may have clinical
evidence of
peritonitis.
Erect CXR may show
free air. A CT scan may
be indicated where there
is diagnostic doubt
Laparotomy
(laparoscopic surgery
for perforated peptic
ulcers is both safe and
feasible in experienced
hands)
Intestinal
obstruction
Colicky abdominal
pain and vomiting
(the nature of
which depends on
the level of the
obstruction).
Abdominal
distension and
constipation (again
depending upon
site of obstruction).
Features of
peritonism may
occur where local
necrosis of bowel
loops is occurring.
A plain abdominal film
may help with making
the diagnosis. A CT
scan may be useful
where diagnostic
uncertainty exists
In those with a virgin
abdomen and lower and
earlier threshold for
laparotomy should exist
than in those who may
have adhesional
obstruction
Mesenteric
infarction
Embolic events
present with sudden
pain and forceful
evacuation.
Acute on chronic
events usually have
Arterial pH and lactate
Arterial phase CT
scanning is the most
sensitive test
Immediate laparotomy
and resection of
affected segments, in
acute embolic events
SMA embolectomy
may be needed.
a longer history and
previous weight
loss.
On examination the
pain is typically
greater than the
physical signs
would suggest.
A. Peritoneal lavage with cetrimide following elective right
hemicolectomy
B. Use of a laparoscopic approach over open surgery
C. Use of talc to coat surgical gloves
D. Performing a Nobles plication of the small bowel
E. Using stapled rather than a hand sewn anastamosis
Laparoscopy results in fewer adhesions. When talc was used to coat surgical gloves it
was a major cause of adhesion formation and withdrawn for that reason. A Nobles
plication is an old fashioned operation which has no place in the prevention of
adhesion formation. Use of an anastamotic stapling device will not influence the
development of adhesions per se although clearly an anastamotic leak will result in
more adhesion formation
Surgical complications
Complications occur in all branches of surgery and require vigilance in their
detection. In many cases anticipating the likely complications and appropriate
avoidance will minimise their occurrence. For the purposes of the MRCS the
important principles to appreciate are:
 The anatomical principles that underpin complications
 The physiological and biochemical derangements that occur
 The most appropriate diagnostic modalities to utilise
 The principles which underpin their management
This is clearly a very broad area and impossible to cover comprehensively. There is
considerable overlap with other topic areas within the website.
Avoiding complications
Some points to hopefully avert complications:
 World Health Organisation checklist- now mandatory prior to all operations
 Prophylactic antibiotics - right dose, right drug, right time.
 Assess DVT/ PE risk and ensure adequate prophylaxis
 MARK site of surgery
 Use tourniquets with caution and with respect for underlying structures
 Remember the danger of end arteries and in situations where they occur avoid
using adrenaline containing solutions and monopolar diathermy.
 Handle tissues with care- devitalised tissue serves as a nidus for infection
 Be very wary of the potential for coupling injuries when using diathermy
during laparoscopic surgery
 The inferior epigastric artery is a favourite target for laparoscopic ports and
surgical drains!
Anatomical principles
Understanding the anatomy of a surgical field will allow appreciation of local and
systemic complications that may occur. For example nerve injuries may occur
following surgery in specific regions the table below lists some of the more important
nerves to consider and mechanisms of injury
Nerve Mechanism
Accessory Posterior triangle lymph node biopsy
Sciatic Posterior approach to hip
Common peroneal Legs in Lloyd Davies position
Long thoracic Axillary node clearance
Pelvic autonomic nerves Pelvic cancer surgery
Recurrent laryngeal nerves During thyroid surgery
Hypoglossal nerve During carotid endarterectomy
Ulnar and median nerves During upper limb fracture repairs
These are just a few. The detailed functional sequelae are particularly important and
will often be tested. In addition to nerve injuries certain procedures carry risks of
visceral or structural injury. Again some particular favourites are given below:
Structure Mechanism
Thoracic duct During thoracic surgery e.g. Pneumonectomy, oesphagectomy
Parathyroid
glands
During difficult thyroid surgery
Ureters During colonic resections/ gynaecological surgery
Bowel
perforation
Use of Verres Needle to establish pneumoperitoneum
Bile duct injury Failure to delineate Calots triangle carefully and careless use of
diathermy
Facial nerve Always at risk during Parotidectomy
Tail of pancreas When ligating splenic hilum
Testicular vessels During re-do open hernia surgery
Hepatic veins During liver mobilisation
Again many could be predicted from the anatomy of the procedure.
Physiological derangements
A very common complication is bleeding and this is covered under the section of
haemorrhagic shock. Another variant is infection either superficial or deep seated.
The organisms are covered under microbiology and the features of sepsis covered
under shock. Do not forget that immunocompromised and elderly patients may
present will atypical physiological parameters.
Selected physiological and biochemical issues are given below:
Complication Physiological/ Biochemical Problem
Arrhythmias following
cardiac surgery
Susceptibility to hypokalaemia (K+
<4.0 in cardiac
patients)
Neurosurgical electrolyte
disturbance
SIADH following cranial surgery causing hyponatraemia
Ileus following
gastrointestinal surgery
Fluid sequestration and loss of electrolytes
Pulmonary oedema
following pneumonectomy
Loss of lung volume makes these patients very sensitive
to fluid overload
Anastamotic leak Generalised sepsis causing mediastinitis or peritonitis
depending on site of leak
Myocardial infarct May follow any type of surgery and in addition to direct
cardiac effects the decreased cardiac output may well
compromise grafts etc.
Try making a short list of problems and causes specific to your own clinical area.
Diagnostic modalities
Depends largely on the suspected complication. In the acutely unwell surgical patient
the following baseline investigations are often helpful:
 Full blood count, urea and electrolytes, C- reactive protein (trend rather than
absolute value), serum calcium, liver function tests, clotting (don't forget to
repeat if on-going bleeding)
 Arterial blood gases
 ECG (+cardiac enzymes if MI suspected)
 Chest x-ray to identify collapse/ consolidation
 Urine analysis for UTI
These will often identify the most common complications.
Special tests
 CT scanning for identification of intra-abdominal abscesses, air and if luminal
contrast is used an anastamotic leak
 Gatrograffin enema- for rectal anastamotic leaks
 Doppler USS of leg veins- for identification of DVT
 CTPA for PE
 Sending peritoneal fluid for U+E (if ureteric injury suspected) or amylase (if
pancreatic injury suspected)
 Echocardiogram if pericardial effusion suspected post cardiac surgery and no
pleural window made.
Management of complications
The guiding principal should be safe and timely intervention. Patients should be
stabilised and if an operation needs to occur in tandem with resuscitation then
generally this should be of a damage limitation type procedure rather than definitive
surgery (which can be more safely undertaken in a stable patient the following day).
Remember that recent surgery is a contra indication to thrombolysis and that in some
patients IV heparin may be preferable to a low molecular weight heparin (easier to
reverse).
As a general rule laparotomies for bleeding should follow the core principle of
quadrant packing and then subsequent pack removal rather than plunging large
clamps into pools of blood. The latter approach invariable worsens the situation is
often accompanied by significant visceral injury particularly when done by the
inexperienced. If packing controls a situation it is entirely acceptable practice to leak
packs in situ and return the patient to ITU for pack removal the subsequent day.
Theme: Abdominal pain
A. Appendicitis
B. Threatened miscarriage
C. Ectopic pregnancy
D. Irritable bowel syndrome
E. Mittelschmerz
F. Pelvic inflammatory disease
G. Adnexial torsion
H. Endometriosis
I. Degenerating fibroid
Please select the most likely cause of abdominal pain for the clinical scenario given.
Each option may be used once, more than once or not at all.
MRCS preparation eMrcs questions   surgery
MRCS preparation eMrcs questions   surgery
MRCS preparation eMrcs questions   surgery
MRCS preparation eMrcs questions   surgery
MRCS preparation eMrcs questions   surgery
MRCS preparation eMrcs questions   surgery
MRCS preparation eMrcs questions   surgery
MRCS preparation eMrcs questions   surgery
MRCS preparation eMrcs questions   surgery
MRCS preparation eMrcs questions   surgery
MRCS preparation eMrcs questions   surgery
MRCS preparation eMrcs questions   surgery
MRCS preparation eMrcs questions   surgery
MRCS preparation eMrcs questions   surgery
MRCS preparation eMrcs questions   surgery
MRCS preparation eMrcs questions   surgery
MRCS preparation eMrcs questions   surgery
MRCS preparation eMrcs questions   surgery
MRCS preparation eMrcs questions   surgery
MRCS preparation eMrcs questions   surgery
MRCS preparation eMrcs questions   surgery
MRCS preparation eMrcs questions   surgery
MRCS preparation eMrcs questions   surgery
MRCS preparation eMrcs questions   surgery
MRCS preparation eMrcs questions   surgery
MRCS preparation eMrcs questions   surgery
MRCS preparation eMrcs questions   surgery
MRCS preparation eMrcs questions   surgery
MRCS preparation eMrcs questions   surgery
MRCS preparation eMrcs questions   surgery
MRCS preparation eMrcs questions   surgery
MRCS preparation eMrcs questions   surgery
MRCS preparation eMrcs questions   surgery
MRCS preparation eMrcs questions   surgery
MRCS preparation eMrcs questions   surgery
MRCS preparation eMrcs questions   surgery
MRCS preparation eMrcs questions   surgery
MRCS preparation eMrcs questions   surgery
MRCS preparation eMrcs questions   surgery
MRCS preparation eMrcs questions   surgery
MRCS preparation eMrcs questions   surgery
MRCS preparation eMrcs questions   surgery
MRCS preparation eMrcs questions   surgery
MRCS preparation eMrcs questions   surgery
MRCS preparation eMrcs questions   surgery
MRCS preparation eMrcs questions   surgery
MRCS preparation eMrcs questions   surgery
MRCS preparation eMrcs questions   surgery
MRCS preparation eMrcs questions   surgery
MRCS preparation eMrcs questions   surgery
MRCS preparation eMrcs questions   surgery
MRCS preparation eMrcs questions   surgery
MRCS preparation eMrcs questions   surgery
MRCS preparation eMrcs questions   surgery
MRCS preparation eMrcs questions   surgery
MRCS preparation eMrcs questions   surgery
MRCS preparation eMrcs questions   surgery
MRCS preparation eMrcs questions   surgery
MRCS preparation eMrcs questions   surgery
MRCS preparation eMrcs questions   surgery
MRCS preparation eMrcs questions   surgery
MRCS preparation eMrcs questions   surgery
MRCS preparation eMrcs questions   surgery
MRCS preparation eMrcs questions   surgery
MRCS preparation eMrcs questions   surgery
MRCS preparation eMrcs questions   surgery
MRCS preparation eMrcs questions   surgery
MRCS preparation eMrcs questions   surgery
MRCS preparation eMrcs questions   surgery
MRCS preparation eMrcs questions   surgery
MRCS preparation eMrcs questions   surgery
MRCS preparation eMrcs questions   surgery
MRCS preparation eMrcs questions   surgery
MRCS preparation eMrcs questions   surgery
MRCS preparation eMrcs questions   surgery
MRCS preparation eMrcs questions   surgery
MRCS preparation eMrcs questions   surgery
MRCS preparation eMrcs questions   surgery
MRCS preparation eMrcs questions   surgery
MRCS preparation eMrcs questions   surgery
MRCS preparation eMrcs questions   surgery
MRCS preparation eMrcs questions   surgery
MRCS preparation eMrcs questions   surgery
MRCS preparation eMrcs questions   surgery
MRCS preparation eMrcs questions   surgery
MRCS preparation eMrcs questions   surgery
MRCS preparation eMrcs questions   surgery
MRCS preparation eMrcs questions   surgery
MRCS preparation eMrcs questions   surgery
MRCS preparation eMrcs questions   surgery
MRCS preparation eMrcs questions   surgery
MRCS preparation eMrcs questions   surgery
MRCS preparation eMrcs questions   surgery
MRCS preparation eMrcs questions   surgery
MRCS preparation eMrcs questions   surgery
MRCS preparation eMrcs questions   surgery
MRCS preparation eMrcs questions   surgery
MRCS preparation eMrcs questions   surgery
MRCS preparation eMrcs questions   surgery
MRCS preparation eMrcs questions   surgery
MRCS preparation eMrcs questions   surgery
MRCS preparation eMrcs questions   surgery
MRCS preparation eMrcs questions   surgery
MRCS preparation eMrcs questions   surgery
MRCS preparation eMrcs questions   surgery
MRCS preparation eMrcs questions   surgery
MRCS preparation eMrcs questions   surgery
MRCS preparation eMrcs questions   surgery
MRCS preparation eMrcs questions   surgery
MRCS preparation eMrcs questions   surgery
MRCS preparation eMrcs questions   surgery
MRCS preparation eMrcs questions   surgery
MRCS preparation eMrcs questions   surgery
MRCS preparation eMrcs questions   surgery
MRCS preparation eMrcs questions   surgery
MRCS preparation eMrcs questions   surgery
MRCS preparation eMrcs questions   surgery
MRCS preparation eMrcs questions   surgery
MRCS preparation eMrcs questions   surgery
MRCS preparation eMrcs questions   surgery
MRCS preparation eMrcs questions   surgery
MRCS preparation eMrcs questions   surgery
MRCS preparation eMrcs questions   surgery
MRCS preparation eMrcs questions   surgery
MRCS preparation eMrcs questions   surgery
MRCS preparation eMrcs questions   surgery
MRCS preparation eMrcs questions   surgery
MRCS preparation eMrcs questions   surgery
MRCS preparation eMrcs questions   surgery
MRCS preparation eMrcs questions   surgery
MRCS preparation eMrcs questions   surgery
MRCS preparation eMrcs questions   surgery
MRCS preparation eMrcs questions   surgery
MRCS preparation eMrcs questions   surgery
MRCS preparation eMrcs questions   surgery
MRCS preparation eMrcs questions   surgery
MRCS preparation eMrcs questions   surgery
MRCS preparation eMrcs questions   surgery
MRCS preparation eMrcs questions   surgery
MRCS preparation eMrcs questions   surgery
MRCS preparation eMrcs questions   surgery
MRCS preparation eMrcs questions   surgery
MRCS preparation eMrcs questions   surgery
MRCS preparation eMrcs questions   surgery
MRCS preparation eMrcs questions   surgery
MRCS preparation eMrcs questions   surgery
MRCS preparation eMrcs questions   surgery
MRCS preparation eMrcs questions   surgery
MRCS preparation eMrcs questions   surgery
MRCS preparation eMrcs questions   surgery
MRCS preparation eMrcs questions   surgery
MRCS preparation eMrcs questions   surgery
MRCS preparation eMrcs questions   surgery
MRCS preparation eMrcs questions   surgery
MRCS preparation eMrcs questions   surgery
MRCS preparation eMrcs questions   surgery
MRCS preparation eMrcs questions   surgery
MRCS preparation eMrcs questions   surgery
MRCS preparation eMrcs questions   surgery
MRCS preparation eMrcs questions   surgery
MRCS preparation eMrcs questions   surgery
MRCS preparation eMrcs questions   surgery
MRCS preparation eMrcs questions   surgery
MRCS preparation eMrcs questions   surgery
MRCS preparation eMrcs questions   surgery
MRCS preparation eMrcs questions   surgery
MRCS preparation eMrcs questions   surgery
MRCS preparation eMrcs questions   surgery
MRCS preparation eMrcs questions   surgery
MRCS preparation eMrcs questions   surgery
MRCS preparation eMrcs questions   surgery
MRCS preparation eMrcs questions   surgery
MRCS preparation eMrcs questions   surgery
MRCS preparation eMrcs questions   surgery
MRCS preparation eMrcs questions   surgery
MRCS preparation eMrcs questions   surgery
MRCS preparation eMrcs questions   surgery
MRCS preparation eMrcs questions   surgery
MRCS preparation eMrcs questions   surgery
MRCS preparation eMrcs questions   surgery
MRCS preparation eMrcs questions   surgery
MRCS preparation eMrcs questions   surgery
MRCS preparation eMrcs questions   surgery
MRCS preparation eMrcs questions   surgery
MRCS preparation eMrcs questions   surgery
MRCS preparation eMrcs questions   surgery
MRCS preparation eMrcs questions   surgery
MRCS preparation eMrcs questions   surgery
MRCS preparation eMrcs questions   surgery
MRCS preparation eMrcs questions   surgery
MRCS preparation eMrcs questions   surgery
MRCS preparation eMrcs questions   surgery
MRCS preparation eMrcs questions   surgery
MRCS preparation eMrcs questions   surgery
MRCS preparation eMrcs questions   surgery
MRCS preparation eMrcs questions   surgery
MRCS preparation eMrcs questions   surgery
MRCS preparation eMrcs questions   surgery
MRCS preparation eMrcs questions   surgery
MRCS preparation eMrcs questions   surgery
MRCS preparation eMrcs questions   surgery
MRCS preparation eMrcs questions   surgery
MRCS preparation eMrcs questions   surgery
MRCS preparation eMrcs questions   surgery
MRCS preparation eMrcs questions   surgery
MRCS preparation eMrcs questions   surgery
MRCS preparation eMrcs questions   surgery
MRCS preparation eMrcs questions   surgery
MRCS preparation eMrcs questions   surgery
MRCS preparation eMrcs questions   surgery
MRCS preparation eMrcs questions   surgery
MRCS preparation eMrcs questions   surgery
MRCS preparation eMrcs questions   surgery
MRCS preparation eMrcs questions   surgery
MRCS preparation eMrcs questions   surgery
MRCS preparation eMrcs questions   surgery
MRCS preparation eMrcs questions   surgery
MRCS preparation eMrcs questions   surgery
MRCS preparation eMrcs questions   surgery
MRCS preparation eMrcs questions   surgery
MRCS preparation eMrcs questions   surgery
MRCS preparation eMrcs questions   surgery
MRCS preparation eMrcs questions   surgery
MRCS preparation eMrcs questions   surgery
MRCS preparation eMrcs questions   surgery
MRCS preparation eMrcs questions   surgery
MRCS preparation eMrcs questions   surgery
MRCS preparation eMrcs questions   surgery
MRCS preparation eMrcs questions   surgery
MRCS preparation eMrcs questions   surgery
MRCS preparation eMrcs questions   surgery
MRCS preparation eMrcs questions   surgery
MRCS preparation eMrcs questions   surgery
MRCS preparation eMrcs questions   surgery
MRCS preparation eMrcs questions   surgery
MRCS preparation eMrcs questions   surgery
MRCS preparation eMrcs questions   surgery
MRCS preparation eMrcs questions   surgery
MRCS preparation eMrcs questions   surgery
MRCS preparation eMrcs questions   surgery
MRCS preparation eMrcs questions   surgery
MRCS preparation eMrcs questions   surgery
MRCS preparation eMrcs questions   surgery
MRCS preparation eMrcs questions   surgery
MRCS preparation eMrcs questions   surgery
MRCS preparation eMrcs questions   surgery
MRCS preparation eMrcs questions   surgery
MRCS preparation eMrcs questions   surgery
MRCS preparation eMrcs questions   surgery
MRCS preparation eMrcs questions   surgery
MRCS preparation eMrcs questions   surgery
MRCS preparation eMrcs questions   surgery
MRCS preparation eMrcs questions   surgery
MRCS preparation eMrcs questions   surgery
MRCS preparation eMrcs questions   surgery
MRCS preparation eMrcs questions   surgery
MRCS preparation eMrcs questions   surgery
MRCS preparation eMrcs questions   surgery
MRCS preparation eMrcs questions   surgery
MRCS preparation eMrcs questions   surgery
MRCS preparation eMrcs questions   surgery
MRCS preparation eMrcs questions   surgery
MRCS preparation eMrcs questions   surgery
MRCS preparation eMrcs questions   surgery
MRCS preparation eMrcs questions   surgery
MRCS preparation eMrcs questions   surgery
MRCS preparation eMrcs questions   surgery
MRCS preparation eMrcs questions   surgery
MRCS preparation eMrcs questions   surgery
MRCS preparation eMrcs questions   surgery
MRCS preparation eMrcs questions   surgery
MRCS preparation eMrcs questions   surgery
MRCS preparation eMrcs questions   surgery
MRCS preparation eMrcs questions   surgery
MRCS preparation eMrcs questions   surgery
MRCS preparation eMrcs questions   surgery
MRCS preparation eMrcs questions   surgery
MRCS preparation eMrcs questions   surgery
MRCS preparation eMrcs questions   surgery
MRCS preparation eMrcs questions   surgery
MRCS preparation eMrcs questions   surgery
MRCS preparation eMrcs questions   surgery
MRCS preparation eMrcs questions   surgery
MRCS preparation eMrcs questions   surgery
MRCS preparation eMrcs questions   surgery
MRCS preparation eMrcs questions   surgery
MRCS preparation eMrcs questions   surgery
MRCS preparation eMrcs questions   surgery
MRCS preparation eMrcs questions   surgery
MRCS preparation eMrcs questions   surgery
MRCS preparation eMrcs questions   surgery
MRCS preparation eMrcs questions   surgery
MRCS preparation eMrcs questions   surgery
MRCS preparation eMrcs questions   surgery
MRCS preparation eMrcs questions   surgery
MRCS preparation eMrcs questions   surgery
MRCS preparation eMrcs questions   surgery
MRCS preparation eMrcs questions   surgery
MRCS preparation eMrcs questions   surgery
MRCS preparation eMrcs questions   surgery
MRCS preparation eMrcs questions   surgery
MRCS preparation eMrcs questions   surgery
MRCS preparation eMrcs questions   surgery
MRCS preparation eMrcs questions   surgery
MRCS preparation eMrcs questions   surgery
MRCS preparation eMrcs questions   surgery
MRCS preparation eMrcs questions   surgery
MRCS preparation eMrcs questions   surgery
MRCS preparation eMrcs questions   surgery
MRCS preparation eMrcs questions   surgery
MRCS preparation eMrcs questions   surgery
MRCS preparation eMrcs questions   surgery
MRCS preparation eMrcs questions   surgery
MRCS preparation eMrcs questions   surgery
MRCS preparation eMrcs questions   surgery
MRCS preparation eMrcs questions   surgery
MRCS preparation eMrcs questions   surgery
MRCS preparation eMrcs questions   surgery
MRCS preparation eMrcs questions   surgery
MRCS preparation eMrcs questions   surgery
MRCS preparation eMrcs questions   surgery
MRCS preparation eMrcs questions   surgery
MRCS preparation eMrcs questions   surgery
MRCS preparation eMrcs questions   surgery
MRCS preparation eMrcs questions   surgery
MRCS preparation eMrcs questions   surgery
MRCS preparation eMrcs questions   surgery
MRCS preparation eMrcs questions   surgery
MRCS preparation eMrcs questions   surgery
MRCS preparation eMrcs questions   surgery
MRCS preparation eMrcs questions   surgery
MRCS preparation eMrcs questions   surgery
MRCS preparation eMrcs questions   surgery
MRCS preparation eMrcs questions   surgery
MRCS preparation eMrcs questions   surgery
MRCS preparation eMrcs questions   surgery
MRCS preparation eMrcs questions   surgery
MRCS preparation eMrcs questions   surgery
MRCS preparation eMrcs questions   surgery
MRCS preparation eMrcs questions   surgery
MRCS preparation eMrcs questions   surgery
MRCS preparation eMrcs questions   surgery
MRCS preparation eMrcs questions   surgery
MRCS preparation eMrcs questions   surgery
MRCS preparation eMrcs questions   surgery
MRCS preparation eMrcs questions   surgery
MRCS preparation eMrcs questions   surgery
MRCS preparation eMrcs questions   surgery
MRCS preparation eMrcs questions   surgery
MRCS preparation eMrcs questions   surgery
MRCS preparation eMrcs questions   surgery
MRCS preparation eMrcs questions   surgery
MRCS preparation eMrcs questions   surgery
MRCS preparation eMrcs questions   surgery
MRCS preparation eMrcs questions   surgery
MRCS preparation eMrcs questions   surgery
MRCS preparation eMrcs questions   surgery
MRCS preparation eMrcs questions   surgery
MRCS preparation eMrcs questions   surgery
MRCS preparation eMrcs questions   surgery
MRCS preparation eMrcs questions   surgery
MRCS preparation eMrcs questions   surgery
MRCS preparation eMrcs questions   surgery
MRCS preparation eMrcs questions   surgery
MRCS preparation eMrcs questions   surgery
MRCS preparation eMrcs questions   surgery
MRCS preparation eMrcs questions   surgery
MRCS preparation eMrcs questions   surgery
MRCS preparation eMrcs questions   surgery
MRCS preparation eMrcs questions   surgery
MRCS preparation eMrcs questions   surgery
MRCS preparation eMrcs questions   surgery
MRCS preparation eMrcs questions   surgery
MRCS preparation eMrcs questions   surgery
MRCS preparation eMrcs questions   surgery
MRCS preparation eMrcs questions   surgery
MRCS preparation eMrcs questions   surgery
MRCS preparation eMrcs questions   surgery
MRCS preparation eMrcs questions   surgery
MRCS preparation eMrcs questions   surgery
MRCS preparation eMrcs questions   surgery
MRCS preparation eMrcs questions   surgery
MRCS preparation eMrcs questions   surgery
MRCS preparation eMrcs questions   surgery
MRCS preparation eMrcs questions   surgery
MRCS preparation eMrcs questions   surgery
MRCS preparation eMrcs questions   surgery
MRCS preparation eMrcs questions   surgery
MRCS preparation eMrcs questions   surgery
MRCS preparation eMrcs questions   surgery
MRCS preparation eMrcs questions   surgery
MRCS preparation eMrcs questions   surgery
MRCS preparation eMrcs questions   surgery
MRCS preparation eMrcs questions   surgery
MRCS preparation eMrcs questions   surgery
MRCS preparation eMrcs questions   surgery
MRCS preparation eMrcs questions   surgery
MRCS preparation eMrcs questions   surgery
MRCS preparation eMrcs questions   surgery
MRCS preparation eMrcs questions   surgery
MRCS preparation eMrcs questions   surgery
MRCS preparation eMrcs questions   surgery
MRCS preparation eMrcs questions   surgery
MRCS preparation eMrcs questions   surgery
MRCS preparation eMrcs questions   surgery
MRCS preparation eMrcs questions   surgery
MRCS preparation eMrcs questions   surgery
MRCS preparation eMrcs questions   surgery
MRCS preparation eMrcs questions   surgery
MRCS preparation eMrcs questions   surgery
MRCS preparation eMrcs questions   surgery
MRCS preparation eMrcs questions   surgery
MRCS preparation eMrcs questions   surgery
MRCS preparation eMrcs questions   surgery
MRCS preparation eMrcs questions   surgery
MRCS preparation eMrcs questions   surgery
MRCS preparation eMrcs questions   surgery
MRCS preparation eMrcs questions   surgery
MRCS preparation eMrcs questions   surgery
MRCS preparation eMrcs questions   surgery
MRCS preparation eMrcs questions   surgery
MRCS preparation eMrcs questions   surgery
MRCS preparation eMrcs questions   surgery
MRCS preparation eMrcs questions   surgery
MRCS preparation eMrcs questions   surgery
MRCS preparation eMrcs questions   surgery
MRCS preparation eMrcs questions   surgery
MRCS preparation eMrcs questions   surgery
MRCS preparation eMrcs questions   surgery
MRCS preparation eMrcs questions   surgery
MRCS preparation eMrcs questions   surgery
MRCS preparation eMrcs questions   surgery
MRCS preparation eMrcs questions   surgery
MRCS preparation eMrcs questions   surgery
MRCS preparation eMrcs questions   surgery
MRCS preparation eMrcs questions   surgery
MRCS preparation eMrcs questions   surgery
MRCS preparation eMrcs questions   surgery
MRCS preparation eMrcs questions   surgery
MRCS preparation eMrcs questions   surgery
MRCS preparation eMrcs questions   surgery
MRCS preparation eMrcs questions   surgery
MRCS preparation eMrcs questions   surgery
MRCS preparation eMrcs questions   surgery
MRCS preparation eMrcs questions   surgery
MRCS preparation eMrcs questions   surgery
MRCS preparation eMrcs questions   surgery
MRCS preparation eMrcs questions   surgery
MRCS preparation eMrcs questions   surgery
MRCS preparation eMrcs questions   surgery
MRCS preparation eMrcs questions   surgery
MRCS preparation eMrcs questions   surgery
MRCS preparation eMrcs questions   surgery
MRCS preparation eMrcs questions   surgery
MRCS preparation eMrcs questions   surgery
MRCS preparation eMrcs questions   surgery
MRCS preparation eMrcs questions   surgery
MRCS preparation eMrcs questions   surgery
MRCS preparation eMrcs questions   surgery
MRCS preparation eMrcs questions   surgery
MRCS preparation eMrcs questions   surgery
MRCS preparation eMrcs questions   surgery
MRCS preparation eMrcs questions   surgery
MRCS preparation eMrcs questions   surgery
MRCS preparation eMrcs questions   surgery
MRCS preparation eMrcs questions   surgery
MRCS preparation eMrcs questions   surgery
MRCS preparation eMrcs questions   surgery
MRCS preparation eMrcs questions   surgery
MRCS preparation eMrcs questions   surgery
MRCS preparation eMrcs questions   surgery
MRCS preparation eMrcs questions   surgery
MRCS preparation eMrcs questions   surgery
MRCS preparation eMrcs questions   surgery
MRCS preparation eMrcs questions   surgery
MRCS preparation eMrcs questions   surgery
MRCS preparation eMrcs questions   surgery
MRCS preparation eMrcs questions   surgery
MRCS preparation eMrcs questions   surgery
MRCS preparation eMrcs questions   surgery
MRCS preparation eMrcs questions   surgery
MRCS preparation eMrcs questions   surgery
MRCS preparation eMrcs questions   surgery
MRCS preparation eMrcs questions   surgery
MRCS preparation eMrcs questions   surgery
MRCS preparation eMrcs questions   surgery
MRCS preparation eMrcs questions   surgery
MRCS preparation eMrcs questions   surgery
MRCS preparation eMrcs questions   surgery
MRCS preparation eMrcs questions   surgery
MRCS preparation eMrcs questions   surgery
MRCS preparation eMrcs questions   surgery
MRCS preparation eMrcs questions   surgery
MRCS preparation eMrcs questions   surgery
MRCS preparation eMrcs questions   surgery
MRCS preparation eMrcs questions   surgery
MRCS preparation eMrcs questions   surgery
MRCS preparation eMrcs questions   surgery
MRCS preparation eMrcs questions   surgery
MRCS preparation eMrcs questions   surgery
MRCS preparation eMrcs questions   surgery
MRCS preparation eMrcs questions   surgery
MRCS preparation eMrcs questions   surgery
MRCS preparation eMrcs questions   surgery
MRCS preparation eMrcs questions   surgery
MRCS preparation eMrcs questions   surgery
MRCS preparation eMrcs questions   surgery
MRCS preparation eMrcs questions   surgery
MRCS preparation eMrcs questions   surgery
MRCS preparation eMrcs questions   surgery
MRCS preparation eMrcs questions   surgery
MRCS preparation eMrcs questions   surgery
MRCS preparation eMrcs questions   surgery
MRCS preparation eMrcs questions   surgery
MRCS preparation eMrcs questions   surgery
MRCS preparation eMrcs questions   surgery
MRCS preparation eMrcs questions   surgery
MRCS preparation eMrcs questions   surgery
MRCS preparation eMrcs questions   surgery
MRCS preparation eMrcs questions   surgery
MRCS preparation eMrcs questions   surgery
MRCS preparation eMrcs questions   surgery
MRCS preparation eMrcs questions   surgery
MRCS preparation eMrcs questions   surgery
MRCS preparation eMrcs questions   surgery
MRCS preparation eMrcs questions   surgery
MRCS preparation eMrcs questions   surgery
MRCS preparation eMrcs questions   surgery
MRCS preparation eMrcs questions   surgery
MRCS preparation eMrcs questions   surgery
MRCS preparation eMrcs questions   surgery
MRCS preparation eMrcs questions   surgery
MRCS preparation eMrcs questions   surgery
MRCS preparation eMrcs questions   surgery
MRCS preparation eMrcs questions   surgery
MRCS preparation eMrcs questions   surgery
MRCS preparation eMrcs questions   surgery
MRCS preparation eMrcs questions   surgery
MRCS preparation eMrcs questions   surgery
MRCS preparation eMrcs questions   surgery
MRCS preparation eMrcs questions   surgery
MRCS preparation eMrcs questions   surgery
MRCS preparation eMrcs questions   surgery
MRCS preparation eMrcs questions   surgery
MRCS preparation eMrcs questions   surgery
MRCS preparation eMrcs questions   surgery
MRCS preparation eMrcs questions   surgery
MRCS preparation eMrcs questions   surgery
MRCS preparation eMrcs questions   surgery
MRCS preparation eMrcs questions   surgery
MRCS preparation eMrcs questions   surgery
MRCS preparation eMrcs questions   surgery
MRCS preparation eMrcs questions   surgery
MRCS preparation eMrcs questions   surgery
MRCS preparation eMrcs questions   surgery
MRCS preparation eMrcs questions   surgery
MRCS preparation eMrcs questions   surgery
MRCS preparation eMrcs questions   surgery
MRCS preparation eMrcs questions   surgery
MRCS preparation eMrcs questions   surgery
MRCS preparation eMrcs questions   surgery
MRCS preparation eMrcs questions   surgery
MRCS preparation eMrcs questions   surgery
MRCS preparation eMrcs questions   surgery
MRCS preparation eMrcs questions   surgery
MRCS preparation eMrcs questions   surgery
MRCS preparation eMrcs questions   surgery
MRCS preparation eMrcs questions   surgery
MRCS preparation eMrcs questions   surgery
MRCS preparation eMrcs questions   surgery
MRCS preparation eMrcs questions   surgery
MRCS preparation eMrcs questions   surgery
MRCS preparation eMrcs questions   surgery
MRCS preparation eMrcs questions   surgery
MRCS preparation eMrcs questions   surgery
MRCS preparation eMrcs questions   surgery
MRCS preparation eMrcs questions   surgery
MRCS preparation eMrcs questions   surgery
MRCS preparation eMrcs questions   surgery
MRCS preparation eMrcs questions   surgery
MRCS preparation eMrcs questions   surgery
MRCS preparation eMrcs questions   surgery
MRCS preparation eMrcs questions   surgery
MRCS preparation eMrcs questions   surgery
MRCS preparation eMrcs questions   surgery
MRCS preparation eMrcs questions   surgery
MRCS preparation eMrcs questions   surgery
MRCS preparation eMrcs questions   surgery
MRCS preparation eMrcs questions   surgery
MRCS preparation eMrcs questions   surgery
MRCS preparation eMrcs questions   surgery
MRCS preparation eMrcs questions   surgery
MRCS preparation eMrcs questions   surgery
MRCS preparation eMrcs questions   surgery
MRCS preparation eMrcs questions   surgery
MRCS preparation eMrcs questions   surgery
MRCS preparation eMrcs questions   surgery
MRCS preparation eMrcs questions   surgery
MRCS preparation eMrcs questions   surgery
MRCS preparation eMrcs questions   surgery
MRCS preparation eMrcs questions   surgery
MRCS preparation eMrcs questions   surgery
MRCS preparation eMrcs questions   surgery
MRCS preparation eMrcs questions   surgery
MRCS preparation eMrcs questions   surgery
MRCS preparation eMrcs questions   surgery
MRCS preparation eMrcs questions   surgery
MRCS preparation eMrcs questions   surgery
MRCS preparation eMrcs questions   surgery
MRCS preparation eMrcs questions   surgery
MRCS preparation eMrcs questions   surgery
MRCS preparation eMrcs questions   surgery
MRCS preparation eMrcs questions   surgery
MRCS preparation eMrcs questions   surgery
MRCS preparation eMrcs questions   surgery
MRCS preparation eMrcs questions   surgery
MRCS preparation eMrcs questions   surgery
MRCS preparation eMrcs questions   surgery
MRCS preparation eMrcs questions   surgery
MRCS preparation eMrcs questions   surgery
MRCS preparation eMrcs questions   surgery
MRCS preparation eMrcs questions   surgery
MRCS preparation eMrcs questions   surgery
MRCS preparation eMrcs questions   surgery
MRCS preparation eMrcs questions   surgery
MRCS preparation eMrcs questions   surgery
MRCS preparation eMrcs questions   surgery
MRCS preparation eMrcs questions   surgery
MRCS preparation eMrcs questions   surgery
MRCS preparation eMrcs questions   surgery
MRCS preparation eMrcs questions   surgery
MRCS preparation eMrcs questions   surgery
MRCS preparation eMrcs questions   surgery
MRCS preparation eMrcs questions   surgery
MRCS preparation eMrcs questions   surgery
MRCS preparation eMrcs questions   surgery
MRCS preparation eMrcs questions   surgery
MRCS preparation eMrcs questions   surgery
MRCS preparation eMrcs questions   surgery
MRCS preparation eMrcs questions   surgery
MRCS preparation eMrcs questions   surgery
MRCS preparation eMrcs questions   surgery
MRCS preparation eMrcs questions   surgery
MRCS preparation eMrcs questions   surgery
MRCS preparation eMrcs questions   surgery
MRCS preparation eMrcs questions   surgery
MRCS preparation eMrcs questions   surgery
MRCS preparation eMrcs questions   surgery
MRCS preparation eMrcs questions   surgery
MRCS preparation eMrcs questions   surgery
MRCS preparation eMrcs questions   surgery
MRCS preparation eMrcs questions   surgery
MRCS preparation eMrcs questions   surgery
MRCS preparation eMrcs questions   surgery
MRCS preparation eMrcs questions   surgery
MRCS preparation eMrcs questions   surgery
MRCS preparation eMrcs questions   surgery
MRCS preparation eMrcs questions   surgery
MRCS preparation eMrcs questions   surgery
MRCS preparation eMrcs questions   surgery
MRCS preparation eMrcs questions   surgery
MRCS preparation eMrcs questions   surgery
MRCS preparation eMrcs questions   surgery
MRCS preparation eMrcs questions   surgery
MRCS preparation eMrcs questions   surgery
MRCS preparation eMrcs questions   surgery
MRCS preparation eMrcs questions   surgery
MRCS preparation eMrcs questions   surgery
MRCS preparation eMrcs questions   surgery
MRCS preparation eMrcs questions   surgery
MRCS preparation eMrcs questions   surgery
MRCS preparation eMrcs questions   surgery
MRCS preparation eMrcs questions   surgery
MRCS preparation eMrcs questions   surgery
MRCS preparation eMrcs questions   surgery
MRCS preparation eMrcs questions   surgery
MRCS preparation eMrcs questions   surgery
MRCS preparation eMrcs questions   surgery
MRCS preparation eMrcs questions   surgery
MRCS preparation eMrcs questions   surgery
MRCS preparation eMrcs questions   surgery
MRCS preparation eMrcs questions   surgery
MRCS preparation eMrcs questions   surgery
MRCS preparation eMrcs questions   surgery
MRCS preparation eMrcs questions   surgery
MRCS preparation eMrcs questions   surgery
MRCS preparation eMrcs questions   surgery
MRCS preparation eMrcs questions   surgery
MRCS preparation eMrcs questions   surgery
MRCS preparation eMrcs questions   surgery
MRCS preparation eMrcs questions   surgery
MRCS preparation eMrcs questions   surgery
MRCS preparation eMrcs questions   surgery
MRCS preparation eMrcs questions   surgery
MRCS preparation eMrcs questions   surgery
MRCS preparation eMrcs questions   surgery
MRCS preparation eMrcs questions   surgery
MRCS preparation eMrcs questions   surgery
MRCS preparation eMrcs questions   surgery
MRCS preparation eMrcs questions   surgery
MRCS preparation eMrcs questions   surgery
MRCS preparation eMrcs questions   surgery
MRCS preparation eMrcs questions   surgery
MRCS preparation eMrcs questions   surgery
MRCS preparation eMrcs questions   surgery
MRCS preparation eMrcs questions   surgery
MRCS preparation eMrcs questions   surgery
MRCS preparation eMrcs questions   surgery
MRCS preparation eMrcs questions   surgery
MRCS preparation eMrcs questions   surgery
MRCS preparation eMrcs questions   surgery
MRCS preparation eMrcs questions   surgery
MRCS preparation eMrcs questions   surgery
MRCS preparation eMrcs questions   surgery
MRCS preparation eMrcs questions   surgery
MRCS preparation eMrcs questions   surgery
MRCS preparation eMrcs questions   surgery
MRCS preparation eMrcs questions   surgery
MRCS preparation eMrcs questions   surgery
MRCS preparation eMrcs questions   surgery
MRCS preparation eMrcs questions   surgery
MRCS preparation eMrcs questions   surgery
MRCS preparation eMrcs questions   surgery
MRCS preparation eMrcs questions   surgery
MRCS preparation eMrcs questions   surgery
MRCS preparation eMrcs questions   surgery
MRCS preparation eMrcs questions   surgery
MRCS preparation eMrcs questions   surgery
MRCS preparation eMrcs questions   surgery
MRCS preparation eMrcs questions   surgery
MRCS preparation eMrcs questions   surgery
MRCS preparation eMrcs questions   surgery
MRCS preparation eMrcs questions   surgery
MRCS preparation eMrcs questions   surgery
MRCS preparation eMrcs questions   surgery
MRCS preparation eMrcs questions   surgery
MRCS preparation eMrcs questions   surgery
MRCS preparation eMrcs questions   surgery
MRCS preparation eMrcs questions   surgery
MRCS preparation eMrcs questions   surgery
MRCS preparation eMrcs questions   surgery
MRCS preparation eMrcs questions   surgery
MRCS preparation eMrcs questions   surgery
MRCS preparation eMrcs questions   surgery
MRCS preparation eMrcs questions   surgery
MRCS preparation eMrcs questions   surgery
MRCS preparation eMrcs questions   surgery
MRCS preparation eMrcs questions   surgery
MRCS preparation eMrcs questions   surgery
MRCS preparation eMrcs questions   surgery
MRCS preparation eMrcs questions   surgery
MRCS preparation eMrcs questions   surgery
MRCS preparation eMrcs questions   surgery
MRCS preparation eMrcs questions   surgery
MRCS preparation eMrcs questions   surgery
MRCS preparation eMrcs questions   surgery
MRCS preparation eMrcs questions   surgery
MRCS preparation eMrcs questions   surgery
MRCS preparation eMrcs questions   surgery
MRCS preparation eMrcs questions   surgery
MRCS preparation eMrcs questions   surgery
MRCS preparation eMrcs questions   surgery
MRCS preparation eMrcs questions   surgery
MRCS preparation eMrcs questions   surgery
MRCS preparation eMrcs questions   surgery
MRCS preparation eMrcs questions   surgery
MRCS preparation eMrcs questions   surgery
MRCS preparation eMrcs questions   surgery
MRCS preparation eMrcs questions   surgery
MRCS preparation eMrcs questions   surgery
MRCS preparation eMrcs questions   surgery
MRCS preparation eMrcs questions   surgery
MRCS preparation eMrcs questions   surgery
MRCS preparation eMrcs questions   surgery
MRCS preparation eMrcs questions   surgery
MRCS preparation eMrcs questions   surgery
MRCS preparation eMrcs questions   surgery
MRCS preparation eMrcs questions   surgery
MRCS preparation eMrcs questions   surgery
MRCS preparation eMrcs questions   surgery
MRCS preparation eMrcs questions   surgery
MRCS preparation eMrcs questions   surgery
MRCS preparation eMrcs questions   surgery
MRCS preparation eMrcs questions   surgery
MRCS preparation eMrcs questions   surgery
MRCS preparation eMrcs questions   surgery
MRCS preparation eMrcs questions   surgery
MRCS preparation eMrcs questions   surgery
MRCS preparation eMrcs questions   surgery
MRCS preparation eMrcs questions   surgery
MRCS preparation eMrcs questions   surgery
MRCS preparation eMrcs questions   surgery
MRCS preparation eMrcs questions   surgery
MRCS preparation eMrcs questions   surgery
MRCS preparation eMrcs questions   surgery
MRCS preparation eMrcs questions   surgery
MRCS preparation eMrcs questions   surgery
MRCS preparation eMrcs questions   surgery
MRCS preparation eMrcs questions   surgery
MRCS preparation eMrcs questions   surgery
MRCS preparation eMrcs questions   surgery
MRCS preparation eMrcs questions   surgery
MRCS preparation eMrcs questions   surgery
MRCS preparation eMrcs questions   surgery
MRCS preparation eMrcs questions   surgery
MRCS preparation eMrcs questions   surgery
MRCS preparation eMrcs questions   surgery
MRCS preparation eMrcs questions   surgery
MRCS preparation eMrcs questions   surgery
MRCS preparation eMrcs questions   surgery
MRCS preparation eMrcs questions   surgery
MRCS preparation eMrcs questions   surgery
MRCS preparation eMrcs questions   surgery
MRCS preparation eMrcs questions   surgery
MRCS preparation eMrcs questions   surgery
MRCS preparation eMrcs questions   surgery
MRCS preparation eMrcs questions   surgery
MRCS preparation eMrcs questions   surgery
MRCS preparation eMrcs questions   surgery
MRCS preparation eMrcs questions   surgery
MRCS preparation eMrcs questions   surgery
MRCS preparation eMrcs questions   surgery
MRCS preparation eMrcs questions   surgery
MRCS preparation eMrcs questions   surgery
MRCS preparation eMrcs questions   surgery
MRCS preparation eMrcs questions   surgery
MRCS preparation eMrcs questions   surgery
MRCS preparation eMrcs questions   surgery
MRCS preparation eMrcs questions   surgery
MRCS preparation eMrcs questions   surgery
MRCS preparation eMrcs questions   surgery
MRCS preparation eMrcs questions   surgery
MRCS preparation eMrcs questions   surgery
MRCS preparation eMrcs questions   surgery
MRCS preparation eMrcs questions   surgery
MRCS preparation eMrcs questions   surgery
MRCS preparation eMrcs questions   surgery
MRCS preparation eMrcs questions   surgery
MRCS preparation eMrcs questions   surgery
MRCS preparation eMrcs questions   surgery
MRCS preparation eMrcs questions   surgery
MRCS preparation eMrcs questions   surgery
MRCS preparation eMrcs questions   surgery
MRCS preparation eMrcs questions   surgery
MRCS preparation eMrcs questions   surgery
MRCS preparation eMrcs questions   surgery
MRCS preparation eMrcs questions   surgery
MRCS preparation eMrcs questions   surgery
MRCS preparation eMrcs questions   surgery
MRCS preparation eMrcs questions   surgery
MRCS preparation eMrcs questions   surgery
MRCS preparation eMrcs questions   surgery
MRCS preparation eMrcs questions   surgery
MRCS preparation eMrcs questions   surgery
MRCS preparation eMrcs questions   surgery
MRCS preparation eMrcs questions   surgery
MRCS preparation eMrcs questions   surgery
MRCS preparation eMrcs questions   surgery
MRCS preparation eMrcs questions   surgery
MRCS preparation eMrcs questions   surgery
MRCS preparation eMrcs questions   surgery
MRCS preparation eMrcs questions   surgery
MRCS preparation eMrcs questions   surgery
MRCS preparation eMrcs questions   surgery
MRCS preparation eMrcs questions   surgery
MRCS preparation eMrcs questions   surgery
MRCS preparation eMrcs questions   surgery
MRCS preparation eMrcs questions   surgery
MRCS preparation eMrcs questions   surgery
MRCS preparation eMrcs questions   surgery
MRCS preparation eMrcs questions   surgery
MRCS preparation eMrcs questions   surgery
MRCS preparation eMrcs questions   surgery
MRCS preparation eMrcs questions   surgery
MRCS preparation eMrcs questions   surgery
MRCS preparation eMrcs questions   surgery
MRCS preparation eMrcs questions   surgery
MRCS preparation eMrcs questions   surgery
MRCS preparation eMrcs questions   surgery
MRCS preparation eMrcs questions   surgery
MRCS preparation eMrcs questions   surgery
MRCS preparation eMrcs questions   surgery
MRCS preparation eMrcs questions   surgery
MRCS preparation eMrcs questions   surgery
MRCS preparation eMrcs questions   surgery
MRCS preparation eMrcs questions   surgery
MRCS preparation eMrcs questions   surgery
MRCS preparation eMrcs questions   surgery
MRCS preparation eMrcs questions   surgery
MRCS preparation eMrcs questions   surgery
MRCS preparation eMrcs questions   surgery
MRCS preparation eMrcs questions   surgery
MRCS preparation eMrcs questions   surgery
MRCS preparation eMrcs questions   surgery
MRCS preparation eMrcs questions   surgery
MRCS preparation eMrcs questions   surgery
MRCS preparation eMrcs questions   surgery
MRCS preparation eMrcs questions   surgery
MRCS preparation eMrcs questions   surgery
MRCS preparation eMrcs questions   surgery
MRCS preparation eMrcs questions   surgery
MRCS preparation eMrcs questions   surgery
MRCS preparation eMrcs questions   surgery
MRCS preparation eMrcs questions   surgery
MRCS preparation eMrcs questions   surgery
MRCS preparation eMrcs questions   surgery
MRCS preparation eMrcs questions   surgery
MRCS preparation eMrcs questions   surgery
MRCS preparation eMrcs questions   surgery
MRCS preparation eMrcs questions   surgery
MRCS preparation eMrcs questions   surgery
MRCS preparation eMrcs questions   surgery
MRCS preparation eMrcs questions   surgery
MRCS preparation eMrcs questions   surgery
MRCS preparation eMrcs questions   surgery
MRCS preparation eMrcs questions   surgery
MRCS preparation eMrcs questions   surgery
MRCS preparation eMrcs questions   surgery
MRCS preparation eMrcs questions   surgery
MRCS preparation eMrcs questions   surgery
MRCS preparation eMrcs questions   surgery
MRCS preparation eMrcs questions   surgery
MRCS preparation eMrcs questions   surgery
MRCS preparation eMrcs questions   surgery
MRCS preparation eMrcs questions   surgery
MRCS preparation eMrcs questions   surgery
MRCS preparation eMrcs questions   surgery
MRCS preparation eMrcs questions   surgery
MRCS preparation eMrcs questions   surgery
MRCS preparation eMrcs questions   surgery
MRCS preparation eMrcs questions   surgery
MRCS preparation eMrcs questions   surgery
MRCS preparation eMrcs questions   surgery
MRCS preparation eMrcs questions   surgery
MRCS preparation eMrcs questions   surgery
MRCS preparation eMrcs questions   surgery
MRCS preparation eMrcs questions   surgery
MRCS preparation eMrcs questions   surgery
MRCS preparation eMrcs questions   surgery
MRCS preparation eMrcs questions   surgery
MRCS preparation eMrcs questions   surgery
MRCS preparation eMrcs questions   surgery
MRCS preparation eMrcs questions   surgery
MRCS preparation eMrcs questions   surgery
MRCS preparation eMrcs questions   surgery
MRCS preparation eMrcs questions   surgery
MRCS preparation eMrcs questions   surgery
MRCS preparation eMrcs questions   surgery
MRCS preparation eMrcs questions   surgery
MRCS preparation eMrcs questions   surgery
MRCS preparation eMrcs questions   surgery
MRCS preparation eMrcs questions   surgery
MRCS preparation eMrcs questions   surgery
MRCS preparation eMrcs questions   surgery
MRCS preparation eMrcs questions   surgery
MRCS preparation eMrcs questions   surgery
MRCS preparation eMrcs questions   surgery
MRCS preparation eMrcs questions   surgery
MRCS preparation eMrcs questions   surgery
MRCS preparation eMrcs questions   surgery
MRCS preparation eMrcs questions   surgery
MRCS preparation eMrcs questions   surgery
MRCS preparation eMrcs questions   surgery
MRCS preparation eMrcs questions   surgery
MRCS preparation eMrcs questions   surgery
MRCS preparation eMrcs questions   surgery
MRCS preparation eMrcs questions   surgery
MRCS preparation eMrcs questions   surgery
MRCS preparation eMrcs questions   surgery
MRCS preparation eMrcs questions   surgery
MRCS preparation eMrcs questions   surgery
MRCS preparation eMrcs questions   surgery
MRCS preparation eMrcs questions   surgery
MRCS preparation eMrcs questions   surgery
MRCS preparation eMrcs questions   surgery
MRCS preparation eMrcs questions   surgery
MRCS preparation eMrcs questions   surgery
MRCS preparation eMrcs questions   surgery
MRCS preparation eMrcs questions   surgery
MRCS preparation eMrcs questions   surgery
MRCS preparation eMrcs questions   surgery
MRCS preparation eMrcs questions   surgery
MRCS preparation eMrcs questions   surgery
MRCS preparation eMrcs questions   surgery
MRCS preparation eMrcs questions   surgery
MRCS preparation eMrcs questions   surgery
MRCS preparation eMrcs questions   surgery
MRCS preparation eMrcs questions   surgery
MRCS preparation eMrcs questions   surgery
MRCS preparation eMrcs questions   surgery
MRCS preparation eMrcs questions   surgery
MRCS preparation eMrcs questions   surgery
MRCS preparation eMrcs questions   surgery
MRCS preparation eMrcs questions   surgery
MRCS preparation eMrcs questions   surgery
MRCS preparation eMrcs questions   surgery
MRCS preparation eMrcs questions   surgery
MRCS preparation eMrcs questions   surgery
MRCS preparation eMrcs questions   surgery
MRCS preparation eMrcs questions   surgery
MRCS preparation eMrcs questions   surgery
MRCS preparation eMrcs questions   surgery
MRCS preparation eMrcs questions   surgery
MRCS preparation eMrcs questions   surgery
MRCS preparation eMrcs questions   surgery
MRCS preparation eMrcs questions   surgery
MRCS preparation eMrcs questions   surgery
MRCS preparation eMrcs questions   surgery
MRCS preparation eMrcs questions   surgery
MRCS preparation eMrcs questions   surgery
MRCS preparation eMrcs questions   surgery
MRCS preparation eMrcs questions   surgery
MRCS preparation eMrcs questions   surgery
MRCS preparation eMrcs questions   surgery
MRCS preparation eMrcs questions   surgery
MRCS preparation eMrcs questions   surgery
MRCS preparation eMrcs questions   surgery
MRCS preparation eMrcs questions   surgery
MRCS preparation eMrcs questions   surgery
MRCS preparation eMrcs questions   surgery
MRCS preparation eMrcs questions   surgery
MRCS preparation eMrcs questions   surgery

More Related Content

What's hot

Management of enterocutaneous fistula
Management of enterocutaneous fistula Management of enterocutaneous fistula
Management of enterocutaneous fistula Zelalem Semegnew
 
Typhoid ileal perforation
Typhoid ileal perforationTyphoid ileal perforation
Typhoid ileal perforationBashir BnYunus
 
Preparation of a patient of obstructive jaundice and periampullary carcinoma
Preparation of a patient of obstructive jaundice and periampullary carcinomaPreparation of a patient of obstructive jaundice and periampullary carcinoma
Preparation of a patient of obstructive jaundice and periampullary carcinomaDr.Manojit Sarkar
 
Adhesive intestinal obstruction
Adhesive intestinal obstructionAdhesive intestinal obstruction
Adhesive intestinal obstructionBashir BnYunus
 
Chapter 26 Appendix
Chapter 26 AppendixChapter 26 Appendix
Chapter 26 Appendixhuang.shuo
 
Acute cholecystitis..
Acute cholecystitis..Acute cholecystitis..
Acute cholecystitis..Sarif Raza
 
Complications of laparoscopic cholecystectomy, by Dr. Bashab Bijoy Roy, PGT,SMC,
Complications of laparoscopic cholecystectomy, by Dr. Bashab Bijoy Roy, PGT,SMC,Complications of laparoscopic cholecystectomy, by Dr. Bashab Bijoy Roy, PGT,SMC,
Complications of laparoscopic cholecystectomy, by Dr. Bashab Bijoy Roy, PGT,SMC,Dr.Bashab Roy
 
Ventral hernia management
Ventral hernia managementVentral hernia management
Ventral hernia managementmanjil malla
 
Infantile hypertrophic pyloric stenosis
Infantile hypertrophic pyloric stenosisInfantile hypertrophic pyloric stenosis
Infantile hypertrophic pyloric stenosisdrssp1967
 
European hernia society guidelines: Adult Inguinal Hernia (Post operative car...
European hernia society guidelines: Adult Inguinal Hernia (Post operative car...European hernia society guidelines: Adult Inguinal Hernia (Post operative car...
European hernia society guidelines: Adult Inguinal Hernia (Post operative car...Jibran Mohsin
 
Innovative Surgical Techiniques in Hepatobiliary and Pancreatic Surgery
Innovative Surgical Techiniques in Hepatobiliary and Pancreatic SurgeryInnovative Surgical Techiniques in Hepatobiliary and Pancreatic Surgery
Innovative Surgical Techiniques in Hepatobiliary and Pancreatic SurgeryISWANTO SUCANDY, M.D, F.A.C.S
 
Resection & anastomosis of boweL its complications PRANAYA PPT
Resection & anastomosis of boweL its complications PRANAYA PPTResection & anastomosis of boweL its complications PRANAYA PPT
Resection & anastomosis of boweL its complications PRANAYA PPTPRANAYA PANIGRAHI
 

What's hot (20)

Gastric outlet obstruction
Gastric outlet obstructionGastric outlet obstruction
Gastric outlet obstruction
 
Management of enterocutaneous fistula
Management of enterocutaneous fistula Management of enterocutaneous fistula
Management of enterocutaneous fistula
 
Enterocutaneous fistulas
Enterocutaneous fistulasEnterocutaneous fistulas
Enterocutaneous fistulas
 
Typhoid ileal perforation
Typhoid ileal perforationTyphoid ileal perforation
Typhoid ileal perforation
 
Inguinal hernia
Inguinal herniaInguinal hernia
Inguinal hernia
 
Stomas
StomasStomas
Stomas
 
Preparation of a patient of obstructive jaundice and periampullary carcinoma
Preparation of a patient of obstructive jaundice and periampullary carcinomaPreparation of a patient of obstructive jaundice and periampullary carcinoma
Preparation of a patient of obstructive jaundice and periampullary carcinoma
 
Adhesive intestinal obstruction
Adhesive intestinal obstructionAdhesive intestinal obstruction
Adhesive intestinal obstruction
 
Axilla
AxillaAxilla
Axilla
 
VENTRAL HERNIA
VENTRAL HERNIAVENTRAL HERNIA
VENTRAL HERNIA
 
Chapter 26 Appendix
Chapter 26 AppendixChapter 26 Appendix
Chapter 26 Appendix
 
Acute cholecystitis..
Acute cholecystitis..Acute cholecystitis..
Acute cholecystitis..
 
Complications of laparoscopic cholecystectomy, by Dr. Bashab Bijoy Roy, PGT,SMC,
Complications of laparoscopic cholecystectomy, by Dr. Bashab Bijoy Roy, PGT,SMC,Complications of laparoscopic cholecystectomy, by Dr. Bashab Bijoy Roy, PGT,SMC,
Complications of laparoscopic cholecystectomy, by Dr. Bashab Bijoy Roy, PGT,SMC,
 
Appendix
AppendixAppendix
Appendix
 
Ventral hernia management
Ventral hernia managementVentral hernia management
Ventral hernia management
 
Infantile hypertrophic pyloric stenosis
Infantile hypertrophic pyloric stenosisInfantile hypertrophic pyloric stenosis
Infantile hypertrophic pyloric stenosis
 
European hernia society guidelines: Adult Inguinal Hernia (Post operative car...
European hernia society guidelines: Adult Inguinal Hernia (Post operative car...European hernia society guidelines: Adult Inguinal Hernia (Post operative car...
European hernia society guidelines: Adult Inguinal Hernia (Post operative car...
 
Innovative Surgical Techiniques in Hepatobiliary and Pancreatic Surgery
Innovative Surgical Techiniques in Hepatobiliary and Pancreatic SurgeryInnovative Surgical Techiniques in Hepatobiliary and Pancreatic Surgery
Innovative Surgical Techiniques in Hepatobiliary and Pancreatic Surgery
 
Resection & anastomosis of boweL its complications PRANAYA PPT
Resection & anastomosis of boweL its complications PRANAYA PPTResection & anastomosis of boweL its complications PRANAYA PPT
Resection & anastomosis of boweL its complications PRANAYA PPT
 
Intra abdominal abscess
Intra abdominal abscessIntra abdominal abscess
Intra abdominal abscess
 

Viewers also liked

liver mass - how to investigate?
liver mass - how to investigate?liver mass - how to investigate?
liver mass - how to investigate?hr77
 
Investigations of lymphatics
Investigations of lymphaticsInvestigations of lymphatics
Investigations of lymphaticsKareem Hamimy
 
Mrcs Part A Experience
Mrcs Part A ExperienceMrcs Part A Experience
Mrcs Part A ExperienceKareem Hamimy
 
MRCS preparation emrcs questions Lowerlimb
MRCS preparation emrcs questions Lowerlimb MRCS preparation emrcs questions Lowerlimb
MRCS preparation emrcs questions Lowerlimb Faisol Kabir
 
Intercollegiate MRCS Examiners Newsletter Volume 2
Intercollegiate MRCS Examiners Newsletter Volume 2Intercollegiate MRCS Examiners Newsletter Volume 2
Intercollegiate MRCS Examiners Newsletter Volume 2meducationdotnet
 
MRCS preparation emrcs questions Pathology
MRCS preparation emrcs questions PathologyMRCS preparation emrcs questions Pathology
MRCS preparation emrcs questions PathologyFaisol Kabir
 
How to observe In Surgery
How to observe In SurgeryHow to observe In Surgery
How to observe In SurgeryKareem Hamimy
 
BPH and obstructive uropathy
BPH and obstructive uropathy BPH and obstructive uropathy
BPH and obstructive uropathy Ahmed Tawfeek
 
MRCS preparation emrcs questions upperlimb
MRCS preparation emrcs questions upperlimbMRCS preparation emrcs questions upperlimb
MRCS preparation emrcs questions upperlimbFaisol Kabir
 
Tips for orthopedics exam
Tips for orthopedics examTips for orthopedics exam
Tips for orthopedics examKareem Hamimy
 

Viewers also liked (20)

liver mass - how to investigate?
liver mass - how to investigate?liver mass - how to investigate?
liver mass - how to investigate?
 
Valgus vs. Varus
Valgus vs. VarusValgus vs. Varus
Valgus vs. Varus
 
Emergencies (non traumatic)
Emergencies (non traumatic)Emergencies (non traumatic)
Emergencies (non traumatic)
 
MRC case
MRC case MRC case
MRC case
 
New MRCS Part A Exam format
New MRCS Part A Exam formatNew MRCS Part A Exam format
New MRCS Part A Exam format
 
Investigations of lymphatics
Investigations of lymphaticsInvestigations of lymphatics
Investigations of lymphatics
 
MRCS Syllabus and recommended textbooks
MRCS Syllabus and recommended textbooksMRCS Syllabus and recommended textbooks
MRCS Syllabus and recommended textbooks
 
Access to urinary system v2
Access to urinary system v2Access to urinary system v2
Access to urinary system v2
 
Mrcs Part A Experience
Mrcs Part A ExperienceMrcs Part A Experience
Mrcs Part A Experience
 
Scrotal disorders
Scrotal disorders Scrotal disorders
Scrotal disorders
 
MRCS preparation emrcs questions Lowerlimb
MRCS preparation emrcs questions Lowerlimb MRCS preparation emrcs questions Lowerlimb
MRCS preparation emrcs questions Lowerlimb
 
Intercollegiate MRCS Examiners Newsletter Volume 2
Intercollegiate MRCS Examiners Newsletter Volume 2Intercollegiate MRCS Examiners Newsletter Volume 2
Intercollegiate MRCS Examiners Newsletter Volume 2
 
MRCS preparation emrcs questions Pathology
MRCS preparation emrcs questions PathologyMRCS preparation emrcs questions Pathology
MRCS preparation emrcs questions Pathology
 
How to observe In Surgery
How to observe In SurgeryHow to observe In Surgery
How to observe In Surgery
 
BPH and obstructive uropathy
BPH and obstructive uropathy BPH and obstructive uropathy
BPH and obstructive uropathy
 
MRC's Decision Support Framework
MRC's Decision Support FrameworkMRC's Decision Support Framework
MRC's Decision Support Framework
 
MRCS preparation emrcs questions upperlimb
MRCS preparation emrcs questions upperlimbMRCS preparation emrcs questions upperlimb
MRCS preparation emrcs questions upperlimb
 
Anuria & acute retention
Anuria & acute retentionAnuria & acute retention
Anuria & acute retention
 
Tips on using the MRC pre-ortho Trainer in myofunctional orthodontics
Tips on using the MRC pre-ortho Trainer in myofunctional orthodonticsTips on using the MRC pre-ortho Trainer in myofunctional orthodontics
Tips on using the MRC pre-ortho Trainer in myofunctional orthodontics
 
Tips for orthopedics exam
Tips for orthopedics examTips for orthopedics exam
Tips for orthopedics exam
 

Similar to MRCS preparation eMrcs questions surgery

SIGMOID VOLVULUS.pdf
SIGMOID VOLVULUS.pdfSIGMOID VOLVULUS.pdf
SIGMOID VOLVULUS.pdfShapi. MD
 
Basic of Small Animals Stomach Surgery
Basic of Small Animals Stomach SurgeryBasic of Small Animals Stomach Surgery
Basic of Small Animals Stomach SurgeryKambiz Yousefi
 
Intestinal Obstruction (1).ppt
Intestinal Obstruction (1).pptIntestinal Obstruction (1).ppt
Intestinal Obstruction (1).pptnagarajan740445
 
Pelvic gynecology intervention, complications and significance of teamwork co...
Pelvic gynecology intervention, complications and significance of teamwork co...Pelvic gynecology intervention, complications and significance of teamwork co...
Pelvic gynecology intervention, complications and significance of teamwork co...Rustem Celami
 
small intestine diseases 2
small intestine diseases 2small intestine diseases 2
small intestine diseases 2Deep Deep
 
Digestive System Procedures
Digestive System ProceduresDigestive System Procedures
Digestive System Procedurestkasprowicz
 
Fwd: Bambury tutorial Upper GI Surgery
Fwd: Bambury tutorial Upper GI SurgeryFwd: Bambury tutorial Upper GI Surgery
Fwd: Bambury tutorial Upper GI SurgeryJeku Jacob
 
Intestinal obstruction by Dr.Usman Haqqani
Intestinal obstruction by Dr.Usman HaqqaniIntestinal obstruction by Dr.Usman Haqqani
Intestinal obstruction by Dr.Usman HaqqaniUsman Haqqani
 
Bowel obstruction
Bowel obstruction Bowel obstruction
Bowel obstruction Srini Vasan
 
INTESTINE OBSTRUCTION.pptx
INTESTINE OBSTRUCTION.pptxINTESTINE OBSTRUCTION.pptx
INTESTINE OBSTRUCTION.pptxamanjotkaursidhu
 
GASTRIC PERFORATION general surgery.pptx
GASTRIC PERFORATION general surgery.pptxGASTRIC PERFORATION general surgery.pptx
GASTRIC PERFORATION general surgery.pptxCivil Hospital, Aizawl.
 
Enterocutaneous fistula
Enterocutaneous fistulaEnterocutaneous fistula
Enterocutaneous fistulaFidelSimba
 
Complications of-peptic-ulcer
Complications of-peptic-ulcerComplications of-peptic-ulcer
Complications of-peptic-ulcersamemeskey
 
Combined 06 clinical training--pathology benign_volvulus
Combined 06 clinical training--pathology benign_volvulusCombined 06 clinical training--pathology benign_volvulus
Combined 06 clinical training--pathology benign_volvulusIknifem
 

Similar to MRCS preparation eMrcs questions surgery (20)

Volvulus of colon
Volvulus of colonVolvulus of colon
Volvulus of colon
 
SIGMOID VOLVULUS.pdf
SIGMOID VOLVULUS.pdfSIGMOID VOLVULUS.pdf
SIGMOID VOLVULUS.pdf
 
Basic of Small Animals Stomach Surgery
Basic of Small Animals Stomach SurgeryBasic of Small Animals Stomach Surgery
Basic of Small Animals Stomach Surgery
 
Meckel’s diverticulum
Meckel’s diverticulumMeckel’s diverticulum
Meckel’s diverticulum
 
Intestinal Obstruction (1).ppt
Intestinal Obstruction (1).pptIntestinal Obstruction (1).ppt
Intestinal Obstruction (1).ppt
 
Pelvic gynecology intervention, complications and significance of teamwork co...
Pelvic gynecology intervention, complications and significance of teamwork co...Pelvic gynecology intervention, complications and significance of teamwork co...
Pelvic gynecology intervention, complications and significance of teamwork co...
 
small intestine diseases 2
small intestine diseases 2small intestine diseases 2
small intestine diseases 2
 
abdominal wall defect
abdominal wall defectabdominal wall defect
abdominal wall defect
 
Digestive System Procedures
Digestive System ProceduresDigestive System Procedures
Digestive System Procedures
 
Intestinal obstruction
Intestinal obstructionIntestinal obstruction
Intestinal obstruction
 
Ventral Hernias.pptx
Ventral Hernias.pptxVentral Hernias.pptx
Ventral Hernias.pptx
 
Fwd: Bambury tutorial Upper GI Surgery
Fwd: Bambury tutorial Upper GI SurgeryFwd: Bambury tutorial Upper GI Surgery
Fwd: Bambury tutorial Upper GI Surgery
 
Intestinal obstruction by Dr.Usman Haqqani
Intestinal obstruction by Dr.Usman HaqqaniIntestinal obstruction by Dr.Usman Haqqani
Intestinal obstruction by Dr.Usman Haqqani
 
Bowel obstruction
Bowel obstruction Bowel obstruction
Bowel obstruction
 
INTESTINE OBSTRUCTION.pptx
INTESTINE OBSTRUCTION.pptxINTESTINE OBSTRUCTION.pptx
INTESTINE OBSTRUCTION.pptx
 
GASTRIC PERFORATION general surgery.pptx
GASTRIC PERFORATION general surgery.pptxGASTRIC PERFORATION general surgery.pptx
GASTRIC PERFORATION general surgery.pptx
 
Enterocutaneous fistula
Enterocutaneous fistulaEnterocutaneous fistula
Enterocutaneous fistula
 
Complications of-peptic-ulcer
Complications of-peptic-ulcerComplications of-peptic-ulcer
Complications of-peptic-ulcer
 
Acquired intestinal ileus
Acquired intestinal ileusAcquired intestinal ileus
Acquired intestinal ileus
 
Combined 06 clinical training--pathology benign_volvulus
Combined 06 clinical training--pathology benign_volvulusCombined 06 clinical training--pathology benign_volvulus
Combined 06 clinical training--pathology benign_volvulus
 

Recently uploaded

Call Girls Viman Nagar 7001305949 All Area Service COD available Any Time
Call Girls Viman Nagar 7001305949 All Area Service COD available Any TimeCall Girls Viman Nagar 7001305949 All Area Service COD available Any Time
Call Girls Viman Nagar 7001305949 All Area Service COD available Any Timevijaych2041
 
Call Girls Frazer Town Just Call 7001305949 Top Class Call Girl Service Avail...
Call Girls Frazer Town Just Call 7001305949 Top Class Call Girl Service Avail...Call Girls Frazer Town Just Call 7001305949 Top Class Call Girl Service Avail...
Call Girls Frazer Town Just Call 7001305949 Top Class Call Girl Service Avail...narwatsonia7
 
Call Girl Service Bidadi - For 7001305949 Cheap & Best with original Photos
Call Girl Service Bidadi - For 7001305949 Cheap & Best with original PhotosCall Girl Service Bidadi - For 7001305949 Cheap & Best with original Photos
Call Girl Service Bidadi - For 7001305949 Cheap & Best with original Photosnarwatsonia7
 
Call Girls Service Nandiambakkam | 7001305949 At Low Cost Cash Payment Booking
Call Girls Service Nandiambakkam | 7001305949 At Low Cost Cash Payment BookingCall Girls Service Nandiambakkam | 7001305949 At Low Cost Cash Payment Booking
Call Girls Service Nandiambakkam | 7001305949 At Low Cost Cash Payment BookingNehru place Escorts
 
VIP Call Girls Mumbai Arpita 9910780858 Independent Escort Service Mumbai
VIP Call Girls Mumbai Arpita 9910780858 Independent Escort Service MumbaiVIP Call Girls Mumbai Arpita 9910780858 Independent Escort Service Mumbai
VIP Call Girls Mumbai Arpita 9910780858 Independent Escort Service Mumbaisonalikaur4
 
Call Girls Jayanagar Just Call 7001305949 Top Class Call Girl Service Available
Call Girls Jayanagar Just Call 7001305949 Top Class Call Girl Service AvailableCall Girls Jayanagar Just Call 7001305949 Top Class Call Girl Service Available
Call Girls Jayanagar Just Call 7001305949 Top Class Call Girl Service Availablenarwatsonia7
 
Russian Call Girls Gunjur Mugalur Road : 7001305949 High Profile Model Escort...
Russian Call Girls Gunjur Mugalur Road : 7001305949 High Profile Model Escort...Russian Call Girls Gunjur Mugalur Road : 7001305949 High Profile Model Escort...
Russian Call Girls Gunjur Mugalur Road : 7001305949 High Profile Model Escort...narwatsonia7
 
Book Call Girls in Yelahanka - For 7001305949 Cheap & Best with original Photos
Book Call Girls in Yelahanka - For 7001305949 Cheap & Best with original PhotosBook Call Girls in Yelahanka - For 7001305949 Cheap & Best with original Photos
Book Call Girls in Yelahanka - For 7001305949 Cheap & Best with original Photosnarwatsonia7
 
Call Girls In Andheri East Call 9920874524 Book Hot And Sexy Girls
Call Girls In Andheri East Call 9920874524 Book Hot And Sexy GirlsCall Girls In Andheri East Call 9920874524 Book Hot And Sexy Girls
Call Girls In Andheri East Call 9920874524 Book Hot And Sexy Girlsnehamumbai
 
call girls in munirka DELHI 🔝 >༒9540349809 🔝 genuine Escort Service 🔝✔️✔️
call girls in munirka  DELHI 🔝 >༒9540349809 🔝 genuine Escort Service 🔝✔️✔️call girls in munirka  DELHI 🔝 >༒9540349809 🔝 genuine Escort Service 🔝✔️✔️
call girls in munirka DELHI 🔝 >༒9540349809 🔝 genuine Escort Service 🔝✔️✔️saminamagar
 
Call Girls Hosur Just Call 7001305949 Top Class Call Girl Service Available
Call Girls Hosur Just Call 7001305949 Top Class Call Girl Service AvailableCall Girls Hosur Just Call 7001305949 Top Class Call Girl Service Available
Call Girls Hosur Just Call 7001305949 Top Class Call Girl Service Availablenarwatsonia7
 
Asthma Review - GINA guidelines summary 2024
Asthma Review - GINA guidelines summary 2024Asthma Review - GINA guidelines summary 2024
Asthma Review - GINA guidelines summary 2024Gabriel Guevara MD
 
Call Girl Nagpur Sia 7001305949 Independent Escort Service Nagpur
Call Girl Nagpur Sia 7001305949 Independent Escort Service NagpurCall Girl Nagpur Sia 7001305949 Independent Escort Service Nagpur
Call Girl Nagpur Sia 7001305949 Independent Escort Service NagpurRiya Pathan
 
97111 47426 Call Girls In Delhi MUNIRKAA
97111 47426 Call Girls In Delhi MUNIRKAA97111 47426 Call Girls In Delhi MUNIRKAA
97111 47426 Call Girls In Delhi MUNIRKAAjennyeacort
 
Call Girls Service in Bommanahalli - 7001305949 with real photos and phone nu...
Call Girls Service in Bommanahalli - 7001305949 with real photos and phone nu...Call Girls Service in Bommanahalli - 7001305949 with real photos and phone nu...
Call Girls Service in Bommanahalli - 7001305949 with real photos and phone nu...narwatsonia7
 
VIP Call Girls Lucknow Nandini 7001305949 Independent Escort Service Lucknow
VIP Call Girls Lucknow Nandini 7001305949 Independent Escort Service LucknowVIP Call Girls Lucknow Nandini 7001305949 Independent Escort Service Lucknow
VIP Call Girls Lucknow Nandini 7001305949 Independent Escort Service Lucknownarwatsonia7
 
Glomerular Filtration rate and its determinants.pptx
Glomerular Filtration rate and its determinants.pptxGlomerular Filtration rate and its determinants.pptx
Glomerular Filtration rate and its determinants.pptxDr.Nusrat Tariq
 
Call Girls Electronic City Just Call 7001305949 Top Class Call Girl Service A...
Call Girls Electronic City Just Call 7001305949 Top Class Call Girl Service A...Call Girls Electronic City Just Call 7001305949 Top Class Call Girl Service A...
Call Girls Electronic City Just Call 7001305949 Top Class Call Girl Service A...narwatsonia7
 
Call Girls Jp Nagar Just Call 7001305949 Top Class Call Girl Service Available
Call Girls Jp Nagar Just Call 7001305949 Top Class Call Girl Service AvailableCall Girls Jp Nagar Just Call 7001305949 Top Class Call Girl Service Available
Call Girls Jp Nagar Just Call 7001305949 Top Class Call Girl Service Availablenarwatsonia7
 
High Profile Call Girls Jaipur Vani 8445551418 Independent Escort Service Jaipur
High Profile Call Girls Jaipur Vani 8445551418 Independent Escort Service JaipurHigh Profile Call Girls Jaipur Vani 8445551418 Independent Escort Service Jaipur
High Profile Call Girls Jaipur Vani 8445551418 Independent Escort Service Jaipurparulsinha
 

Recently uploaded (20)

Call Girls Viman Nagar 7001305949 All Area Service COD available Any Time
Call Girls Viman Nagar 7001305949 All Area Service COD available Any TimeCall Girls Viman Nagar 7001305949 All Area Service COD available Any Time
Call Girls Viman Nagar 7001305949 All Area Service COD available Any Time
 
Call Girls Frazer Town Just Call 7001305949 Top Class Call Girl Service Avail...
Call Girls Frazer Town Just Call 7001305949 Top Class Call Girl Service Avail...Call Girls Frazer Town Just Call 7001305949 Top Class Call Girl Service Avail...
Call Girls Frazer Town Just Call 7001305949 Top Class Call Girl Service Avail...
 
Call Girl Service Bidadi - For 7001305949 Cheap & Best with original Photos
Call Girl Service Bidadi - For 7001305949 Cheap & Best with original PhotosCall Girl Service Bidadi - For 7001305949 Cheap & Best with original Photos
Call Girl Service Bidadi - For 7001305949 Cheap & Best with original Photos
 
Call Girls Service Nandiambakkam | 7001305949 At Low Cost Cash Payment Booking
Call Girls Service Nandiambakkam | 7001305949 At Low Cost Cash Payment BookingCall Girls Service Nandiambakkam | 7001305949 At Low Cost Cash Payment Booking
Call Girls Service Nandiambakkam | 7001305949 At Low Cost Cash Payment Booking
 
VIP Call Girls Mumbai Arpita 9910780858 Independent Escort Service Mumbai
VIP Call Girls Mumbai Arpita 9910780858 Independent Escort Service MumbaiVIP Call Girls Mumbai Arpita 9910780858 Independent Escort Service Mumbai
VIP Call Girls Mumbai Arpita 9910780858 Independent Escort Service Mumbai
 
Call Girls Jayanagar Just Call 7001305949 Top Class Call Girl Service Available
Call Girls Jayanagar Just Call 7001305949 Top Class Call Girl Service AvailableCall Girls Jayanagar Just Call 7001305949 Top Class Call Girl Service Available
Call Girls Jayanagar Just Call 7001305949 Top Class Call Girl Service Available
 
Russian Call Girls Gunjur Mugalur Road : 7001305949 High Profile Model Escort...
Russian Call Girls Gunjur Mugalur Road : 7001305949 High Profile Model Escort...Russian Call Girls Gunjur Mugalur Road : 7001305949 High Profile Model Escort...
Russian Call Girls Gunjur Mugalur Road : 7001305949 High Profile Model Escort...
 
Book Call Girls in Yelahanka - For 7001305949 Cheap & Best with original Photos
Book Call Girls in Yelahanka - For 7001305949 Cheap & Best with original PhotosBook Call Girls in Yelahanka - For 7001305949 Cheap & Best with original Photos
Book Call Girls in Yelahanka - For 7001305949 Cheap & Best with original Photos
 
Call Girls In Andheri East Call 9920874524 Book Hot And Sexy Girls
Call Girls In Andheri East Call 9920874524 Book Hot And Sexy GirlsCall Girls In Andheri East Call 9920874524 Book Hot And Sexy Girls
Call Girls In Andheri East Call 9920874524 Book Hot And Sexy Girls
 
call girls in munirka DELHI 🔝 >༒9540349809 🔝 genuine Escort Service 🔝✔️✔️
call girls in munirka  DELHI 🔝 >༒9540349809 🔝 genuine Escort Service 🔝✔️✔️call girls in munirka  DELHI 🔝 >༒9540349809 🔝 genuine Escort Service 🔝✔️✔️
call girls in munirka DELHI 🔝 >༒9540349809 🔝 genuine Escort Service 🔝✔️✔️
 
Call Girls Hosur Just Call 7001305949 Top Class Call Girl Service Available
Call Girls Hosur Just Call 7001305949 Top Class Call Girl Service AvailableCall Girls Hosur Just Call 7001305949 Top Class Call Girl Service Available
Call Girls Hosur Just Call 7001305949 Top Class Call Girl Service Available
 
Asthma Review - GINA guidelines summary 2024
Asthma Review - GINA guidelines summary 2024Asthma Review - GINA guidelines summary 2024
Asthma Review - GINA guidelines summary 2024
 
Call Girl Nagpur Sia 7001305949 Independent Escort Service Nagpur
Call Girl Nagpur Sia 7001305949 Independent Escort Service NagpurCall Girl Nagpur Sia 7001305949 Independent Escort Service Nagpur
Call Girl Nagpur Sia 7001305949 Independent Escort Service Nagpur
 
97111 47426 Call Girls In Delhi MUNIRKAA
97111 47426 Call Girls In Delhi MUNIRKAA97111 47426 Call Girls In Delhi MUNIRKAA
97111 47426 Call Girls In Delhi MUNIRKAA
 
Call Girls Service in Bommanahalli - 7001305949 with real photos and phone nu...
Call Girls Service in Bommanahalli - 7001305949 with real photos and phone nu...Call Girls Service in Bommanahalli - 7001305949 with real photos and phone nu...
Call Girls Service in Bommanahalli - 7001305949 with real photos and phone nu...
 
VIP Call Girls Lucknow Nandini 7001305949 Independent Escort Service Lucknow
VIP Call Girls Lucknow Nandini 7001305949 Independent Escort Service LucknowVIP Call Girls Lucknow Nandini 7001305949 Independent Escort Service Lucknow
VIP Call Girls Lucknow Nandini 7001305949 Independent Escort Service Lucknow
 
Glomerular Filtration rate and its determinants.pptx
Glomerular Filtration rate and its determinants.pptxGlomerular Filtration rate and its determinants.pptx
Glomerular Filtration rate and its determinants.pptx
 
Call Girls Electronic City Just Call 7001305949 Top Class Call Girl Service A...
Call Girls Electronic City Just Call 7001305949 Top Class Call Girl Service A...Call Girls Electronic City Just Call 7001305949 Top Class Call Girl Service A...
Call Girls Electronic City Just Call 7001305949 Top Class Call Girl Service A...
 
Call Girls Jp Nagar Just Call 7001305949 Top Class Call Girl Service Available
Call Girls Jp Nagar Just Call 7001305949 Top Class Call Girl Service AvailableCall Girls Jp Nagar Just Call 7001305949 Top Class Call Girl Service Available
Call Girls Jp Nagar Just Call 7001305949 Top Class Call Girl Service Available
 
High Profile Call Girls Jaipur Vani 8445551418 Independent Escort Service Jaipur
High Profile Call Girls Jaipur Vani 8445551418 Independent Escort Service JaipurHigh Profile Call Girls Jaipur Vani 8445551418 Independent Escort Service Jaipur
High Profile Call Girls Jaipur Vani 8445551418 Independent Escort Service Jaipur
 

MRCS preparation eMrcs questions surgery

  • 1. A 53 year old man undergoes a reversal of a loop colostomy. He recovers well and is discharged home. He is readmitted 10 days later with symptoms of vomiting and colicky abdominal pain. On examination he has a swelling of the loop colostomy site and it is tender. What is the most likely underlying diagnosis? A. Haematoma B. Intra abdominal adhesions C. Anastomotic leak D. Anastomotic stricture E. Obstructed incisional hernia Theme from September 2011 Exam In this scenario the most likely diagnosis would be obstructed incisional hernia. The tender swelling coupled with symptoms of obstruction point to this diagnosis. Prompt surgical exploration is warranted. Loop colostomy reversals are at high risk of this complication as the operative site is at increased risk of the development of post operative wound infections. Acute incisional hernia  Any surgical procedure involving entry into a cavity containing viscera may be complicated by post operative hernia  The abdomen is the commonest site  The deep layer of the wound has usually broken down, allowing internal viscera to protrude through  Management is dictated by the patients clinical status and the timing of the hernia in relation to recent surgery  Bowel obstruction or tenderness at the hernia site both mandate early surgical intervention to reduce the risk of bowel necrosis  Mature incisional hernias with a wide neck and no symptoms may be either left or listed for elective repair  Risk factors for the development of post operative incisional hernias include post operative wound infections, long term steroid use, obesity and chronic cough Theme: Abdominal stomas A. End ileostomy B. End colostomy C. Loop ileostomy D. Loop colostomy
  • 2. E. End jejunostomy F. Loop jejunostomy G. Caecostomy For each of the following scenarios, please select the most appropriate type of stoma to be constructed. Each option may be selected once, more than once or not at all. 2. A 56 year old man is undergoing a low anterior resection for carcinoma of the rectum. It is planned to restore intestinal continuity. You answered End colostomy The correct answer is Loop ileostomy Colonic resections with an anastomosis below the peritoneal reflection may have an anastomotic leak rate (both clinical and radiological) of up to 15%. Therefore most surgeons will defunction such an anastomosis to reduce the clinical severity of an anastomotic leak. A loop ileostomy will achieve this end point and is relatively easy to reverse. 3. A 23 year old man with uncontrolled ulcerative colitis is undergoing an emergency sub total colectomy. You answered Loop ileostomy The correct answer is End ileostomy Following a sub total colectomy the immediate surgical options include an end ileostomy or ileorectal anastomosis. In the emergency setting an ileorectal anastomosis would be unsafe. 4. A 63 year old women presents with large bowel obstruction. On examination she has a carcinoma 10cm from the anal verge. You answered End colostomy The correct answer is Loop colostomy Large bowel obstruction resulting from carcinoma should be resected, stented or defunctioned. The first two options typically apply to tumours above the peritoneal reflection. Lower tumours should be defunctioned with a loop colostomy and then formal staging undertaken prior to definitive surgery. An emergency attempted rectal resection carries a high risk of involvement of the circumferential resection margin and is not recommended.
  • 3. Abdominal stomas Stomas may be sited during a range of abdominal procedures and involve bringing the lumen or visceral contents onto the skin. In most cases this applies to the bowel. However, other organs or their contents may be diverted in case of need. With bowel stomas the type method of construction and to a lesser extent the site will be determined by the contents of the bowel. In practice, small bowel stomas should be spouted so that their irritant contents are not in contact with the skin. Colonic stomas do not need to be spouted as their contents are less irritant. In the ideal situation the site of the stoma should be marked with the patient prior to surgery. Stoma siting is important as it will ultimately influence the ability of the patient to manage their stoma and also reduce the risk of leakage. Leakage of stoma contents and subsequent maceration of the surrounding skin can rapidly progress into a spiraling loss of control of stoma contents. Types of stomas Name of stoma Use Common sites Gastrostomy  Gastric decompression or fixation  Feeding Epigastrium Loop jejunostomy  Seldom used as very high output  May be used following emergency laparotomy with planned early closure Any location according to need Percutaneous jejunostomy  Usually performed for feeding purposes and site in the proximal bowel Usually left upper quadrant Loop ileostomy  Defunctioning of colon e.g. following rectal cancer surgery  Does not decompress colon (if ileocaecal valve competent) Usually right iliac fossa End ilestomy  Usually following complete excision of colon or where ileo- colic anastomosis is not planned  May be used to defunction colon, but reversal is more difficult Usually right iliac fossa End colostomy Where a colon is diverted or resected and anastomosis is not primarily achievable or desirable Either left or right iliac fossa
  • 4. Loop colostomy  To defunction a distal segment of colon  Since both lumens are present the distal lumen acts as a vent May be located in any region of the abdomen, depending upon colonic segment used Caecostomy Stoma of last resort where loop colostomy is not possible Right iliac fossa Mucous fistula  To decompress a distal segment of bowel following colonic division or resection  Where closure of a distal resection margin is not safe or achievable May be located in any region of the abdomen according to clinical need Theme: Acute abdominal pain A. Ruptured abdominal aortic aneurysm B. Perforated peptic ulcer C. Perforated appendicitis D. Mesenteric infarction E. Small bowel obstruction F. Large bowel obstruction G. Pelvic inflammatory disease H. Mesenteric adenitis I. Pancreatitis J. None of the above Please select the most likely cause of abdominal pain for the scenario given. Each option may be used once, more than once or not at all. 5. A 75 year old man is admitted with sudden onset severe generalised abdominal pain, vomiting and a single episode of bloody diarrhoea. On examination he looks unwell and is in uncontrolled atrial fibrillation. Although diffusely tender his abdomen is soft. Mesenteric infarction In mesenteric infarction there is sudden onset of pain together with vomiting and occasionally passage of bloody diarrhoea. The pain present is usually out of proportion to the physical signs. 6. A 19 year old lady is admitted with lower abdominal pain. On examination she is diffusely tender. A laparoscopy is performed and at operation multiple fine
  • 5. adhesions are noted between the liver and abdominal wall. Her appendix is normal. You answered Small bowel obstruction The correct answer is Pelvic inflammatory disease This is Fitz Hugh Curtis syndrome in which pelvic inflammatory disease (usually Chlamydia) causes the formation of fine peri hepatic adhesions. 7. A 78 year old man is walking to the bus stop when he suddenly develops severe back pain and collapses. On examination he has a blood pressure of 90/40 and pulse rate of 110. His abdomen is distended and he is obese. Though tender his abdomen itself is soft. Ruptured abdominal aortic aneurysm This will be a retroperitoneal rupture (anterior ones generally don't survive to hospital). The debate regarding CT varies, it is the authors opinion that a systolic BP of <100mmHg at presentation mandates immediate laparotomy. Acute mesenteric ischaemia- Pain out of proportion to the physical signs. Atrial fibrillation is often present. Fitz Hugh Curtis = Fine Hepatic Connections Acute abdominal pain-diagnoses Conditions presenting with acute abdominal pain Condition Features Investigations Management Appendicitis History of migratory pain. Fever. Anorexia. Evidence of right iliac fossa tenderness. Mild pyrexia. Differential white cell count Pregnancy test C-Reactive protein Amylase Urine dipstick testing Appendicectomy Mesenteric adenitis Usually recent upper respiratory tract infection. High fever. Generalised Full blood count- may show slightly raised white cell count Urine dipstick often normal Conservative management- appendicectomy if diagnostic doubt
  • 6. abdominal discomfort- true localised pain and signs are rare. Abdominal ultrasound scan - usually no free fluid Mittelschmerz Only seen in females. Mid cycle pain. Usually occurs two weeks after last menstrual period. Pain is usually has a supra-pubic location. Usually subsides over a 24-48 hour period. Full blood count- normal Urine dipstick- normal Abdominal and pelvic ultrasound- may show a trace of pelvic free fluid Manage conservatively if doubt or symptoms fail to settle then laparoscopy Fitz-Hugh Curtis syndrome Disseminated infection with Chlamydia. Usually seen in females. Consists of evidence of pelvic inflammatory disease together with peri-hepatic inflammation and subsequent adhesion formation. Abdominal ultrasound scan- may show free fluid High vaginal swabs - may show evidence of sexually transmitted infections Usually medically managed- doxycycline or azithromycin Abdominal aortic aneurysm (ruptured) Sudden onset of abdominal pain radiating to the back in older adults (look for risk factors). Collapse. May be moribund on arrival in casualty, more stable if contained haematoma. Careful clinical assessment may reveal pulsatile mass. Patients who are haemodynamically stable should have a CT scan Unstable patients should undergo immediate surgery (unless it is not in their best interests). Those with evidence of contained leak on CT should undergo immediate surgery Increasing aneurysmal size is an indication for urgent surgical intervention (that can wait until the next working day) Perforated peptic ulcer Sudden onset of pain (usually epigastric). Often preceding Erect CXR may show free air. A CT scan may be indicated where there is diagnostic doubt Laparotomy (laparoscopic surgery for perforated peptic ulcers is both safe and
  • 7. history of upper abdominal pain. Soon develop generalised abdominal pain. On examination may have clinical evidence of peritonitis. feasible in experienced hands) Intestinal obstruction Colicky abdominal pain and vomiting (the nature of which depends on the level of the obstruction). Abdominal distension and constipation (again depending upon site of obstruction). Features of peritonism may occur where local necrosis of bowel loops is occurring. A plain abdominal film may help with making the diagnosis. A CT scan may be useful where diagnostic uncertainty exists In those with a virgin abdomen and lower and earlier threshold for laparotomy should exist than in those who may have adhesional obstruction Mesenteric infarction Embolic events present with sudden pain and forceful evacuation. Acute on chronic events usually have a longer history and previous weight loss. On examination the pain is typically greater than the physical signs would suggest. Arterial pH and lactate Arterial phase CT scanning is the most sensitive test Immediate laparotomy and resection of affected segments, in acute embolic events SMA embolectomy may be needed. Theme: Gastrointestinal bleeding A. Haemorroids B. Meckels diverticulum C. Angiodysplasia D. Colonic cancer E. Diverticular bleed F. Ulcerative colitis
  • 8. G. Ischaemic colitis Please select the most likely cause of colonic bleeding for the scenario given. Each option may be used once, more than once or not at all 8. A 73 year old lady is admitted with a brisk rectal bleed. She is otherwise well and the bleed settles. On examination her abdomen is soft and non tender. Elective colonoscopy shows a small erythematous lesion in the right colon, but no other abnormality. Angiodysplasia Angiodysplasia can be difficult to identify and treat. The colonoscopic stigmata are easily missed by poor bowel preparation. 9. A 23 year old man complains of passing bright red blood rectally. It has been occurring over the past week and tends to occur post defecation. He also suffers from pruritus ani. Haemorroids Classical haemorroidal symptoms include bright red rectal bleeding, it typically occurs post defecation and is noticed on the toilet paper and in the toilet pan. It is usually painless, however, thrombosed external haemorroids may be very painful. 10. A 63 year old man presents with episodic rectal bleeding the blood tends to be dark in colour and may be mixed with stool. His bowel habit has been erratic since an abdominal aortic aneurysm repair 6 weeks previously. Ischaemic colitis The inferior mesenteric artery may have been ligated and being an arteriopath collateral flow through the marginal may be imperfect. Gastrointestinal bleeding Colonic bleeding This typically presents as bright red or dark red blood per rectum. Colonic bleeding rarely presents as malaena type stool, this is because blood in the colon has a powerful laxative effect and is rarely retained long enough for transformation to occur and because the digestive enzymes present in the small bowel are not present in the colon.
  • 9. Up to 15% of patients presenting with haemochezia will have an upper gastrointestinal source of haemorrhage. As a general rule right sided bleeds tend to present with darker coloured blood than left sided bleeds. Haemorrhoidal bleeding typically presents as bright red rectal bleeding that occurs post defecation either onto toilet paper or into the toilet pan. It is very unusual for haemorrhoids alone to cause any degree of haemodynamic compromise. Causes Cause Presenting features Colitis Bleeding may be brisk in advanced cases, diarrhoea is commonly present. Abdominal x-ray may show featureless colon. Diverticular disease Acute diverticulitis often is not complicated by major bleeding and diverticular bleeds often occur sporadically. 75% all will cease spontaneously within 24-48 hours. Bleeding is often dark and of large volume. Cancer Colonic cancers often bleed and for many patients this may be the first sign of the disease. Major bleeding from early lesions is uncommon Haemorrhoidal bleeding Typically bright red bleeding occurring post defecation. Although patients may give graphic descriptions bleeding of sufficient volume to cause haemodynamic compromise is rare. Angiodysplasia Apart from bleeding, which may be massive, these arteriovenous lesions cause little in the way of symptoms. The right side of the colon is more commonly affected. Management  Prompt correction of any haemodynamic compromise is required. Unlike upper gastrointestinal bleeding the first line management is usually supportive. This is because in the acute setting endoscopy is rarely helpful.  When haemorrhoidal bleeding is suspected a proctosigmoidoscopy is reasonable as attempts at full colonoscopy are usually time consuming and often futile.  In the unstable patient the usual procedure would be an angiogram (either CT or percutaneous), when these are performed during a period of haemodynamic instability they may show a bleeding point and may be the only way of identifying a patch of angiodysplasia.  In others who are more stable the standard procedure would be a colonoscopy in the elective setting. In patients undergoing angiography attempts can be made to address the lesion in question such as coiling. Otherwise surgery will be necessary.  In patients with ulcerative colitis who have significant haemorrhage the standard approach would be a sub total colectomy, particularly if medical management has already been tried and is not effective.
  • 10. Indications for surgery Patients > 60 years Continued bleeding despite endoscopic intervention Recurrent bleeding Known cardiovascular disease with poor response to hypotension Surgery Selective mesenteric embolisation if life threatening bleeding. This is most helpful if conducted during a period of relative haemodynamic instability. If all haemodynamic parameters are normal then the bleeding is most likely to have stopped and any angiography normal in appearance. In many units a CT angiogram will replace selective angiography but the same caveats will apply. If source of colonic bleeding unclear perform a laparotomy, on table colonic lavage and following this attempt a resection. A blind sub total colectomy is most unwise, for example bleeding from an small bowel arterio-venous malformation will not be treated by this manoeuvre. Summary of Acute Lower GI bleeding recommendations Consider admission if: * Over 60 years * Haemodynamically unstable/profuse PR bleeding * On aspirin or NSAID * Significant co morbidity Management  All patients should have a history and examination, PR and proctoscopy  Colonoscopic haemostasis aimed for in post polypectomy or diverticular bleeding References http://www.sign.ac.uk/guidelines/fulltext/105/index.html heme: Surgical signs A. Rovsing's sign B. Boas' sign C. Psoas stretch sign D. Cullen's sign E. Grey-Turner's sign F. Murphy's sign G. None of the above Please select the most appropriate eponymous abdominal sign for the scenario given. Each option may be used once, more than once or not at all.
  • 11. 11. Severe acute peri-umbilical bruising in the setting of acute pancreatitis. Cullen's sign Cullens sign occurs when there has been intraabdominal haemorrage. It is seen in cases of severe haemorrhagic pancreatitis and is associated with a poor prognosis. It is also seen in other cases of intraabdominal haemorrhage (such as ruptured ectopic pregnancy). 12. In acute cholecystitis there is hyperaesthesia beneath the right scapula. Boas' sign Boas sign refers to this hyperaesthesia. It occurs because the abdominal wall innervation of this region is from the spinal roots that lie at this level. 13. In appendicitis palpation of the left iliac fossa causes pain in the right iliac fossa. Rovsing's sign Rovsings sign elicits tenderness because the deep palpation induces shift of the appendix (which is inflamed) against the peritoneal surface. This has somatic innervation and will therefore localise the pain. It is less reliable in pelvic appendicitis and when the appendix is truly retrocaecal Abdominal signs A number of eponymous abdominal signs are noted. These include:  Rovsings sign- appendicitis  Boas sign -cholecystitis  Murphys sign- cholecystitis  Cullens sign- pancreatitis (other intraabdominal haemorrhage)  Grey-Turners sign- pancreatitis (or other retroperitoneal haemorrhage) In clinical practice haemorrhagic pancreatitis is thankfully rare. The signs are important and thus shown below: Cullen's sign
  • 12. Image sourced from Wikipedia Grey Turner's sign Image sourced from Wikipedia Theme: Surgical access A. Gridiron B. Lanz C. McEvedy D. Midline abdominal E. Rutherford Morrison F. Battle (abdominal) G. Lower midline Please select the most appropriate incision for the procedure required. Each option may be used once, more than once or not at all. 14. A 78 year old lady is admitted with a tender lump in her right groin. It is within the femoral triangle and there is concern that there may be small bowel obstruction developing.
  • 13. McEvedy This is one approach to an obstructed femoral hernia. It is possible to undertake a small bowel resection through this approach. Although recourse to laparotomy may be needed if access is difficult. 15. A 45 year old woman with end stage renal failure is due to undergo a cadaveric renal transplant. This will be her first transplant. Rutherford Morrison This is the incision of choice for the extraperitoneal approach to the iliac vessels which will be required for a renal transplant. 16. A slim 20 year old lady is suffering from appendicitis and requires an appendicectomy. Lanz Either a Lanz or Gridiron incision will give access for appendicectomy. However, in the case described a Lanz incision will give better cosmesis and can be extended should pelvic surgery be required eg for gynaecological disease. Abdominal incisions Theme in January 2012 exam Midline incision  Commonest approach to the abdomen  Structures divided: linea alba, transversalis fascia, extraperitoneal fat, peritoneum (avoid falciform ligament above the umbilicus)  Bladder can be accessed via an extraperitoneal approach through the space of Retzius Paramedian incision  Parallel to the midline (about 3-4cm)  Structures divided/retracted: anterior rectus sheath, rectus (retracted), posterior rectus sheath, transversalis fascia, extraperitoneal fat, peritoneum  Incision is closed in layers
  • 14. Battle  Similar location to paramedian but rectus displaced medially (and thus denervated)  Now seldom used Kocher's Incision under right subcostal margin e.g. Cholecystectomy (open) Lanz Incision in right iliac fossa e.g. Appendicectomy Gridiron Oblique incision centered over McBurneys point- usually appendicectomy (less cosmetically acceptable than Lanz Gable Rooftop incision Pfannenstiel's Transverse supra pubic, primarily used to access pelvic organs McEvedy's Groin incision e.g. Emergency repair strangulated femoral hernia Rutherford Morrison Extraperitoneal approach to left or right lower quadrants. Gives excellent access to iliac vessels and is the approach of choice for first time renal transplantation. Image sourced from Wikipedia Theme: Hernias A. Littres hernia B. Richters hernia C. Bochdalek hernia D. Morgagni hernia E. Spigelian hernia F. Lumbar hernia G. Obturator hernia Please select the type of hernia that most closely matches the description given. Each option may be used once, more than once or not at all.
  • 15. 17. A 73 year old lady presents with peritonitis and tenderness of the left groin. At operation she has a left femoral hernia with perforation of the anti mesenteric border of ileum associated with the hernia. Richters hernia When part of the bowel wall is trapped in a hernia such as this it is termed a Richters hernia and may complicate any hernia although femoral and obturator hernias are most typically implicated. 18. A 22 year old man is operated on for a left inguinal hernia, at operation the sac is opened to reveal a large Meckels diverticulum. Littres hernia Hernia containing Meckels diverticulum is termed a Littres hernia. 19. A 45 year old man has recurrent colicky abdominal pain. As part of a series of investigations he undergoes a CT scan and this demonstrates a hernia lateral to the rectus muscle at the level of the arcuate line. Spigelian hernia This is the site for a spigelian hernia. Theme: Groin masses A. Femoral hernia B. Lymphadenitis C. Inguinal hernia D. Psoas abscess E. Saphenous varix F. Femoral artery aneurysm G. Metastatic lymphadenopathy H. Lymphangitis I. False femoral artery aneurysm What is the likely diagnosis for groin mass described? Each option may be used once, more than once, or not at all. 20. A 52 year old obese lady reports a painless grape sized mass in her groin area.
  • 16. She has no medical conditions apart from some varicose veins. There is a cough impulse and the mass disappears on lying down. Saphenous varix The history of varicose veins should indicate a more likely diagnosis of a varix. The varix can enlarge during coughing/sneezing. A blue discolouration may be noted. 21. A 32 year old male is noted to have a tender mass in the right groin area. There are also red streaks on the thigh, extending from a small abrasion. You answered Lymphangitis The correct answer is Lymphadenitis The red streaks are along the line of the lymphatics, indicating infection of the lymphatic vessels. Lymphadenitis is infection of the local lymph nodes. 22. A 23 year old male suffering from hepatitis C presents with right groin pain and swelling. On examination there is a large abscess in the groin. Adjacent to this is an expansile swelling. There is no cough impulse. False femoral artery aneurysm False aneurysms may occur following arterial trauma in IVDU. They may have associated blood borne virus infections and should undergo duplex scanning prior to surgery. False aneurysms do not contain all layers of the arterial wall. Groin masses clinical Groin masses are common and include:  Herniae  Lipomas  Lymph nodes  Undescended testis  Femoral aneurysm  Saphena varix (more a swelling than a mass!) In the history features relating to systemic illness and tempo of onset will often give a
  • 17. clue as to the most likely underlying diagnosis. Groin lumps- some key questions  Is there a cough impulse  Is it pulsatile AND is it expansile (to distinguish between false and true aneurysm)  Are both testes intra scrotal  Any lesions in the legs such as malignancy or infections (?lymph nodes)  Examine the ano rectum as anal cancer may metastasise to the groin  Is the lump soft, small and very superficial (?lipoma) Scrotal lumps - some key questions  Is the lump entirely intra scrotal  Does it transilluminate (?hydrocele)  Is there a cough impulse (?hernia) In most cases a diagnosis can be made clinically. Where it is not clear an ultrasound scan is often the most convenient next investigation. heme: Right iliac fossa pain A. Urinary tract infection B. Appendicitis C. Mittelschmerz D. Mesenteric adenitis E. Crohns disease F. Ulcerative colitis G. Meckels diverticulum Please select the most likely cause for right iliac fossa pain for the scenario given. Each option may be used once, more than once or not at all. 23. A 17 year old male is admitted with lower abdominal discomfort. He has been suffering from intermittent right iliac fossa pain for the past few months. His past medical history includes a negative colonoscopy and gastroscopy for iron deficiency anaemia. The pain is worse after meals. Inflammatory markers are normal. Meckels diverticulum This scenario should raise suspicion for Meckels as these may contain ectopic gastric mucosa which may secrete acid with subsequent bleeding and ulceration.
  • 18. 24. A 14 year old female is admitted with sudden onset right iliac fossa pain. She is otherwise well and on examination has some right iliac fossa tenderness but no guarding. She is afebrile. Urinary dipstick is normal. Her previous menstrual period two weeks ago was normal and pregnancy test is negative. Mittelschmerz Typical story and timing for mid cycle pain. Mid cycle pain typically occurs because a small amount of fluid is released at the time of ovulation. It will usually resolve over 24-48 hours. 25. A 21 year old male is admitted with a 3 month history of intermittent right iliac fossa pain. He suffers from episodic diarrhoea and has lost 2 kilos in weight. On examination he has some right iliac fossa tenderness and is febrile. Crohns disease Weight loss and chronic symptoms coupled with change in bowel habit should raise suspicion for Crohns. The presence of intermittent right iliac fossa pain is far more typical of terminal ileal Crohns disease. Both UC and Crohns may be associated with a low grade pyrexia. The main concern here would be locally perforated Crohns disease with a small associated abscess. Right iliac fossa pain Differential diagnosis Appendicitis  Pain radiating to right iliac fossa  Anorexia  Typically short history  Diarrhoea and profuse vomiting rare Crohn's disease  Often long history  Signs of malnutrition  Change in bowel habit, especially diarrhoea Mesenteric adenitis  Mainly affects children  Causes include Adenoviruses, Epstein Barr Virus, beta-haemolytic Streptococcus, Staphylococcus spp., Escherichia coli, Streptococcus viridans and Yersinia spp.  Patients have a higher temperature than those with appendicitis
  • 19.  If laparotomy is performed, enlarged mesenteric lymph nodes will be present Diverticulitis  Both left and right sided disease may present with right iliac fossa pain  Clinical history may be similar, although some change in bowel habit is usual  When suspected a CT scan may help in refining the diagnosis Meckel's diverticulitis  A Meckel's diverticulum is a congenital abnormality that is present in about 2% of the population  Typically 2 feet proximal to the ileocaecal valve  May be lined by ectopic gastric mucosal tissue and produce bleeding Perforated peptic ulcer  This usually produces upper quadrant pain but pain may be lower  Perforations typically have a sharp sudden onset of pain in the history Incarcerated right inguinal or femoral hernia  Usually only right iliac fossa pain if right sided or bowel obstruction. Bowel perforation secondary to caecal or colon carcinoma  Seldom localised to right iliac fossa, although complete large bowel obstruction with caecal distension may cause pain prior to perforation. Gynaecological causes  Pelvic inflammatory disease/salpingitis/pelvic abscess/Ectopic pregnancy/Ovarian torsion/Threatened or complete abortion/Mittelschmerz Urological causes  Ureteric colic/UTI/Testicular torsion Other causes  TB/Typhoid/Herpes Zoster/AAA/Situs inversus A 78 year old lady presents with colicky abdominal pain and a tender mass in her groin. On examination there is a small firm mass below and lateral to the pubic tubercle. Which of the following is the most likely underlying diagnosis? A. Incarcerated inguinal hernia B. Thrombophlebitis of a saphena varix C. Incarcerated femoral hernia
  • 20. D. Incarcerated obturator hernia E. Deep vein thrombosis Femoral hernia = High risk of strangulation (repair urgently) Femoral herniae account for <10% of all groin hernias. In the scenario the combination of symptoms of intestinal compromise with a mass in the region of the femoral canal points to femoral hernia as the most likely cause. Femoral canal The femoral canal lies at the medial aspect of the femoral sheath. The femoral sheath is a fascial tunnel containing both the femoral artery laterally and femoral vein medially. The canal lies medial to the vein. Borders of the femoral canal Laterally Femoral vein Medially Lacunar ligament Anteriorly Inguinal ligament Posteriorly Pectineal ligament Image showing dissection of femoral canal
  • 21. Image sourced from Wikipedia Contents  Lymphatic vessels  Cloquet's lymph node Physiological significance Allows the femoral vein to expand to allow for increased venous return to the lower limbs. Pathological significance As a potential space, it is the site of femoral hernias. The relatively tight neck places these at high risk of strangulation. hich of the following is not a typical feature of acute appendicitis? A. Neutrophilia B. Profuse vomiting
  • 22. C. Anorexia D. Low grade pyrexia E. Small amounts of protein on urine analysis Profuse vomiting and diarrhoea are rare in early appendicitis Whilst patients may vomit once or twice, profuse vomiting is unusual, and would fit more with gastroenteritis or an ileus. A trace of protein is not an uncommon occurrence in acute appendicitis. A free lying pelvic appendix may result in localised bladder irritation, with inflammation occurring as a secondary phenomena. This latter feature may result in patients being incorrectly diagnosed as having a urinary tract infection. A urine dipstick test is useful in differentiating between the two conditions. Appendicitis History  Peri umbilical abdominal pain (visceral stretching of appendix lumen and appendix is mid gut structure) radiating to the right iliac fossa due to localised parietal peritoneal inflammation.  Vomit once or twice but marked and persistent vomiting is unusual.  Diarrhoea is rare. However, pelvic appendicitis may cause localised rectal irritation of some loose stools. A pelvic abscess may also cause diarrhoea.  Mild pyrexia is common - temperature is usually 37.5 -38o C. Higher temperatures are more typical of conditions like mesenteric adenitis.  Anorexia is very common. It is very unusual for patients with appendicitis to be hungry. Examination  Generalised peritonitis if perforation has occurred or localised peritonism.  Retrocaecal appendicitis may have relatively few signs.  Digital rectal examination may reveal boggy sensation if pelvic abscess is present, or even tenderness with a pelvic appendix. Diagnosis  Typically raised inflammatory markers coupled with compatible history and examination findings should be enough to justify appendicectomy.  Urine analysis may show mild leucocytosis but no nitrites.  Ultrasound is useful if females where pelvic organ pathology is suspected. Although it is not always possible to visualise the appendix on ultrasound the presence of free fluid (always pathological in males) should raise suspicion.
  • 23. Ultrasound examination may show evidence of lumenal obstruction and thickening of the appendiceal wall as shown below Image sourced from Wikipedia Treatment  Appendicectomy which can be performed via either an open or laparoscopic approach.  Administration of metronidazole reduces wound infection rates.  Patients with perforated appendicitis require copious abdominal lavage.  Patients without peritonitis who have an appendix mass should receive broad spectrum antibiotics and consideration given to performing an interval appendicectomy.  Be wary in the older patients who may have either an underlying caecal malignancy or perforated sigmoid diverticular disease. Laparoscopic appendicectomy is becoming increasing popular as demonstrated below
  • 24. Image sourced from Wikipedia An 28 year old man presents with a direct inguinal hernia. A decision is made to perform an open inguinal hernia repair. Which of the following is the best option for abdominal wall reconstruction in this case? A. Suture plication of the transversalis fascia using PDS only B. Suture plication of the hernial defect with nylon and placement of prolene mesh anterior to external oblique C. Suture plication of the hernia defect using nylon and re-enforcing with a sutured repair of the abdominal wall D. Sutured repair of the hernial defect with prolene and placement of prolene mesh over the cord structures in the inguinal canal E. Sutured repair of the hernial defect using nylon and placement of a prolene mesh posterior to the cord structures Laparoscopic repair- bilateral and recurrent cases During an inguinal hernia repair in males the cord structures will always lie anterior to the mesh. In the conventional open repairs the cord structures are mobilised and the mesh placed behind them, with a slit made to allow passage of the cord structures through the deep inguinal ring. Placement of the mesh over the cord structures results in chronic pain and usually a higher risk of recurrence. Laparoscopic inguinal hernia repair is the procedure of choice for bilateral inguinal
  • 25. hernias. Types of surgery include:  Onlay mesh repair (Litchenstein style)  Inguinal herniorrhaphy  Shouldice repair  Darn repair  Laparoscopic mesh repair Open mesh repair and laparoscopic repair are the two main procedures in mainstream use. The Shouldice repair is a useful procedure in cases where a mesh repair would be associated with increased risk of infection, e.g. repair of case with strangulated bowel, as it avoids the use of mesh. It is, however, far more technically challenging to perform. Inguinal hernia surgery Inguinal hernias occur when the abdominal viscera protrude through the anterior abdominal wall into the inguinal canal. They may be classified as being either direct or indirect. The distinction between these two rests on their relation to Hesselbach's triangle. Boundaries of Hesselbach's Triangle  Medial: Rectus abdominis  Lateral: Inferior epigastric vessels  Inferior: Inguinal ligament
  • 26. Image sourced from Wikipedia Hernias occurring within the triangle tend to be direct and those outside - indirect. Diagnosis Most cases are diagnosed clinically, a reducible swelling may be located at the level of the inguinal canal. Large hernia may extend down into the male scrotum, these will not trans-illuminate and it is not possible to "get above" the swelling. Cases that are unclear on examination, but suspected from the history, may be further investigated using ultrasound or by performing a herniogram. Treatment Hernias associated with few symptoms may be managed conservatively. Symptomatic hernias or those which are at risk of developing complications are usually treated surgically. First time hernias may be treated by performing an open inguinal hernia repair; the inguinal canal is opened, the hernia reduced and the defect repaired. A prosthetic mesh may be placed posterior to the cord structures to re-inforce the repair and reduce the risk of recurrence. Recurrent hernias and those which are bilateral are generally managed with a laparoscopic approach. This may be via an intra or extra peritoneal route. As in open surgery a mesh is deployed. However, it will typically lie posterior to the deep ring. Inguinal hernia in children Inguinal hernias in children are almost always of an indirect type and therefore are usually dealt with by herniotomy, rather than herniorraphy. Neonatal hernias especially in those children born prematurely are at highest risk of strangulation and should be repaired urgently. Other hernias may be repaired on an elective basis.
  • 27. References The UK Based National Institute of Clinical Excellence has published guidelines relating to the choice between open and laparoscopic inguinal hernia repair. Which users may find interesting: Theme: Abdominal closure methods A. Looped 1/0 PDS (polydiaxone) B. Looped 1/0 silk C. 1/0 Vicryl (polyglactin) D. 1/0 Vicryl rapide E. 2/0 Prolene (Polypropylene) F. Re-inforced 1/0 Nylon G. Re-inforced 1/0 Silk H. Application of VAC system without separation film I. Application of VAC System with separation film J. Application of a 'Bogota Bag' Please select the most appropriate wound closure method (for the deep layer) for the abdominal surgery described. 29. A 59 year old man with morbid obesity undergoes a laparotomy and Hartmans procedure for perforated sigmoid diverticular disease. At the conclusion of the procedure the abdomen cannot be primarily closed. The Vac system is not available for use. Application of a 'Bogota Bag' Application of a Bogota bag is safest as attempted closure will almost certainly fail. Repeat look at 48 hours to determine the best definitive option is needed. 30. A 73 year old lady undergoes a low anterior resection for carcinoma of the rectum. Looped 1/0 PDS (polydiaxone) Mass closure obeying Jenkins rule is required and this states that the suture must be 4 times the length of the wound with tissue bites 1cm deep and 1 cm apart. 31. A 67 year old is returned to theatre after developing a burst abdomen on the
  • 28. ward. She has originally undergone a right hemicolectomy and the SHO who closed the wound had failed to tie the midline suture correctly. The wound edges appear healthy. You answered Re-inforced 1/0 Silk The correct answer is Re-inforced 1/0 Nylon Attempt at re-closing the wound is reasonable in which case 1/0 nylon (reinforced with drainage tubing) is often used. Abdominal wound dehiscence  This is a significant problem facing all surgeons who undertake abdominal surgery on a regular basis. Traditionally it is said to occur when all layers of an abdominal mass closure fail and the viscera protrude externally (associated with 30% mortality).  It can be subdivided into superficial, in which the skin wound alone fails and complete, implying failure of all layers. Factors which increase the risk are: * Malnutrition * Vitamin deficiencies * Jaundice * Steroid use * Major wound contamination (e.g. faecal peritonitis) * Poor surgical technique (Mass closure technique is the preferred method-Jenkins Rule) When sudden full dehiscence occurs the management is as follows: * Analgesia * Intravenous fluids * Intravenous broad spectrum antibiotics * Coverage of the wound with saline impregnated gauze (on the ward) * Arrangements made for a return to theatre Surgical strategy  Correct the underlying cause (eg TPN or NG feed if malnourished)  Determine the most appropriate strategy for managing the wound Options Resuturing of the wound This may be an option if the wound edges are healthy and there is enough tissue for sufficient coverage. Deep tension sutures are
  • 29. traditionally used for this purpose. Application of a wound manager This is a clear dressing with removable front. Particularly suitable when some granulation tissue is present over the viscera or where there is a high output bowel fistula present in the dehisced wound. Application of a 'Bogota bag' This is a clear plastic bag that is cut and sutured to the wound edges and is only a temporary measure to be adopted when the wound cannot be closed and will necessitate a return to theatre for definitive management. Application of a VAC dressing system These can be safely used BUT ONLY if the correct layer is interposed between the suction device and the bowel. Failure to adhere to this absolute rule will almost invariably result in the development of multiple bowel fistulae and create an extremely difficult management problem. Theme: Hernias A. Umbilical hernia B. Para umbilical hernia C. Morgagni hernia D. Littres hernia E. Bochdalek hernia F. Richters hernia G. Obturator hernia Please select the hernia that most closely matches the description given. Each option may be used once, more than once or not at all. 32. A 1 day old infant is born with severe respiratory compromise. On examination he has a scaphoid abdomen and an absent apex beat. Bochdalek hernia Theme from 2011 exam The large hernia may displace the heart although true dextrocardia is not present. The associated pulmonary hypoplasia will compromise lung development. 33. A 2 month old infant is troubled by recurrent colicky abdominal pain and intermittent intestinal obstruction. On imaging the transverse colon is herniated into the thoracic cavity, through a mid line defect. Morgagni hernia Morgagni hernia may contain the transverse colon. Unless there is substantial
  • 30. herniation pulmonary hypoplasia is uncommon. 34. A 78 year old lady is admitted with small bowel obstruction, on examination she has a distended abdomen and the leg is held semi flexed. She has some groin pain radiating to the ipsilateral knee. Obturator hernia The groin swelling in obturator hernia is subtle and hard to elicit clinically. Hernia Hernias occur when a viscus or part of it protrudes from within its normal anatomical cavity. Specific hernias are covered under their designated titles the remainder are addressed here. Spigelian hernia  Interparietal hernia occurring at the level of the arcuate line.  Rare.  May lie beneath internal oblique muscle. Usually between internal and external oblique.  Equal sex distribution.  Position is lateral to rectus abdominis.  Both open and laparoscopic repair are possible, the former in cases of strangulation. Lumbar hernia  The lumbar triangle (through which these may occur) is bounded by: Crest of ilium (inferiorly) External oblique (laterally) Latissimus dorsi (medially)  Primary lumbar herniae are rare and most are incisional hernias following renal surgery.  Direct anatomical repair with or without mesh re-enforcement is the procedure of choice. Obturator hernia  Herniation through the obturator canal.
  • 31.  Commoner in females.  Usually lies behind pectineus muscle.  Elective diagnosis is unusual most will present acutely with obstruction.  When presenting acutely most cases with require laparotomy or laparoscopy (and small bowel resection if indicated). Richters hernia  Condition in which part of the wall of the small bowel (usually the anti mesenteric border) is strangulated within a hernia (of any type).  They do not present with typical features of intestinal obstruction as luminal patency is preserved.  Where vomiting is prominent it usually occurs as a result of paralytic ileus from peritonitis (as these hernias may perforate). Incisional hernia  Occur through sites of surgical access into the abdominal cavity.  Most common following surgical wound infection.  To minimise following midline laparotomy Jenkins Rule should be followed and this necessitates a suture length 4x length of incision with bites taken at 1cm intervals, 1 cm from the wound edge.  Repair may be performed either at open surgery or laparoscopically and a wide variety of techniques are described. Bochdalek hernia  Typically congenital diaphragmatic hernia.  85% cases are located in the left hemi diaphragm.  Associated with lung hypoplasia on the affected side.  More common in males.  Associated with other birth defects.  May contain stomach.  May be treated by direct anatomical apposition or placement of mesh. In infants that have severe respiratory compromise mechanical ventilation may be needed and mortality rate is high. Morgagni Hernia  Rare type of diaphragmatic hernia (approx 2% cases).  Herniation through foramen of Morgagni.  Usually located on the right and tend to be less symptomatic.  More advanced cases may contain transverse colon.  As defects are small pulmonary hypoplasia is less common.  Direct anatomical repair is performed.
  • 32. Umbilical hernia  Hernia through weak umbilicus.  Usually presents in childhood.  Often symptomatic.  Equal sex incidence.  95% will resolve by the age of 2 years. Thereafter surgical repair is warranted. Paraumbilical hernia  Usually a condition of adulthood.  Defect is in the linea alba.  More common in females.  Multiparity and obesity are risk factors.  Traditionally repaired using Mayos technique - overlapping repair, mesh may be used though not if small bowel resection is required owing to acute strangulation. Littres hernia  Hernia containing Meckels diverticulum.  Resection of the diverticulum is usually required and this will preclude a mesh repair. Theme: Right iliac fossa pain A. Open Appendicectomy B. Laparoscopic appendicectomy C. Laparotomy D. CT Scan E. Colonoscopy F. Ultrasound scan abdomen/pelvis G. Active observation For each scenario please select the most appropriate management option from the list. Each option may be used once, more than once or not at all. 35. A 21 year old women is admitted with a 48 hour history of worsening right iliac fossa pain. She has been nauseated and vomited twice. On examination she is markedly tender in the right iliac fossa with localised guarding. Vaginal examination is unremarkable. Urine dipstick is negative. Blood tests show a WCC of 13.5 and CRP 70.
  • 33. You answered Open Appendicectomy The correct answer is Laparoscopic appendicectomy She is likely to have appendicitis. In women of this age there is always diagnostic uncertainty. With a normal vaginal exam laparoscopy would be preferred over USS. 36. An 8 year old boy presents with a 4 hour history of right iliac fossa pain with nausea and vomiting. He has been back at school for two days after being kept home with a flu like illness. On examination he is tender in the right iliac fossa, although his abdomen is soft. Temperature is 38.3o c. Blood tests show a CRP of 40 and a WCC of 8.1. You answered CT Scan The correct answer is Active observation This is mesenteric adenitis. Note history of flu like illness and temp > 38o c. The decision as to how to manage this situation is based on the abdominal findings. Patients with localising signs such as guarding or peritonism should undergo surgery. 37. A 21 year old women presents with right iliac fossa pain. She reports some bloodstained vaginal discharge. She has a HR of 65 bpm. Ultrasound scan abdomen/pelvis This patient is suspected of having an ectopic pregnancy. She needs an urgent β HCG and USS of the pelvis. If she were haemodynamically unstable then laparotomy would be indicated. Right iliac fossa pain Differential diagnosis Appendicitis  Pain radiating to right iliac fossa  Anorexia  Typically short history  Diarrhoea and profuse vomiting rare Crohn's disease  Often long history  Signs of malnutrition
  • 34.  Change in bowel habit, especially diarrhoea Mesenteric adenitis  Mainly affects children  Causes include Adenoviruses, Epstein Barr Virus, beta-haemolytic Streptococcus, Staphylococcus spp., Escherichia coli, Streptococcus viridans and Yersinia spp.  Patients have a higher temperature than those with appendicitis  If laparotomy is performed, enlarged mesenteric lymph nodes will be present Diverticulitis  Both left and right sided disease may present with right iliac fossa pain  Clinical history may be similar, although some change in bowel habit is usual  When suspected a CT scan may help in refining the diagnosis Meckel's diverticulitis  A Meckel's diverticulum is a congenital abnormality that is present in about 2% of the population  Typically 2 feet proximal to the ileocaecal valve  May be lined by ectopic gastric mucosal tissue and produce bleeding Perforated peptic ulcer  This usually produces upper quadrant pain but pain may be lower  Perforations typically have a sharp sudden onset of pain in the history Incarcerated right inguinal or femoral hernia  Usually only right iliac fossa pain if right sided or bowel obstruction. Bowel perforation secondary to caecal or colon carcinoma  Seldom localised to right iliac fossa, although complete large bowel obstruction with caecal distension may cause pain prior to perforation. Gynaecological causes  Pelvic inflammatory disease/salpingitis/pelvic abscess/Ectopic pregnancy/Ovarian torsion/Threatened or complete abortion/Mittelschmerz Urological causes  Ureteric colic/UTI/Testicular torsion Other causes  TB/Typhoid/Herpes Zoster/AAA/Situs inversus Which of the following is not a typical feature of irritable bowel syndrome?
  • 35. A. A change in the consistency of stools B. Abdominal pain relieved with defecation C. A change in frequency of defecation D. Abdominal bloating E. Pain at a single fixed site The pain or discomfort of IBS is typically migratory and variable in intensity. Pain at a fixed site is suggestive of malignancy. Abdominal bloating is an extremely common feature. Irritable bowel syndrome The diagnosis of irritable bowel syndrome is made according to the ROME III diagnostic criteria which state: Recurrent abdominal pain or discomfort at 3 days per month for the past 3 months associated with two or more of the following:  Improvement with defecation.  Onset associated with a change in the frequency of stool.  Onset associated with a change in the form of the stool. Features such as lethargy, nausea, backache and bladder symptoms may also support the diagnosis Red flag features should be inquired about:  Rectal bleeding  Unexplained/unintentional weight loss  Family history of bowel or ovarian cancer  Onset after 60 years of age Suggested investigations are:  Full blood count  ESR/CRP  Coeliac disease screen (tissue transglutaminase antibodies)  Colonoscopy (if worrying symptoms, positive family history)  Thyroid function tests  Glucose (ensure not diabetic)
  • 36. The NICE criteria state that blood tests alone will suffice in people fulfilling the diagnostic criteria. We would point out that luminal colonic studies should be considered early in patients with altered bowel habit referred to hospital and a diagnosis of IBS should still be largely one of exclusion. Treatment  Usually reduce fibre intake.  Tailored prescriptions of laxatives or loperamide according to clinical picture.  Dietary modification (caffeine avoidance, less carbonated drinks).  Consider low dose tricyclic antidepressants if pain is a dominant symptom.  Biofeedback may help. Theme: Causes of abdominal pain A. Acute on chronic mesenteric ischaemia B. Ruptured aortic aneurysm C. Acute Pancreatitis D. Acute mesenteric embolus E. Acute appendicitis F. Chronic pancreatitis G. Mesenteric vein thrombosis Please select the most likely underlying diagnosis from the list above. Each option may be used once, more than once or not at all. 39. A 41 year old man is admitted with peritonitis secondary to a perforated appendix. He is treated with a laparoscopic appendicectomy but has a stormy post operative course. He in now developing increasing abdominal pain and has been vomiting. A laparotomy is performed and at operation a large amount of small bowel shows evidence of patchy areas of infarction. Mesenteric vein thrombosis Mesenteric vein thrombosis may complicate severe intra abdominal sepsis and when it progresses may impair bowel perfusion. The serosa is quite resistant to ischaemia so in this case the appearances are usually patchy. 40. A 68 year old man is admitted with abdominal pain and vomiting of 48 hours duration, the pain radiates to his back and he has required a considerable amount of volume replacement. Amylase is 741.
  • 37. Acute Pancreatitis Although back pain and abdominal pain coupled with haemodynamic compromise may suggest ruptured AAA the 48 hour history and amylase >3 times normal go against this diagnosis. 41. A 79 year old lady develops sudden onset of abdominal pain and collapses, she has passed a large amount of diarrhoea. In casualty her pH is 7.35 and WCC is 18. You answered Acute on chronic mesenteric ischaemia The correct answer is Acute mesenteric embolus Although mesenteric infarct may raise the lactate the pH may be raised often secondary to vomiting. Mesenteric vessel disease Mesenteric ischaemia accounts for 1 in 1000 acute surgical admissions. It is primarily caused by arterial embolism resulting in infarction of the colon. It is more likely to occur in areas such as the splenic flexure that are located at the borders of the territory supplied by the superior and inferior mesenteric arteries. Types Acute mesenteric embolus (commonest 50%)  Sudden onset abdominal pain followed by profuse diarrhoea.  May be associated with vomiting.  Rapid clinical deterioration.  Serological tests: WCC, lactate, amylase may all be abnormal particularly in established disease. These can be normal in the early phases. Acute on chronic mesenteric ischaemia  Usually longer prodromal history.  Post prandial abdominal discomfort and weight loss are dominant features. Patients will usually present with an acute on chronic event, but otherwise will tend not to present until mesenteric flow is reduced by greater than 80%.  When acute thrombosis occurs presentation may be as above. In the chronic setting the symptoms will often be those of ischaemic colitis (mucosa is the most sensitive area to this insult).
  • 38. Mesenteric vein thrombosis  Usually a history over weeks.  Overt abdominal signs and symptoms will not occur until venous thrombosis has reached a stage to compromise arterial inflow.  Thrombophilia accounts for 60% of cases. Low flow mesenteric infarction  This occurs in patients with multiple co morbidities in whom mesenteric perfusion is significantly compromised by overuse of inotropes or background cardiovascular compromise.  The end result is that the bowel is not adequately perfused and infarcts occur from the mucosa outwards. Diagnosis  Serological tests: WCC, lactate, CRP, amylase (can be normal in early disease).  Cornerstone for diagnosis of arterial AND venous mesenteric disease is CT angiography scanning in the arterial phase with thin slices (<5mm). Venous phase contrast is not helpful.  SMA duplex USS is useful in the evaluation of proximal SMA disease in patients with chronic mesenteric ischaemia.  MRI is of limited use due to gut peristalsis and movement artefact. Management  Overt signs of peritonism: Laparotomy  Mesenteric vein thrombosis: If no peritonism: Medical management with IV heparin  At operation limited resection of frankly necrotic bowel with view to relook laparotomy at 24-48h. In the interim urgent bowel revascularisation via endovascular (preferred) or surgery. Prognosis Overall poor. Best outlook is from an acute ischaemia from an embolic event where surgery occurs within 12h. Survival may be 50%. This falls to 30% with treatment delay. The other conditions carry worse survival figures. Theme: Groin masses A. Femoral aneurysm B. Lymphadenitis C. Saphena varix D. Femoral hernia E. Indirect inguinal hernia F. Direct inguinal hernia
  • 39. G. Psoas abscess What is the likely diagnosis for the groin mass described? Each option may be used once, more than once or not at all. 42. A 3 year old boy is referred to the clinic with a scrotal swelling. On examination the mass does not transilluminate and it is impossible to palpate normal cord above it. Indirect inguinal hernia This is likely to be an indirect hernia. In children these arise from persistent processus vaginalis and require herniotomy. 43. A 52 year old obese lady reports a painless mass in the groin area. A mass is noted on coughing. It is below and lateral to the pubic tubercle. Femoral hernia A mass below and lateral to the pubic tubercle is indicative of a femoral hernia. 44. A 21 year old man is admitted with a tender mass in the right groin, fevers and sweats. He is on multiple medical therapy for HIV infection. On examination he has a swelling in his right groin, hip extension exacerbates the pain. Psoas abscess Psoas abscesses may be either primary or secondary. Primary cases often occur in the immunosuppressed and may occur as a result of haematogenous spread. Secondary cases may complicated intra abdominal diseases such as Crohns. Patients usually present with low back pain and if the abscess is extensive a mass that may be localised to the inguinal region or femoral triangle . Smaller collections may be percutaneously drained. If the collection is larger, or the percutaneous route fails, then surgery (via a retroperitoneal approach) should be performed. Groin masses clinical Groin masses are common and include:  Herniae
  • 40.  Lipomas  Lymph nodes  Undescended testis  Femoral aneurysm  Saphena varix (more a swelling than a mass!) In the history features relating to systemic illness and tempo of onset will often give a clue as to the most likely underlying diagnosis. Groin lumps- some key questions  Is there a cough impulse  Is it pulsatile AND is it expansile (to distinguish between false and true aneurysm)  Are both testes intra scrotal  Any lesions in the legs such as malignancy or infections (?lymph nodes)  Examine the ano rectum as anal cancer may metastasise to the groin  Is the lump soft, small and very superficial (?lipoma) Scrotal lumps - some key questions  Is the lump entirely intra scrotal  Does it transilluminate (?hydrocele)  Is there a cough impulse (?hernia) In most cases a diagnosis can be made clinically. Where it is not clear an ultrasound scan is often the most convenient next investigation. A 56 year old lady is admitted with colicky abdominal pain. A plain x-ray is performed. Which of the following should not show fluid levels on a plain abdominal film? A. Stomach B. Jejunum C. Ileum D. Caecum E. Descending colon Fluid levels in the distal colon are nearly always pathological. In general contents of the left colon transit quickly and are seldom held in situ for long periods, the content is also more solid.
  • 41. Abdominal radiology Plain abdominal x-rays are often used as a first line investigation in patients with acute abdominal pain. A plain abdominal film may demonstrate free air, evidence of bowel obstruction and possibly other causes of pain (e.g. renal or gallbladder stones). Investigation of potential visceral perforation is usually best performed by obtaining an erect chest x-ray, as this is a more sensitive investigation for suspected visceral perforation. Features which are usually abnormal  Large amounts of free air (colonic perforation), smaller volumes seen with more proximal perforations.  A positive Riglers sign (gas on both sides of the bowel wall).  Caecal diameter of >8cm  Fluid levels in the colon  Ground glass appearance to film (usually due to large amounts of free fluid).  Sentinel loop in patients with inflammation of other organs (e.g. pancreatitis). Features which should be expected/ or occur without pathology  In Chialditis syndrome, a loop of bowel may be interposed between the liver and diaphragm, giving the mistaken impression that free air is present.  Following ERCP (and sphincterotomy) air may be identified in the biliary tree.  Free intra abdominal air following laparoscopy / laparotomy, although usually dissipates after 48-72 hours. A 56 year old lady presents with a large bowel obstruction and abdominal distension. Which of the following confirmatory tests should be performed prior to surgery A. Abdominal ultrasound scan B. Barium enema C. Rectal MRI Scan D. Endoanal ultrasound scan E. Gastrograffin enema Patients with clinical evidence of large bowel obstruction, should have the presence or absence of an obstructing lesion confirmed prior to surgery. This is because colonic pseudo-obstruction may produce a similar radiological picture. A gastrograffin enema is the traditional test, as barium is too toxic if it spills into the abdominal cavity. An MRI scan will not provide the relevant information, unless the lesion is rectal and below the peritoneal reflection.
  • 42. Abdominal radiology Plain abdominal x-rays are often used as a first line investigation in patients with acute abdominal pain. A plain abdominal film may demonstrate free air, evidence of bowel obstruction and possibly other causes of pain (e.g. renal or gallbladder stones). Investigation of potential visceral perforation is usually best performed by obtaining an erect chest x-ray, as this is a more sensitive investigation for suspected visceral perforation. Features which are usually abnormal  Large amounts of free air (colonic perforation), smaller volumes seen with more proximal perforations.  A positive Riglers sign (gas on both sides of the bowel wall).  Caecal diameter of >8cm  Fluid levels in the colon  Ground glass appearance to film (usually due to large amounts of free fluid).  Sentinel loop in patients with inflammation of other organs (e.g. pancreatitis). Features which should be expected/ or occur without pathology  In Chialditis syndrome, a loop of bowel may be interposed between the liver and diaphragm, giving the mistaken impression that free air is present.  Following ERCP (and sphincterotomy) air may be identified in the biliary tree.  Free intra abdominal air following laparoscopy / laparotomy, although usually dissipates after 48-72 hours. Theme: Management of splenic trauma A. Splenectomy B. Angiography C. CT Scan D. Admit for bed rest and observation E. Ultrasound scan F. Splenic conservation G. MRI of the abdomen Please select the most appropriate intervention for the scenario given. Each option may be used once, more than once or not at all. 1. A 7 year old boy falls off a wall the distance is 7 feet. He lands on his left side and there is left flank bruising. There is no haematuria. He is otherwise stable and haemoglobin is within normal limits. Ultrasound scan
  • 43. This will demonstrate any overt splenic injury. A CT scan carries a significant dose of radiation. In the absence of haemodynamic instability or other major associated injuries the use of USS to exclude intraabdominal free fluid (blood) would seem safe when coupled with active observation. An USS will also show splenic haematomas. 2. A 42 year old motorcyclist is involved in a road traffic accident. A FAST scan in the emergency department shows free intrabdominal fluid and a laparotomy is performed. At operation there is evidence of small liver laceration that has stopped bleeding and a tear to the inferior pole of the spleen. Splenic conservation As minimum damage, attempt conservation. 3. An 18 year old man is involved in a road traffic accident. A CT scan shows disruption of the splenic hilum and a moderate sized perisplenic haematoma. Splenectomy Hilar injuries usually mandate splenectomy. The main risk with conservative management here is that he will rebleed and with hilar injuries this can be dramatic. Splenic trauma  The spleen is one of the more commonly injured intra abdominal organs  In most cases the spleen can be conserved. The management is dictated by the associated injuries, haemodynamic status and extent of direct splenic injury. Management of splenic trauma Conservative Small subcapsular haematoma Minimal intra abdominal blood No hilar disruption Laparotomy with conservation Increased amounts of intraabdominal blood Moderate haemodynamic compromise Tears or lacerations affecting <50% Resection Hilar injuries Major haemorrhage Major associated injuries
  • 44. Splenectomy Technique Trauma  GA  Long midline incision  If time permits insert a self retaining retractor (e.g. Balfour/ omnitract)  Large amount of free blood is usually present. Pack all 4 quadrants of the abdomen. Allow the anaesthetist to 'catch up'  Remove the packs and assess the viability of the spleen. Hilar injuries and extensive parenchymal lacerations will usually require splenectomy.  Divide the short gastric vessels and ligate them.  Clamp the splenic artery and vein. Two clamps on the patient side are better and allow for double ligation and serve as a safety net if your assistant does not release the clamp smoothly.  Be careful not to damage the tail of the pancreas, if you do then this will need to be formally removed and the pancreatic duct closed.  Wash out the abdomen and place a tube drain to the splenic bed.  Some surgeons implant a portion of spleen into the omentum, whether you decide to do this is a matter of personal choice.  Post operatively the patient will require prophylactic penicillin V and pneumococcal vaccine. Elective Elective splenectomy is a very different operation from that performed in the emergency setting. The spleen is often large (sometimes massive). Most cases can be performed laparoscopically. The spleen will often be macerated inside a specimen bag to facilitate extraction. Complications  Haemorrhage (may be early and either from short gastrics or splenic hilar vessels  Pancreatic fistula (from iatrogenic damage to pancreatic tail)  Thrombocytosis: prophylactic aspirin  Encapsulated bacteria infection e.g. Strep. pneumoniae, Haemophilus influenzae and Neisseria meningitidis Which of the following does not increase the risk of abdominal wound dehiscence following laparotomy? A. Jaundice B. Abdominal compartment syndrome C. Poorly controlled diabetes mellitus
  • 45. D. Administration of intravenous steroids E. Use of Ketamine as an anaesthetic agent Ketamine does not affect healing. All the other situations in the list carry a strong association with poor healing and risk of dehisence. Abdominal wound dehiscence  This is a significant problem facing all surgeons who undertake abdominal surgery on a regular basis. Traditionally it is said to occur when all layers of an abdominal mass closure fail and the viscera protrude externally (associated with 30% mortality).  It can be subdivided into superficial, in which the skin wound alone fails and complete, implying failure of all layers. Factors which increase the risk are: * Malnutrition * Vitamin deficiencies * Jaundice * Steroid use * Major wound contamination (e.g. faecal peritonitis) * Poor surgical technique (Mass closure technique is the preferred method-Jenkins Rule) When sudden full dehiscence occurs the management is as follows: * Analgesia * Intravenous fluids * Intravenous broad spectrum antibiotics * Coverage of the wound with saline impregnated gauze (on the ward) * Arrangements made for a return to theatre Surgical strategy  Correct the underlying cause (eg TPN or NG feed if malnourished)  Determine the most appropriate strategy for managing the wound Options Resuturing of the wound This may be an option if the wound edges are healthy and there is enough tissue for sufficient coverage. Deep tension sutures are traditionally used for this purpose. Application of a wound manager This is a clear dressing with removable front. Particularly suitable when some granulation tissue is present over the viscera or where there is a high output bowel fistula present in the dehisced wound. Application of a This is a clear plastic bag that is cut and sutured to the wound edges
  • 46. 'Bogota bag' and is only a temporary measure to be adopted when the wound cannot be closed and will necessitate a return to theatre for definitive management. Application of a VAC dressing system These can be safely used BUT ONLY if the correct layer is interposed between the suction device and the bowel. Failure to adhere to this absolute rule will almost invariably result in the development of multiple bowel fistulae and create an extremely difficult management problem. Theme: Causes of diarrhoea A. Campylobacter jejuni infection B. Salmonella gastroenteritis infection C. Crohns disease D. Ulcerative colitis E. Irritable bowel syndrome F. Ischaemic colitis G. Laxative abuse H. Clostridium difficile infection Please select the most likely cause of diarrhoea for each scenario given. Each option may be used once, more than once or not at all. 5. A 23 year old lady has suffered from diarrhoea for 8 months, she has also lost 2 Kg in weight. At colonoscopy appearances of melanosis coli are identified and confirmed on biopsy You answered Ulcerative colitis The correct answer is Laxative abuse This may occur as a result of laxative abuse and consists of lipofuschin laden marcophages that appear brown. 6. A 68 year old lady has recently undergone an abdominal aortic aneurysm repair. The operation was performed electively and was uncomplicated. Since surgery she has had repeated episodes of diarrhoea. Ischaemic colitis The IMA is commonly ligated during and AAA repair and this may then render the left colon relatively ischaemic, thereby causing mesenteric colitis. Treatment is supportive and most cases will settle with conservative management.
  • 47. 7. A 23 year old man is admitted to hospital with diarrhoea and severe abdominal pain. He was previously well and his illness has lasted 18 hours. You answered Irritable bowel syndrome The correct answer is Campylobacter jejuni infection Severe abdominal pain tends to favour Campylobacter infection. Diarrhoea World Health Organisation definitions Diarrhoea: > 3 loose or watery stool per day Acute diarrhoea < 14 days Chronic diarrhoea > 14 days Acute Diarrhoea Gastroenteritis May be accompanied by abdominal pain or nausea/vomiting Diverticulitis Classically causes left lower quadrant pain, diarrhoea and fever Antibiotic therapy More common with broad spectrum antibiotics Clostridium difficile is also seen with antibiotic use Constipation causing overflow A history of alternating diarrhoea and constipation may be given May lead to faecal incontinence in the elderly Chronic Diarrhoea Irritable bowel syndrome Extremely common. The most consistent features are abdominal pain, bloating and change in bowel habit. Patients may be divided into those with diarrhoea predominant IBS and those with constipation predominant IBS. Features such as lethargy, nausea, backache and bladder symptoms may also be present Ulcerative colitis Bloody diarrhoea may be seen. Crampy abdominal pain and weight loss are also common. Faecal urgency and tenesmus may occur Crohn's disease Crampy abdominal pains and diarrhoea. Bloody diarrhoea less common than in ulcerative colitis. Other features include malabsorption, mouth ulcers perianal disease and intestinal obstruction Colorectal cancer Symptoms depend on the site of the lesion but include diarrhoea, rectal bleeding, anaemia and constitutional symptoms e.g. Weight loss and anorexia
  • 48. Coeliac disease  In children may present with failure to thrive, diarrhoea and abdominal distension  In adults lethargy, anaemia, diarrhoea and weight loss are seen. Other autoimmune conditions may coexist Other conditions associated with diarrhoea include:  Thyrotoxicosis  Laxative abuse  Appendicitis with pelvic abscess or pelvic appendix  Radiation enteritis Diagnosis Stool culture Abdominal and digital rectal examination Consider colonoscopy (radiological studies unhelpful) Thyroid function tests, serum calcium, anti endomysial antibodies, glucose A 6 year old child presents with colicky abdominal pain, vomiting and the passage of red current jelly stool per rectum. On examination the child has a tender abdomen and a palpable mass in the right upper quadrant. Imaging shows an intussusception. Which of the conditions below is least recognised as a precipitant A. Inflammation of Payers patches B. Cystic fibrosis C. Meckels diverticulum D. Mesenteric cyst E. Mucosal polyps Mesenteric cysts may be associated with intra abdominal catastrophes where these occur they are typically either intestinal volvulus or intestinal infarction. They seldom cause intussusception. Cystic fibrosis may lead to the formation of meconium ileus equivalent and plugs may occasionally serve as the lead points for an intussusception. Intussusception- Paediatric Intussusception typcially presents with colicky abdominal pain and vomiting. The telescoping of the bowel produces mucosal ischaemia and bleeding may occur resulting in the passage of "red current jelly" stools. Recognised causes include lumenal pathologies such as polyps, lymphadenopathy and diseases such as cystic fibrosis. Idiopathic intussceception of the ileocaecal valve and terminal ileum is the most common variant and typically affects young children and toddlers. The diagnosis is usually made by abdominal ultrasound investigation. The decision as
  • 49. to the optimal treatment is dictated by the patients physiological status and abdominal signs. In general children who are unstable with localising peritoneal signs should undergo laparotomy as should those in whom attempted radiological reduction has failed. In relatively well children without localising signs attempted hydrostatic reduction under fluroscopic guidance is the usual treatment. Which one of the following is least likely to cause malabsorption? A. Primary biliary cirrhosis B. Ileo-colic bypass C. Chronic pancreatitis D. Whipples disease E. Hartmans procedure In a Hartmans procedure the sigmoid colon is removed and an end colostomy is fashioned. The bowel remains in continuity and no absorptive ability is lost. An ileo-colic bypass leaves a redundant loop of small bowel in continuity, where the contents will stagnate and bacterial overgrowth will occur. Therefore this is recognised cause of malabsorption. Malabsorption Malabsorption is characterised by diarrhoea, steatorrhoea and weight loss. Causes may be broadly divided into intestinal (e.g. villous atrophy), pancreatic (deficiency of pancreatic enzyme production or secretion) and biliary (deficiency of bile-salts needed for emulsification of fats) Intestinal causes of malabsorption  coeliac disease  Crohn's disease  tropical sprue  Whipple's disease  Giardiasis  brush border enzyme deficiencies (e.g. lactase insufficiency) Pancreatic causes of malabsorption  chronic pancreatitis  cystic fibrosis  pancreatic cancer
  • 50. Biliary causes of malabsorption  biliary obstruction  primary biliary cirrhosis Other causes  bacterial overgrowth (e.g. systemic sclerosis, diverticulae, blind loop)  short bowel syndrome  lymphoma Theme: Intra abdominal malignancies A. Metastatic adenocarcinoma of the pancreas B. Metastatic appendiceal carcinoid C. Metastatic colonic cancer D. Pseudomyxoma peritonei E. MALT lymphoma F. Retroperitoneal liposarcoma G. Retroperitoneal fibrosis For the disease given please give the most likely primary disease process. Each option may be used once, more than once or not at all. 10. A 32 year old man is admitted with a distended tense abdomen. He previously underwent a difficult appendicectomy 1 year previously and was discharged. At laparotomy the abdomen is filled with a gelatinous substance. You answered Metastatic appendiceal carcinoid The correct answer is Pseudomyxoma peritonei Pseudomyxoma is classically associated with mucin production and the appendix is the commonest source. 11. A 62 year old man is admitted with dull lower back pain and abdominal discomfort. On examination he is hypertensive and a lower abdominal fullness is elicited on examination. An abdominal ultrasound demonstrates hydronephrosis and intravenous urography demonstrated medially displaced ureters. A CT scan shows a periaortic mass. You answered Metastatic colonic cancer
  • 51. The correct answer is Retroperitoneal fibrosis Retroperitoneal fibrosis is an uncommon condition and its aetiology is poorly understood. In a significant proportion the ureters are displaced medially. In most retroperitoneal malignancies they are displaced laterally. Hypertension is another common finding. A CT scan will often show a para-aortic mass 12. A 48 year old lady is admitted with abdominal distension. On examination she is cachectic and has ascites. Her CA19-9 returns highly elevated. Metastatic adenocarcinoma of the pancreas Although not specific CA 19-9 in the context of this history is highly suggestive of pancreatic cancer over the other scenarios. Pseudomyxoma peritoneii- Curative treatment is peritonectomy (Sugarbaker procedure) and heated intra peritoneal chemotherapy. Pseudomyxoma Peritonei  Rare mucinous tumour  Most commonly arising from the appendix (other abdominal viscera are also recognised as primary sites)  Incidence of 1-2/1,000,000 per year  The disease is characterised by the accumulation of large amounts of mucinous material in the abdominal cavity Treatment Is usually surgical and consists of cytoreductive surgery (and often peritonectomy c.f Sugarbaker procedure) combined with intra peritoneal chemotherapy with mitomycin C. Survival is related to the quality of primary treatment and in Sugarbakers own centre 5 year survival rates of 75% have been quoted. Patients with disseminated intraperitoneal malignancy from another source fare far worse. In selected patients a second look laparotomy is advocated and some practice this routinely. Theme: Abdominal pain A. Acute mesenteric embolus B. Acute on chronic mesenteric ischaemia
  • 52. C. Mesenteric vein thrombosis D. Ruptured abdominal aortic aneurysm E. Pancreatitis F. Appendicitis G. Acute cholecystitis Please select the most likely underlying diagnosis from the list above. Each option may be used once, more than once or not at all. 13. A 72 year old man collapses with sudden onset abdominal pain. He has been suffering from back pain recently and has been taking ibuprofen. Ruptured abdominal aortic aneurysm Back pain is a common feature with expanding aneurysms and may be miss classified as being of musculoskeletal origin. 14. A 73 year old women collapses with sudden onset of abdominal pain and the passes a large amount of diarrhoea. On admission she is vomiting repeatedly. She has recently been discharged from hospital following a myocardial infarct but recovered well. You answered Mesenteric vein thrombosis The correct answer is Acute mesenteric embolus Sudden onset of abdominal pain and forceful bowel evacuation are features of acute mesenteric infarct. 15. A 66 year old man has been suffering from weight loss and develops severe abdominal pain. He is admitted to hospital and undergoes a laparotomy. At operation the entire small bowel is infarcted and only the left colon is viable. You answered Mesenteric vein thrombosis The correct answer is Acute on chronic mesenteric ischaemia This man is likely to have underlying chronic mesenteric vascular disease. Only 15% of emboli will occlude SMA orifice leading to entire small bowel infarct. The background history of weight loss also favours an acute on chronic event.
  • 53. Mesenteric vessel disease Mesenteric ischaemia accounts for 1 in 1000 acute surgical admissions. It is primarily caused by arterial embolism resulting in infarction of the colon. It is more likely to occur in areas such as the splenic flexure that are located at the borders of the territory supplied by the superior and inferior mesenteric arteries. Types Acute mesenteric embolus (commonest 50%)  Sudden onset abdominal pain followed by profuse diarrhoea.  May be associated with vomiting.  Rapid clinical deterioration.  Serological tests: WCC, lactate, amylase may all be abnormal particularly in established disease. These can be normal in the early phases. Acute on chronic mesenteric ischaemia  Usually longer prodromal history.  Post prandial abdominal discomfort and weight loss are dominant features. Patients will usually present with an acute on chronic event, but otherwise will tend not to present until mesenteric flow is reduced by greater than 80%.  When acute thrombosis occurs presentation may be as above. In the chronic setting the symptoms will often be those of ischaemic colitis (mucosa is the most sensitive area to this insult). Mesenteric vein thrombosis  Usually a history over weeks.  Overt abdominal signs and symptoms will not occur until venous thrombosis has reached a stage to compromise arterial inflow.  Thrombophilia accounts for 60% of cases. Low flow mesenteric infarction  This occurs in patients with multiple co morbidities in whom mesenteric perfusion is significantly compromised by overuse of inotropes or background cardiovascular compromise.  The end result is that the bowel is not adequately perfused and infarcts occur from the mucosa outwards. Diagnosis  Serological tests: WCC, lactate, CRP, amylase (can be normal in early disease).  Cornerstone for diagnosis of arterial AND venous mesenteric disease is CT angiography scanning in the arterial phase with thin slices (<5mm). Venous phase contrast is not helpful.
  • 54.  SMA duplex USS is useful in the evaluation of proximal SMA disease in patients with chronic mesenteric ischaemia.  MRI is of limited use due to gut peristalsis and movement artefact. Management  Overt signs of peritonism: Laparotomy  Mesenteric vein thrombosis: If no peritonism: Medical management with IV heparin  At operation limited resection of frankly necrotic bowel with view to relook laparotomy at 24-48h. In the interim urgent bowel revascularisation via endovascular (preferred) or surgery. Prognosis Overall poor. Best outlook is from an acute ischaemia from an embolic event where surgery occurs within 12h. Survival may be 50%. This falls to 30% with treatment delay. The other conditions carry worse survival figures. Theme: Surgical incisions A. Lanz incision B. Gridiron incision C. Kochers incision D. Rutherford Morrison E. Rooftop incision F. McEvedy Incision G. Lothissen Incision Please select the most appropriate incision for the procedure described. Each option may be used once, more than once or not at all. 16. A 78 year old lady is admitted with an incarcerated femoral hernia. Abdominal signs are absent and there are no symptoms of obstruction. AXR is normal. McEvedy Incision From the list the McEvedy approach is the most appropriate. The Lothissen incision may compromise the posterior wall of the inguinal canal and is best avoided. The
  • 55. author prefers a limited pfannenstial type incision for this procedure, as it gives better control of the hernia, but this is not on the list. 17. A 15 year old girl presents with right iliac fossa pain and guarding, pregnancy test is negative and WCC is 16. Lanz incision She requires an appendicectomy although there is an increasing vogue for performing this procedure laparoscopically an open procedure is entirely suitable. However, although both a Gridiron and Lanz incision are suitable for appendicectomy a Lanz will give a superior cosmetic result and would be the preferred option for most young females. 18. A 45 year old man is due to undergo a live donor renal transplant. This will be his first procedure. Rutherford Morrison The Rutherford Morrison incision will typically give access to the iliac vessels and bladder for the procedure Abdominal incisions Theme in January 2012 exam Midline incision  Commonest approach to the abdomen  Structures divided: linea alba, transversalis fascia, extraperitoneal fat, peritoneum (avoid falciform ligament above the umbilicus)  Bladder can be accessed via an extraperitoneal approach through the space of Retzius Paramedian incision  Parallel to the midline (about 3-4cm)  Structures divided/retracted: anterior rectus sheath, rectus (retracted), posterior rectus sheath, transversalis fascia, extraperitoneal fat, peritoneum  Incision is closed in layers
  • 56. Battle  Similar location to paramedian but rectus displaced medially (and thus denervated)  Now seldom used Kocher's Incision under right subcostal margin e.g. Cholecystectomy (open) Lanz Incision in right iliac fossa e.g. Appendicectomy Gridiron Oblique incision centered over McBurneys point- usually appendicectomy (less cosmetically acceptable than Lanz Gable Rooftop incision Pfannenstiel's Transverse supra pubic, primarily used to access pelvic organs McEvedy's Groin incision e.g. Emergency repair strangulated femoral hernia Rutherford Morrison Extraperitoneal approach to left or right lower quadrants. Gives excellent access to iliac vessels and is the approach of choice for first time renal transplantation. Image sourced from Wikipedia Theme: Acute abdominal pain A. Appendicitis B. Henoch Schonlein purpura C. Diabetes mellitus D. Intussusception E. Mittelschmerz F. Pneumonia
  • 57. G. Sickle cell crisis H. Spontaneous bacterial peritonitis I. Rupure of follicular cyst Please select the most likely cause of abdominal pain for the scenario given. Each option may be used once, more than once or not at all. 19. An 11 month-old girl develops sudden onset abdominal pain. She has a high pitched scream and draws up her legs. Her BP is 90/40 mm/Hg, her pulse 118/min and abdominal examination is normal. Intussusception Intussusception should be considered in toddlers and infants presenting with screaming attacks. The child often has a history of being unwell for one to three days prior to presentation. The child may pass bloody mucus stool, which is a late sign. Examination of the abdomen is often normal as the sausage mass in the right upper quadrant is difficult to feel. 20. An 8 year-old West Indian boy presents with periumbilical abdominal pain. He has vomited twice and is refusing fluids. His temperature is 38.1o C and blood tests are as follows: Haemoglobin 8 g/dl, WCC 13 x 109 /l, with a neutrophilia. Sickle cell crisis Sickle cell anaemia is characterised by severe chronic haemolytic anaemia resulting from poorly formed erythrocytes. Painful crises result from vaso- occlusive episodes, which may occur spontaneously or may be precipitated by infection. Consider this diagnosis in all children of appropriate ethnic background. 21. A 15-month-old girl presents with a three day history of periorbital oedema. She is brought to hospital. On examination she has facial oedema and a tender distended abdomen. Her temperature is 39o C and her blood pressure is 90/45 mmHg. There is clinical evidence of poor peripheral perfusion. Spontaneous bacterial peritonitis The 15-month-old girl is a patient with nephrotic syndrome. Patients with this condition are at risk of septicaemia and peritonitis from Streptococcus pneumoniae, due to the loss of immunoglobulins and opsonins in the urine.
  • 58. Acute abdominal pain-diagnoses Conditions presenting with acute abdominal pain Condition Features Investigations Management Appendicitis History of migratory pain. Fever. Anorexia. Evidence of right iliac fossa tenderness. Mild pyrexia. Differential white cell count Pregnancy test C-Reactive protein Amylase Urine dipstick testing Appendicectomy Mesenteric adenitis Usually recent upper respiratory tract infection. High fever. Generalised abdominal discomfort- true localised pain and signs are rare. Full blood count- may show slightly raised white cell count Urine dipstick often normal Abdominal ultrasound scan - usually no free fluid Conservative management- appendicectomy if diagnostic doubt Mittelschmerz Only seen in females. Mid cycle pain. Usually occurs two weeks after last menstrual period. Pain is usually has a supra-pubic location. Usually subsides over a 24-48 hour period. Full blood count- normal Urine dipstick- normal Abdominal and pelvic ultrasound- may show a trace of pelvic free fluid Manage conservatively if doubt or symptoms fail to settle then laparoscopy Fitz-Hugh Curtis syndrome Disseminated infection with Chlamydia. Usually seen in females. Consists of evidence of pelvic inflammatory disease together with peri-hepatic inflammation and subsequent adhesion formation. Abdominal ultrasound scan- may show free fluid High vaginal swabs - may show evidence of sexually transmitted infections Usually medically managed- doxycycline or azithromycin Abdominal aortic Sudden onset of abdominal pain Patients who are haemodynamically Unstable patients should undergo
  • 59. aneurysm (ruptured) radiating to the back in older adults (look for risk factors). Collapse. May be moribund on arrival in casualty, more stable if contained haematoma. Careful clinical assessment may reveal pulsatile mass. stable should have a CT scan immediate surgery (unless it is not in their best interests). Those with evidence of contained leak on CT should undergo immediate surgery Increasing aneurysmal size is an indication for urgent surgical intervention (that can wait until the next working day) Perforated peptic ulcer Sudden onset of pain (usually epigastric). Often preceding history of upper abdominal pain. Soon develop generalised abdominal pain. On examination may have clinical evidence of peritonitis. Erect CXR may show free air. A CT scan may be indicated where there is diagnostic doubt Laparotomy (laparoscopic surgery for perforated peptic ulcers is both safe and feasible in experienced hands) Intestinal obstruction Colicky abdominal pain and vomiting (the nature of which depends on the level of the obstruction). Abdominal distension and constipation (again depending upon site of obstruction). Features of peritonism may occur where local necrosis of bowel loops is occurring. A plain abdominal film may help with making the diagnosis. A CT scan may be useful where diagnostic uncertainty exists In those with a virgin abdomen and lower and earlier threshold for laparotomy should exist than in those who may have adhesional obstruction Mesenteric infarction Embolic events present with sudden pain and forceful evacuation. Acute on chronic events usually have Arterial pH and lactate Arterial phase CT scanning is the most sensitive test Immediate laparotomy and resection of affected segments, in acute embolic events SMA embolectomy may be needed.
  • 60. a longer history and previous weight loss. On examination the pain is typically greater than the physical signs would suggest. A. Peritoneal lavage with cetrimide following elective right hemicolectomy B. Use of a laparoscopic approach over open surgery C. Use of talc to coat surgical gloves D. Performing a Nobles plication of the small bowel E. Using stapled rather than a hand sewn anastamosis Laparoscopy results in fewer adhesions. When talc was used to coat surgical gloves it was a major cause of adhesion formation and withdrawn for that reason. A Nobles plication is an old fashioned operation which has no place in the prevention of adhesion formation. Use of an anastamotic stapling device will not influence the development of adhesions per se although clearly an anastamotic leak will result in more adhesion formation Surgical complications Complications occur in all branches of surgery and require vigilance in their detection. In many cases anticipating the likely complications and appropriate avoidance will minimise their occurrence. For the purposes of the MRCS the important principles to appreciate are:  The anatomical principles that underpin complications  The physiological and biochemical derangements that occur  The most appropriate diagnostic modalities to utilise  The principles which underpin their management This is clearly a very broad area and impossible to cover comprehensively. There is considerable overlap with other topic areas within the website. Avoiding complications Some points to hopefully avert complications:
  • 61.  World Health Organisation checklist- now mandatory prior to all operations  Prophylactic antibiotics - right dose, right drug, right time.  Assess DVT/ PE risk and ensure adequate prophylaxis  MARK site of surgery  Use tourniquets with caution and with respect for underlying structures  Remember the danger of end arteries and in situations where they occur avoid using adrenaline containing solutions and monopolar diathermy.  Handle tissues with care- devitalised tissue serves as a nidus for infection  Be very wary of the potential for coupling injuries when using diathermy during laparoscopic surgery  The inferior epigastric artery is a favourite target for laparoscopic ports and surgical drains! Anatomical principles Understanding the anatomy of a surgical field will allow appreciation of local and systemic complications that may occur. For example nerve injuries may occur following surgery in specific regions the table below lists some of the more important nerves to consider and mechanisms of injury Nerve Mechanism Accessory Posterior triangle lymph node biopsy Sciatic Posterior approach to hip Common peroneal Legs in Lloyd Davies position Long thoracic Axillary node clearance Pelvic autonomic nerves Pelvic cancer surgery Recurrent laryngeal nerves During thyroid surgery Hypoglossal nerve During carotid endarterectomy Ulnar and median nerves During upper limb fracture repairs These are just a few. The detailed functional sequelae are particularly important and will often be tested. In addition to nerve injuries certain procedures carry risks of visceral or structural injury. Again some particular favourites are given below: Structure Mechanism Thoracic duct During thoracic surgery e.g. Pneumonectomy, oesphagectomy Parathyroid glands During difficult thyroid surgery Ureters During colonic resections/ gynaecological surgery Bowel perforation Use of Verres Needle to establish pneumoperitoneum Bile duct injury Failure to delineate Calots triangle carefully and careless use of diathermy Facial nerve Always at risk during Parotidectomy
  • 62. Tail of pancreas When ligating splenic hilum Testicular vessels During re-do open hernia surgery Hepatic veins During liver mobilisation Again many could be predicted from the anatomy of the procedure. Physiological derangements A very common complication is bleeding and this is covered under the section of haemorrhagic shock. Another variant is infection either superficial or deep seated. The organisms are covered under microbiology and the features of sepsis covered under shock. Do not forget that immunocompromised and elderly patients may present will atypical physiological parameters. Selected physiological and biochemical issues are given below: Complication Physiological/ Biochemical Problem Arrhythmias following cardiac surgery Susceptibility to hypokalaemia (K+ <4.0 in cardiac patients) Neurosurgical electrolyte disturbance SIADH following cranial surgery causing hyponatraemia Ileus following gastrointestinal surgery Fluid sequestration and loss of electrolytes Pulmonary oedema following pneumonectomy Loss of lung volume makes these patients very sensitive to fluid overload Anastamotic leak Generalised sepsis causing mediastinitis or peritonitis depending on site of leak Myocardial infarct May follow any type of surgery and in addition to direct cardiac effects the decreased cardiac output may well compromise grafts etc. Try making a short list of problems and causes specific to your own clinical area. Diagnostic modalities Depends largely on the suspected complication. In the acutely unwell surgical patient the following baseline investigations are often helpful:  Full blood count, urea and electrolytes, C- reactive protein (trend rather than absolute value), serum calcium, liver function tests, clotting (don't forget to repeat if on-going bleeding)  Arterial blood gases  ECG (+cardiac enzymes if MI suspected)  Chest x-ray to identify collapse/ consolidation  Urine analysis for UTI
  • 63. These will often identify the most common complications. Special tests  CT scanning for identification of intra-abdominal abscesses, air and if luminal contrast is used an anastamotic leak  Gatrograffin enema- for rectal anastamotic leaks  Doppler USS of leg veins- for identification of DVT  CTPA for PE  Sending peritoneal fluid for U+E (if ureteric injury suspected) or amylase (if pancreatic injury suspected)  Echocardiogram if pericardial effusion suspected post cardiac surgery and no pleural window made. Management of complications The guiding principal should be safe and timely intervention. Patients should be stabilised and if an operation needs to occur in tandem with resuscitation then generally this should be of a damage limitation type procedure rather than definitive surgery (which can be more safely undertaken in a stable patient the following day). Remember that recent surgery is a contra indication to thrombolysis and that in some patients IV heparin may be preferable to a low molecular weight heparin (easier to reverse). As a general rule laparotomies for bleeding should follow the core principle of quadrant packing and then subsequent pack removal rather than plunging large clamps into pools of blood. The latter approach invariable worsens the situation is often accompanied by significant visceral injury particularly when done by the inexperienced. If packing controls a situation it is entirely acceptable practice to leak packs in situ and return the patient to ITU for pack removal the subsequent day. Theme: Abdominal pain A. Appendicitis B. Threatened miscarriage C. Ectopic pregnancy D. Irritable bowel syndrome E. Mittelschmerz F. Pelvic inflammatory disease G. Adnexial torsion H. Endometriosis I. Degenerating fibroid Please select the most likely cause of abdominal pain for the clinical scenario given. Each option may be used once, more than once or not at all.